Você está na página 1de 215

1

14ª edição do Programa de Preparação - UFRJ Consulting Club Apostila

Sumário
Sumário 2

Introdução 5

1. Currículo, Cover Letter e vídeo 6


1.1. Currículo 6
1.1.1. Introdução 6
1.1.2. Boas práticas 6
1.1.3. Mas práticas 7
1.1.4. Formato 8
1.1.5. Estrutura 9
1.2. Cover Letter 14
1.2.1. Introdução 14
1.2.2. Boas práticas 15
1.2.3. Mas práticas 16
1.2.4. Formato 16
1.2.5. Estrutura 16
1.2.6. Exemplo Cover Letter 17
1.3. Vídeo 19
1.3.1. Conceito 19
1.3.2. Formato 19

2. GMAT 20
2.1. Introdução 20
2.1.1. Importância para os processos seletivos de consultoria 21
2.1.2. Provas das principais consultorias 21
2.2. Critical Reasoning 22
2.2.1. Introdução 22
2.2.2. Estratégias 24
2.3. Data Sufficiency 27
2.3.1. Introdução 27
2.3.2. Estratégias 28
2.4. Problem Solving 30
2.4.1. Introdução 30
2.4.2. Estratégias 30
2.5. Dicas gerais 32
2.5.1. Antes da prova 32
2.5.2. Durante a prova 33
2.6. Solve (Imbellus) 33
2.7. Revisão de conceitos matemáticos 35
2.7.1. Fórmulas 35

2
14ª edição do Programa de Preparação - UFRJ Consulting Club Apostila

2.7.2. Tipos de problemas 36


2.7.3. Pontos de atenção 37
2.8. Exemplos com resolução comentada 38
2.8.1. Critical Reasoning 38
2.8.2. Data Sufficiency 48
2.8.3. Problem Solving 57
2.9. Exercícios extras 64
2.9.1. Critical Reasoning 64
2.9.2. Data Sufficiency 73
2.9.3 Problem Solving 81
2.10. Conclusão 86

3. Business Case 87
3.1 O que é? 87
3.1.1. Definição 87
3.1.2. O que ela busca avaliar 87
3.1.3. Como se preparar? 88
3.1.4. Como ela é aplicada em cada uma das grandes empresas de consultoria
estratégica? 88
3.2. Como resolver a prova 89
3.2.1. Entender o que de fato foi perguntado 90
3.2.2. Termos de business 90
3.2.3. Análise gráfica 97
3.2.4. Tipos de gráfico 100
3.3. Tipos de pergunta 104
3.4. Exercícios 112
3.4.1. WFGLA Case 112
3.4.2. Challenge Airlines Case 114
3.4.3. Imagine Learning Case 118
3.4.4. Basketball League Case 119
3.4.5. Comfort Taxi Case 120
3.4.6. TravelCo Case 121
3.4.7. Kosher Franks 124
3.4.8. Expensive Oil 125
3.5. Gabaritos 126
3.5.1. WFGLA Case 126
3.5.2. Challenge Airlines Case 128
3.5.3. Imagine Learning Case 133
3.5.4. Basketball League Case 134
3.5.5. Comfort Taxi Case 135
3.5.6. TravelCo Case 136
3.5.7. Kosher Franks 139

3
14ª edição do Programa de Preparação - UFRJ Consulting Club Apostila

3.5.8. Expensive Oil 140

4. Fit Interview 142


4.1. Introdução 142
4.1.1. O que é Fit Interview? 142
4.1.2. Como Funciona? 142
4.1.3. As Big 3 142
4.2. Características procuradas em um candidato 144
4.3. Técnica do questionamento 146
4.4. Perguntas mais frequentes 147
4.4.1. Me conte mais sobre a sua trajetória 148
4.4.2. Por que consultoria? 149
4.4.3. Por quê essa empresa? 151
4.4.4. Você tem alguma pergunta pra mim? 153
4.5. Recursos de comunicação - Método STAR 154
4.5.1. Situação 154
4.5.2. Tarefa 155
4.5.3. Ação 155
4.5.4. Resultado 155
4.6. Como se portar 157
4.6.1. Linguagem 157
4.6.2. Vestimenta 160
4.7. Dicas finais 160
4.8. Como se preparar? 161
4.9. Lista de perguntas 162
4.9.1. Exemplos para praticar 162
4.9.2. As perguntas mais difíceis 163

5. Case Interview 164


5.1. Introdução 164
5.1.1. O que é a Case Interview? 164
5.1.2. Tipos de Aplicação 164
5.1.3.Habilidades Requeridas 166
5.1.4. Categorias de case 168
5.2. Resolução de um case 172
5.2.1. Início 172
5.2.2. Estruturação 177
5.2.3. Desenvolvimento 182
5.2.4. Recomendação, Riscos e Próximos Passos 186
5.3. Aprofundando o desenvolvimento 190
5.3.1. Introdução 190
5.3.2. Brainstorming 190

4
14ª edição do Programa de Preparação - UFRJ Consulting Club Apostila

5.3.3. Frameworks 195


5.3.4. Estimate 201
5.3.5. Interpretação de gráficos 206
5.4. Dicas gerais 213

Conclusão 216

Introdução
Ser consultor é ser desafiado todos os dias. Trabalhando para resolver os maiores
problemas das empresas mais influentes do país e do mundo, percorrendo as mais
diversas indústrias e aconselhando os mais experientes executivos, a necessidade de
conhecimento é inquestionável. É preciso ter grandes habilidades de capacidade
analítica, raciocínio lógico, comunicação e organização, além de versatilidade para se
adaptar rapidamente às mudanças de cenário.

É de se esperar, portanto, que os processos de seleção de novos consultores sejam


extremamente rigorosos. Por meio de etapas como provas, jogos, entrevistas pessoais e
resoluções de problemas de negócios, os recrutadores realizam testes bastante
específicos, desenhados para avaliar as competências exigidas diariamente ao ingressar
em uma consultoria.

Assim, entrar para esse time requer, acima de tudo, dedicação e esforço para estudar e
desenvolver o conhecimento e as habilidades necessárias. Mais do que estar se
preparando para passar nos processos seletivos, você estará desenvolvendo as
características indispensáveis para que você seja um profissional de alto desempenho.

Sendo assim, para auxiliar você nessa jornada de aprendizado e capacitação, nós do
UFRJ Consulting Club, criamos o Programa de Preparação. Por meio das aulas
teóricas, das atividades práticas, desta apostila com conteúdo exclusivo, dos eventos
com as consultorias parceiras e da interação entre alunos que compartilham deste
mesmo sonho, esperamos que você seja capaz de atingir seu objetivo de se tornar um
excelente consultor.

5
14ª edição do Programa de Preparação - UFRJ Consulting Club Apostila

1. Currículo, Cover Letter e vídeo

1.1. Currículo

1.1.1. Introdução

O Curriculum Vitae (CV), ou Currículo, é o documento que apresenta uma síntese


dos dados pessoais, formação acadêmica, experiências profissionais e outras
qualificações de um candidato a uma vaga.

Por passar a primeira impressão do candidato ao avaliador, o currículo funciona


como um "cartão de visitas", induzindo o leitor a se interessar mais por ele e por
suas experiências. Por isso, o currículo deve ser convidativo e interessante,
destacando as informações relevantes para o cargo, as quais serão abordadas em
detalhes mais para frente.

Apesar de um bom currículo não garantir sozinho uma vaga de emprego, um CV


mal estruturado pode resultar na perda de boas oportunidades, visto que uma
parcela significativa dos candidatos deixa de ser aprovado para etapas
posteriores por enviar um currículo pouco desenvolvido. Portanto, neste módulo,
será apresentado como estruturar um currículo de excelência.

1.1.2. Boas práticas

Nesse momento, serão apresentadas algumas práticas que podem agregar mais
valor ao seu currículo e gerar uma boa impressão ao avaliador.

Considerando que é essencial passar para o leitor o impacto do candidato em


suas iniciativas, uma maneira muito eficaz de transmitir isso é quantificando suas
conquistas, mostrando em números o que o candidato agregou à experiência.
Dessa forma, qualquer dado quantitativo pode ajudar a reforçar a mensagem
que o candidato deseja passar. Por exemplo:

● Liderança de uma equipe para traçar estratégias de aumento de


lucratividade, elevando as margens operacionais da empresa em 30%.

6
14ª edição do Programa de Preparação - UFRJ Consulting Club Apostila

● Conquista do 1° lugar num desafio estratégico sobre o setor de veículos


elétricos com 1000 participantes.

Outra boa prática seria mostrar variedade. Caso o indivíduo tenha muitas
experiências e precise optar por alguma delas, dada a importância de se
apresentar um currículo resumido, é aconselhável que o candidato tente
diversificar ao máximo, escolhendo pontos que mostrem diferentes qualidades,
habilidades e conhecimentos.

Por fim, tente, ao máximo, usar palavras-chaves que se encaixam com o


cotidiano das grandes firmas de consultoria. Alguns exemplos são: liderança,
proatividade, gestão, organização, destaque, iniciativa, desenvolvimento, criação,
entre outros.

1.1.3. Mas práticas

Em seguida, vale pontuar três más práticas que, em geral, não são bem vistas
pelos recrutadores e podem prejudicar o candidato. Todas elas abordam a
linguagem das descrições, que deve ser objetiva, clara, específica e, sobretudo,
ativa e protagonista.

A primeira má prática é usar frases muito longas para descrever as experiências,


usando, sobretudo, um excesso de construções verbais em gerúndio, o que
atrapalha a fluidez da leitura. Um bom parâmetro é evitar passar de duas linhas
por descrição, prezando sempre pela objetividade.

Não é recomendado escrever as experiências de forma vaga, abrangente e


subjetiva. No lugar disso, é interessante expandir a questão de quantificar
conquistas para detalhar as experiências, abordada no tópico de boas práticas,
tornando mais claro o que exatamente o candidato fez. Observe um exemplo
dessa má prática:

● Desenvolvimento de vários aplicativos, em paralelo à gestão de uma


grande equipe.

Perceba como não fica claro quantos aplicativos foram desenvolvidos ou quantas
pessoas havia na equipe, de modo a não atribuir valor ao recrutador.

7
14ª edição do Programa de Preparação - UFRJ Consulting Club Apostila

Por fim, como terceira má prática, não seja o personagem secundário da sua
experiência. Por mais que não seja recomendado utilizar a primeira pessoa no
currículo, e sim uma linguagem mais impessoal, como será visto nos próximos
exemplos, é muito importante que as descrições mostrem o candidato como
alguém ativo e relevante dentro da atividade. Nesse sentido, é essencial evitar
palavras que não demonstram protagonismo, como "ajuda", "auxílio" e
"colaboração", e, no lugar delas, vale utilizar as palavras-chave apresentadas
anteriormente nas boas práticas.

1.1.4. Formato

Agora que já foram introduzidos os conceitos de Curriculum Vitae e sua


importância, vale fazer alguns apontamentos sobre seu formato e como ele deve
ser estruturado em termos estéticos.

Em primeiro lugar, caso a empresa não especifique, envie seu currículo em PDF,
pois, assim, é possível garantir a manutenção da formatação original do
documento.

Não existe uma única resposta correta para a fonte do documento, porém é
importante utilizar uma opção simples e básica. Assim, o uso de fontes
rebuscadas é mal visto pelos entrevistadores, tendo em vista que são mais
difíceis de ler e podem indicar falta de profissionalismo do candidato. Algumas
opções para utilizar em seu currículo são: Arial, Times New Roman, Verdana e
Calibri.

A letra deve ser sempre na cor preta ou, no máximo, incluir alguma cor de
destaque que seja visível e séria, utilizando, de preferência, o negrito, que, por
conta própria, consegue dar o destaque necessário para as palavras. Bons
exemplos são azul marinho, verde escuro e vinho. Além disso, evite usar cores
de empresas de consultorias estratégicas diferentes daquela para a qual o CV
está sendo enviado.

Para a letra, é interessante um tamanho não muito pequeno, de forma a não


impedir a leitura por parte do entrevistador, mas, ao mesmo tempo, não muito
grande para não ocupar muito espaço. Portanto, é recomendado usar uma letra
entre 10 e 12.

8
14ª edição do Programa de Preparação - UFRJ Consulting Club Apostila

É importante que o currículo seja esteticamente agradável. Por isso, não é


indicado sobrecarregá-lo com informações visuais, visto que um documento
criativo e colorido não é procurado por firmas de consultoria estratégica, já que
pode passar uma imagem informal do candidato. O importante é ele estar bem
estruturado e passar objetivamente as experiências do candidato, utilizando
uma estética simples.

Não é recomendado inserir fotos no currículo. Caso seja requisitado pela firma,
utilize uma foto que apareça apenas o candidato. Nesse caso, é bem visto usar
roupa formal, tirada em fundo liso e mostrar a região apenas acima dos ombros.

Outros pontos que garantem uma organização estética e que podem estar
presentes dentro do CV são: a utilização de bullet points, para que o
entrevistador tenha clareza da separação dos conteúdos; espaçamento,
preferencialmente entre 1 e 1.5cm; formatação uniforme ao longo de todo o CV;
texto justificado; utilização de margens entre 1 e 1,5cm.

O currículo deve conter apenas 1 página, pois textos longos desviam a atenção
do leitor, provocando um menor interesse no conteúdo lido. Nesse sentido, é
necessário lembrar que o avaliador estará corrigindo diversos currículos ao
mesmo tempo, ou seja, textos reduzidos agradam mais e evitam que
informações relevantes passem despercebidas em meio a muitas outras. Outro
ponto importante é que os currículos devem conter informações resumidas
sobre o candidato e a vaga desejada. Portanto, currículos muito extensos podem
indicar que algumas informações contidas são desnecessárias, pois o candidato
não teve a capacidade de sintetizá-las.

1.1.5. Estrutura

O CV é dividido em seções que, em conjunto, formam sua estrutura completa.


Cada seção contará com experiências, que devem ser, por meio de uma
formatação organizada, separadas uma da outra. Dentro de cada segmentação
dessa, devem adicionar informações essenciais, como: local, duração (começo e
fim), cargo e descrição da atividade (mais a frente será explicado como fazer uma
boa descrição).

9
14ª edição do Programa de Preparação - UFRJ Consulting Club Apostila

Além disso, é importante escrever seus cargos e conquistas em ordem


cronológica dentro de cada seção, onde a primeira deve ser aquela que teve
início mais recente. Isso é aplicável, principalmente, para experiências
profissionais, acadêmicas e extracurriculares. O restante das informações de
outras seções podem ser apresentadas em ordem de relevância para o cargo
desejado.

Nesse material, abordaremos uma estrutura bastante utilizada, porém, de forma


alguma, rígida ou obrigatória: é possível usar diferentes seções ou alterar a
ordem delas conforme o que fizer mais sentido para destacar as experiências do
candidato.

As seções indicadas para montar um currículo são: informações pessoais,


experiências acadêmicas, experiências profissionais, experiências
extracurriculares e informações complementares. Normalmente, o currículo
começa com os tópicos “informações pessoais, experiências acadêmicas e
experiências profissionais” nessa mesma ordem, porém os outros pontos são
alteráveis.

● Informações pessoais

○ As informações necessárias nesse tópico são: nome completo,


telefone, email e cidade/estado. Além disso, apesar de não ser
obrigatório, o LinkedIn seria uma informação interessante a ser
adicionada, caso esteja atualizado e seja estritamente profissional
para não passar a imagem indesejada.

○ Lembre-se de colocar um email usado diariamente, para que


nenhuma informação sobre os resultados do processo seletivo
sejam perdidas. Além disso, utilize emails estritamente profissionais,
que não contenham apelidos.

○ Caso seja utilizado o LinkedIn, altere o URL para ficar com seu
nome. Isso demonstrará que o candidato teve um cuidado maior ao
colocar seus dados no currículo. Para aprender a fazer isso, basta
acessar esse link.

● Formação acadêmica

10
14ª edição do Programa de Preparação - UFRJ Consulting Club Apostila

○ Essa seção consiste nas informações básicas sobre a educação


formal.

○ Nela, é indicado incluir as instituições acadêmicas pelas quais o


candidato passou, informando o nome, o curso, o título (graduação,
pós graduação, bacharelado…), local (caso seja numa cidade ou país
que não seja facilmente deduzido pelo nome da universidade) e
duração dos estudos na instituição.

○ Esse é o setor do currículo adequado para adicionar informações


sobre formação escolar. Nesse sentido, não há problema em incluir
o ensino médio, sobretudo para candidatos com poucas
experiências profissionais e ainda no início ou meio da primeira
graduação. No entanto, para candidatos já formados na
universidade, pode não fazer tanto sentido utilizar espaço para
incluir a escola, dada a distância temporal e a maior quantidade de
experiências que possam agregar mais ao serem abordadas no CV.

○ Nessa seção, é muito interessante incluir experiências de


intercâmbio acadêmico, caso o candidato tenha feito algum. Vale
ressaltar que, caso o intercâmbio não tenha envolvido nada
acadêmico, como uma aula ou curso, não é interessante
mencioná-lo nessa seção, mas sim em experiências
extracurriculares.

○ Além de colocar as instituições e cursos, é possível aprimorar essa


informação com notas, ou seja, adicionar o Coeficiente de
Rendimento Acumulado (CRA) ou GPA, mas apenas se fizer sentido
e for agregador ao currículo.

● Experiências profissionais

○ Essa seção aborda as experiências no mercado de trabalho,


tornando necessário incluir empregos e/ou estágios, lembrando de
adicionar informações como local, período e descrição. Além disso,
é possível adicionar outros tipos de experiências, desde que sejam
pertinentes ao mercado de trabalho e ao ramo da consultoria

11
14ª edição do Programa de Preparação - UFRJ Consulting Club Apostila

estratégica, como, por exemplo, a participação em uma empresa


júnior na universidade.

● Experiências extracurriculares

○ Essa seção do currículo já é mais abrangente em comparação às


apresentadas anteriormente. Nesse momento, o candidato pode
incluir qualquer experiência com um viés acadêmico que tenha
tido iniciativa em participar. É importante lembrar que essas
experiências, idealmente, devem estar alinhadas com o cargo
buscado e, também, com consultoria estratégica.

○ Dentre as possibilidades nesse tópico, podemos citar: iniciação


científica, bolsas de estudo, entidades estudantis, competições,
prêmios, medalhas de olimpíadas, monitorias, trabalhos voluntário,
intercâmbios, cursos, desafios, empresas juniores, clube de
consultoria, ligas acadêmicas, capítulos estudantis, entre outros.

○ Dentro de cada uma dessas experiências, deve-se fazer um maior


detalhamento, como visto anteriormente. Portanto, é necessário
adicionar qual era o cargo dentro dessa experiência, a duração e,
principalmente, a explicação do que foi feito.

○ Bolsas de estudos podem ser inseridas tanto em formação


acadêmica, quanto em experiências extracurriculares, cabendo ao
candidato adicioná-las ou não, pois não existe nenhuma regra sobre
isso. O mesmo pode ser dito para a Empresa Júnior, podendo estar
dentro de experiências profissionais ou experiências
extracurriculares.

● Informações adicionais

○ Nessa seção, os aspectos mais relevantes para serem abordados são


os cursos, treinamentos e idiomas.

○ Quando se trata de idioma, é interessante começar informando o


idioma nativo e depois passar para outros. Nessa parte, é essencial
apresentar aspectos quantitativos que representam seu nível em
alguma experiência, adicionando certificados ou diplomas. Por

12
14ª edição do Programa de Preparação - UFRJ Consulting Club Apostila

exemplo, no caso do inglês, podem ser incluídos certificados de


provas oficiais, com as provas de Cambridge e o TOEFL. Também é
possível incluir aspectos qualitativos, adicionando um nível básico,
intermediário, avançado ou fluente.

○ Com relação a cursos e treinamentos realizados, o principal fator é


ser um curso alinhado com o cargo que deseja conquistar dentro
de uma firma de consultoria. Dentre eles podemos citar: curso de
Excel, curso de Power BI, Gestão de projetos, Gestão de pessoas,
entre outros.

1.1.6. Exemplo de currículo

13
14ª edição do Programa de Preparação - UFRJ Consulting Club Apostila

14
14ª edição do Programa de Preparação - UFRJ Consulting Club Apostila

1.2. Cover Letter

1.2.1. Introdução

A Cover Letter (CL) é uma carta de apresentação complementar ao currículo,


que, em geral, pode ou não ser enviada - no Brasil, não é muito comum ser
exigido o envio.

As principais firmas de consultoria estratégica, como Bain e Mckinsey, não


necessitam da avaliação de uma Cover Letter, apenas o currículo. Entretanto, o
BCG solicita de seus candidatos uma CL, apenas se eles forem reincidentes no
processo seletivo - nesse caso, o candidato deve ter chegado à fase de
entrevistas, mas sido reprovado em uma delas (seja nos firsts ou nos finals, que
serão vistos no módulo de Fit Interview). Desse modo, o candidato, por meio de
sua CL deverá, também, ressaltar quais foram as mudanças, nesse período, de
forma prática.

Nesse documento, será possível explorar objetivos profissionais e o interesse por


consultoria, assim como motivações pessoais, atrelando isso às experiências
vividas e às habilidades desenvolvidas. No caso de candidatos reincidentes do
BCG, é muito importante mostrar os pontos de melhora em relação ao processo
seletivo anterior de modo prático, levando exemplos que evidenciem essa
evolução. Por fim, é essencial conectar tudo isso à cultura da empresa, sendo
personalizada para cada instituição.

1.2.2. Boas práticas

No geral, as boas práticas da Cover Letter se aproximam das do currículo,


sobretudo na questão da quantificação de conquistas, na clareza dos termos e
na objetividade. Além dessas, outras duas boas práticas seriam: exemplificar
melhorias e utilizar palavras-chave que remetem à cultura da empresa.

Para deixar claro o impacto provocado pelo candidato, é interessante


exemplificar melhorias. Portanto, é necessário mostrar de forma prática, por
meio de experiências, como foi o processo de evolução em relação aos feedbacks
recebidos, sobretudo quando abordamos candidatos que fizeram o processo
seletivo do BCG e não obtiveram êxito. Nesse sentido, vale correlacionar os
feedbacks a exemplos de atividades práticas realizadas para melhorá-los.

15
14ª edição do Programa de Preparação - UFRJ Consulting Club Apostila

Além disso, para que o texto fique alinhado à cultura da empresa, é essencial
estudar a firma antes de começar a escrever sua Cover Letter. Procure saber mais
sobre as principais características que essas empresas estão buscando nos
candidatos, para que seja possível usar palavras-chave que remetem a aspectos
importantes da cultura e do perfil dos consultores.

1.2.3. Mas práticas

Em termos de más práticas, a Cover Letter também tem aspectos linguísticos a


serem evitados que se assemelham aos do currículo, com destaque para o uso
de termos vagos. Porém, existe um erro particular e importante ligado ao caráter
mais pessoal da Cover Letter, cometido por muitos candidatos, que é a
justificativa de resultados negativos. Nesse sentido, é essencial pular explicações
que possam soar como "desculpas" por erros cometidos previamente, e, no lugar
disso, focar em descrever o que vem sendo feito de forma prática para corrigir
esses erros. Isso é muito importante para candidatos reincidentes do BCG, que
devem, idealmente, escolher pelo menos uma experiência capaz de exemplificar
como foi desenvolvida uma habilidade tida como fundamental, mas que faltou
no candidato em um processo seletivo anterior.

1.2.4. Formato

Assim como o Currículo, a Cover Letter deve ser enviada em PDF, usar fontes
clássicas, ter letra preta, conter apenas uma página e não apresentar
informações visuais exageradas. Entretanto, diferentemente do CV, ela possui a
estrutura de um texto com introdução, desenvolvimento e conclusão. Por esse
motivo, indica-se utilizar letras no padrão acadêmico, dando preferência para o
tamanho 11 ou 12. Além desses pontos, um aspecto essencial que a diferencia do
CV é o uso da primeira pessoa. Assim, é possível criar um texto mais pessoal e
argumentativo, descrevendo seu histórico e motivações.

1.2.5. Estrutura

Como abordado no formato, a Cover Letter se divide em 3 partes: introdução,


desenvolvimento e conclusão, cada uma com conteúdos a serem idealmente
abordados.

● Introdução

16
14ª edição do Programa de Preparação - UFRJ Consulting Club Apostila

Essa parte deve conter apenas o nome e sobrenome na primeira linha,


juntamente com o email. Em seguida, na segunda linha, deve conter
apenas a data de envio. E na terceira e última linha da introdução, o cargo
pelo qual o candidato está aplicando.

● Desenvolvimento

Essa seção deve conter 3 parágrafos, e deve abordar as realizações e


vivências da trajetória do candidato, respondendo a 3 perguntas, uma em
cada parágrafo:

● Por que sou um bom candidato?


● Por que a consultoria é o encaixe perfeito para mim?
● Por que estou me candidatando a essa posição específica?

Portanto, é válido buscar experiências que se encaixem nas respostas


dessas 3 perguntas, criando uma conexão com a empresa.

● Conclusão

Nesse único parágrafo de conclusão, é importante deixar claro como o


candidato poderia agregar e impactar a firma com suas experiências,
qualidades e qualificações. Por último, é válido agradecer o tempo
disponibilizado e incluir uma despedida com algum termo como
“Atenciosamente”, ”Cordialmente” ou ”Com os melhores comprimentos”.

1.2.6. Exemplo Cover Letter

Maria dos Reis - mariadosreis@gmail.com

25/02/2023

Prezado Pedro Almeida,

Eu gostaria de me candidatar para a vaga de Business Analyst no BCG


do Rio de Janeiro.

Sou formada em Engenharia Química na UFRJ e, simultaneamente à


universidade, no meu segundo período, me tornei membro do UFRJ
Consulting Club. Atualmente, sou gerente de processos da empresa Café
da Vila há dois anos, onde sou responsável por liderar a definição do
planejamento estratégico para o aumento das vendas. Essa experiência

17
14ª edição do Programa de Preparação - UFRJ Consulting Club Apostila

me fez desenvolver um maior pensamento analítico, além de possibilitar


que eu demonstrasse muita iniciativa e capacidade de inovação, uma
vez que era esperado que eu trouxesse resultados de aumento de receita
para a empresa, o que, logo no primeiro semestre de trabalho, foi
possível. Sou fluente em inglês e alemão, habilidades que me qualificam
para trabalhar no escritório do BCG em Berlim, que é um grande objetivo
pessoal e profissional meu.

Durante os 10 meses em que participei da iniciativa do Consulting Club,


iniciei meu contato com o mercado de trabalho e conheci mais de perto
a consultoria estratégica por meio da liderança de projetos reais para
empresas como a Americanas e a Eleva Educação. Durante esse período,
também tive a oportunidade de participar de 3 eventos com o BCG e
conversar diretamente com consultores sobre projetos que eles fizeram
envolvendo Data Science, o que despertou meu interesse pela área e
fomentou ainda mais meu objetivo de conquistar uma vaga na firma.​
Após o último processo de seleção do BCG do qual participei, me
dediquei a desenvolver mais habilidades de relacionamento interpessoal,
em busca de responder a esse feedback recebido. Para que isso fosse
possível, fiz dois cursos online sobre liderança e gestão de pessoas da
University of Chicago e da University of Oxford, nos quais obtive bolsas de
estudo por destaque entre mais de 200 alunos de todo mundo, e isso me
permitiu adquirir uma posição de destaque em meu estágio após um
semestre de trabalho.

O BCG me atrai por duas razões principais. Primeiro, devido a minha


grande vontade de trabalhar em um ambiente diverso, com pessoas do
mundo inteiro, e, além disso, pela alta experiência dos consultores,
sobretudo em assuntos que me despertam interesse, como análise de
dados e inteligência artificial com o BCG Gamma. Desse modo, acredito
que, ao me juntar à firma, teria a oportunidade de aprender de forma
direta e prática com essas pessoas, de forma a crescer pessoal e
profissionalmente.

Com isso, acredito que a minha trajetória acadêmica, focada em


consultoria, minhas experiências de trabalho em questões estratégicas e

18
14ª edição do Programa de Preparação - UFRJ Consulting Club Apostila

minhas habilidades de liderança e gestão possam ser uma grande


contribuição para o BCG, dada a evolução prática que tive desde o
último processo seletivo. Agradeço cordialmente pelo tempo
disponibilizado e estou ansiosa para conversarmos sobre a possibilidade
de realizar uma entrevista em breve.

1.3. Vídeo

1.3.1. Conceito

Em algumas consultorias, como o BCG, uma das etapas iniciais exige o envio de
um vídeo, buscando avaliar a desenvoltura e comunicação do candidato. Em
geral, as firmas fornecem comandos específicos do que o vídeo deve conter - no
caso do BCG, é solicitado que o candidato responda duas perguntas.
Normalmente, a primeira pergunta é sobre o que mais te chama atenção ou
sobre a sua identificação com a cultura da empresa, enquanto a segunda varia
de processo para processo. O foco da avaliação não está exatamente no
conteúdo da resposta, mas sim na comunicação, e, por isso, focaremos em dar
dicas nesse âmbito.

1.3.2. Formato

Nesse tópico, serão abordadas boas práticas para a realização do vídeo de


processos seletivos, que podem melhorar a imagem do candidato.

● Aspectos técnicos

Os aspectos técnicos são aqueles ligados ao ambiente e às condições de


filmagem. Portanto, para garantir que o vídeo fique bem formatado, é
importante ficar atento a alguns pontos. Começando pela iluminação, é
essencial que o avaliador consiga ver o candidato com clareza. Para isso, se
possível, aproveite a luz natural e busque um fundo liso ou um ambiente
com aspecto mais profissional e organizado, como uma estante de livros,
sem outros indivíduos e ruídos. Em seguida, passando para um ponto
mais ligado às condições de filmagem, deve-se gravar o vídeo com a tela

19
14ª edição do Programa de Preparação - UFRJ Consulting Club Apostila

na horizontal e buscar um apoio, evitando que a câmera se movimente ao


longo do vídeo.

● Postura

O modo como o candidato se apresenta visualmente e transmite a


mensagem para o avaliador pode causar boas impressões em termos de
comunicação, que é o foco da avaliação. Por isso, é muito importante ter
uma postura adequada e uma boa linguagem corporal. Para garantir isso,
vista roupas formais, assim como usaria em uma entrevista presencial,
pois trará uma imagem mais profissional do candidato. Além disso,
procure ficar com a postura ereta, não gesticular excessivamente, fixar o
olhar na câmera, evitar vícios de linguagem e não decorar ou ler um texto,
para que a fala fique natural.

2. GMAT

2.1. Introdução
O GMAT, sigla para Graduate Management Admission Test, é uma prova de múltipla
escolha utilizada em grande parte do mundo como requisito para concorrer em
programas de MBA internacionais e instituições de ensino superior de negócios. No
universo da consultoria, esse teste serve como base para as provas de muitas firmas,
que aplicam nos candidatos, normalmente na segunda etapa (logo após o envio do
currículo e da cover letter) do processo seletivo, um teste analítico nos moldes do
GMAT.

Dentre os tipos de questões aproveitados do GMAT original, estão as de Critical


Reasoning, Data Sufficiency e Problem Solving, que serão explicadas detalhadamente
em próximos tópicos. A prova, toda em inglês, é feita num computador, com cada
questão apresentando 5 opções de resposta. Sua resolução é sequencial, ou seja, não
é permitido retornar às questões anteriores ao pular para a seguinte, de modo que o
controle de tempo é essencial para um bom desempenho.

No teste analítico para adentrar em firmas de consultoria, não existe um número


específico de acertos conhecidos para passar, visto que variam bastante entre as

20
14ª edição do Programa de Preparação - UFRJ Consulting Club Apostila

empresas e esses dados não costumam ser divulgados. No próximo tópico, será
abordado o porquê da utilização desse método em processos seletivos, assim como o
que é avaliado nos candidatos.

2.1.1. Importância para os processos seletivos de consultoria

O GMAT é uma prova capaz de medir diversas habilidades dos candidatos,


sobretudo o raciocínio lógico e o pensamento crítico, essenciais para a carreira de
um consultor. Por isso, é tratada pelas firmas como um processo eliminatório
importante para filtrar os candidatos melhor avaliados para as etapas seguintes
e, em geral, é onde a maior parte é eliminada. As perguntas do teste analítico
podem ser divididas em Verbal Reasoning e Quantitative Reasoning. Dentro
da primeira, estão as de Critical Reasoning, e, da segunda, as de Data Sufficiency
e Problem Solving.

Na parte verbal, serão analisadas a compreensão de texto e o raciocínio crítico


do candidato. Uma boa pontuação nesse quesito indica um raciocínio verbal
condizente com o esperado nesse tipo de trabalho, que exige a elaboração de
pesquisas, relatórios e recomendações de projetos.

As habilidades relacionadas ao raciocínio lógico e interpretação de dados são


avaliadas na parte quantitativa da prova. Um bom desempenho indica que o
candidato está preparado para lidar com números e extrair insights interessantes
deles, representando bem o cotidiano de um consultor, que, frequentemente,
precisa entender e priorizar quais informações serão necessárias, combiná-las
para obter resultados e realizar análises para chegar a uma conclusão.

Para entender melhor o que cada consultoria procura e como é o formato de


prova de cada uma, iremos nos aprofundar em suas particularidades.

2.1.2. Provas das principais consultorias

● BCG: a prova do BCG tem duração de 45 minutos e conta com 10


questões - Problem Solving e Critical Reasoning - e 10 questões de
Business Case, outro modelo de prova que será abordado no próximo
módulo do curso. É recomendado que se utilize 20 minutos para realizar a

21
14ª edição do Programa de Preparação - UFRJ Consulting Club Apostila

primeira parte da prova, referente aos exercícios semelhantes ao GMAT,


resultando em 2 minutos para cada questão.

● Bain: a prova da Bain, por sua vez, tem duração de 25 minutos e conta
com 15 questões de Problem Solving, Data Sufficiency e Critical
Reasoning, resultando em 1 minuto e meio para cada questão. Além disso,
a firma conta com um teste cognitivo - Pymetrics, que inclui 16 jogos a
serem resolvidos em 35 minutos.

● Visagio: a prova da Visagio tem duração de 75 minutos e conta com 30


questões, resultando em 2 minutos e meio para cada uma.

● L.E.K Consultoria: a prova da L.E.K. possui 25 questões para serem feitas


em 50 minutos, resultando em 2 minutos e meio para cada uma.

● Mirow: a prova da Mirow tem 30 minutos de duração e possui 21


questões, oferecendo 1 minuto e meio por questão, em média.

● McKinsey: fugindo do padrão, a McKinsey não conta com uma avaliação


nos moldes do GMAT, mas sim com uma etapa chamada Solve -
antigamente conhecido como Imbellus -, onde o candidato é avaliado por
meio de um jogo, que conta com 3 desafios para serem resolvidos em 1
hora. Ao final deste módulo, serão exploradas mais a fundo características
e dicas para um bom desempenho.

2.2. Critical Reasoning

2.2.1. Introdução

Nas perguntas de Critical Reasoning, são avaliadas competências como a


interpretação de texto e análise de argumentos e suas premissas. Para ilustrar
como esse molde de questão é estruturado e como deve ser resolvido de forma
eficiente, temos a seguinte pergunta:

Pergunta:

Archeologists working in the Andes Mountains recently excavated a buried


4,000-year-old temple containing structures that align with a stone carving on

22
14ª edição do Programa de Preparação - UFRJ Consulting Club Apostila

a distant hill to indicate the direction of the rising sun at the summer solstice.
Alignments in the temple were also found to point toward the position, at the
summer solstice, of a constellation known in Andean culture as the Fox. Since
the local mythology represents the fox as teaching people how to cultivate and
irrigate plants, the ancient Andeans may have built the temple as a religious
representation of the fox.

Which of the following is an assumption on which the argument is based?

a) The constellation known as the Fox has the same position at the summer
solstice as it did 4,000 years ago.

b) In the region around the temple, the summer solstice marks the time for
planting.

c) The temple was protected from looters by dirt and debris built up over
thousands of years.

d) Other structural alignments at the temple point to further constellations


with agricultural significance.

e) The site containing the temple was occupied for a significant amount of
time before abandonment.

Observa-se que, essas questões geralmente são formadas por textos com
menos de 100 palavras, sobre assuntos gerais, como economia, ciência ou
política. Na parte das alternativas, são 5 múltiplas escolhas entre as quais o
candidato deve determinar qual é logicamente relacionada à pergunta.

Para resolvê-las, não é necessário ter nenhum conhecimento prévio sobre os


assuntos citados acima, mas sim, estar familiarizado com os termos: premissa,
argumento, conclusão e suposição.

● Premissas: afirmativas ou proposições que procuram sustentar uma ideia


central ou uma conclusão. Fatos são, por definição, premissas irrefutáveis.
No entanto, qualquer opinião também pode ser considerada uma
premissa, que dependerá de validação ou refutação por evidências.

● Argumento: uma série de afirmativas que, a partir de certas premissas,


levam a uma conclusão.

● Conclusão: a ideia central que o argumento procura defender a partir de

23
14ª edição do Programa de Preparação - UFRJ Consulting Club Apostila

premissas e evidências. Conclusões são, normalmente, marcadas por


palavras e expressões características, como “portanto”, “assim”, “por isso” e
“consequentemente”.

● Suposições: proposta hipotética, ou seja, ponto de vista formado sem


comprovação, mas tomado como verdadeiro.

As questões podem pedir que o candidato encontre:

● A opção de resposta que apresente uma suposição da qual o argumento


dependa;

● A opção de resposta que apresente o fato que enfraqueceria o


argumento da pergunta;

● A opção de resposta que apresente o fato que fortaleceria o argumento


da pergunta;

● A opção de resposta que apresente o papel de determinadas informações


e/ou relações entre elas;

● A opção de resposta que apresenta ineficiências em planos de ações.

2.2.2. Estratégias

Iremos ver algumas dicas para resolver esse tipo de questão de Critical
Reasoning em específico:

● Comece lendo o comando e as alternativas, para depois ler o


argumento. Com isso, é possível identificar as palavras chaves da pergunta
que são relevantes para a resposta e ler o texto já sabendo o que está
sendo requisitado na questão, otimizando o tempo.

● Preste atenção às questões que requerem respostas extras, como pedir


as opções que levariam a um aumento ou diminuição de certos
resultados, tais quais custo, lucro ou demanda, por exemplo. Esse tipo de
questão demanda mais atenção e tempo para ser resolvida.

● Procure por termos como therefore, since, thereby, because, due to,
so, since, clearly, obviously, in conclusion para indicar a conclusão do
texto, e palavras como since, as, for para indiciar uma premissa ou razão.

24
14ª edição do Programa de Preparação - UFRJ Consulting Club Apostila

Muitas vezes, a partir dessas palavras específicas, pode-se encontrar a


resposta do problema.

● Atente-se à expressão “if true”, visto que, recorrentemente, os


enunciados perguntarão quais observações, se verdadeiras, enfraquecem
ou fortalecem uma conclusão. Nesses casos, não cabe ao candidato julgar
se a afirmação é factível ou não, pois o exercício já está indicando para
considerar que aquela informação é verdadeira. O que você deve fazer é
simplesmente avaliar se, dada a veracidade da afirmativa, ela de fato
influencia diretamente na conclusão em questão.

● Elimine alternativas que fogem do contexto da questão.

Aplicando as dicas mencionadas acima, seria possível resolver o exemplo


apresentado anteriormente por meio do seguinte passo a passo.

Pergunta:

Archeologists working in the Andes Mountains recently excavated a buried


4,000-year-old temple containing structures that align with a stone carving on
a distant hill to indicate the direction of the rising sun at the summer solstice.
Alignments in the temple were also found to point toward the position, at the
summer solstice, of a constellation known in Andean culture as the Fox. Since
the local mythology represents the fox as teaching people how to cultivate and
irrigate plants, the ancient Andeans may have built the temple as a religious
representation of the fox.

Which of the following is an assumption on which the argument is based?

a) The constellation known as the Fox has the same position at the summer
solstice as it did 4,000 years ago.
b) In the region around the temple, the summer solstice marks the time for
planting.
c) The temple was protected from looters by dirt and debris built up over
thousands of years.
d) Other structural alignments at the temple point to further constellations
with agricultural significance.
e) The site containing the temple was occupied for a significant amount of
time before abandonment.

25
14ª edição do Programa de Preparação - UFRJ Consulting Club Apostila

Resolução:

I. Após ter lido o enunciado e as alternativas, pode-se analisar que o comando


da questão é indicar a alternativa em que o argumento se baseia e realizar
uma leitura, procurando por respostas específicas e palavras-chaves.

II. No texto, as palavras chaves encontradas são since e may, que indicam razão
e conclusão, respectivamente, mostrando que isso é interessante para
solucionar a questão..

III. Como informações gerais do texto, temos que os construtores, para projetar o
templo, levaram em consideração a posição dos corpos no solstício de verão.
Além disso, a estrutura aponta para a posição da constelação da raposa,
tornando-a uma possível representação religiosa para comunidade andina.

IV. Inicialmente, elimina-se as alternativas que não são viáveis, como a B, pois a
suposição que o solstício de verão marca o início das plantações não foi
mencionado ao longo do texto, não sendo uma suposição obrigatória.

V. Já a alternativa C é falsa, pois não é uma suposição obrigatória do texto, tendo


em vista que não sustenta a suposição que templo é uma representação para
a raposa.

VI. A alternativa D é um caso de enfraquecimento do argumento, à medida que


o templo é construído para destacar a agricultura e não a constelação da
raposa, como o texto menciona. Assim, essa opção pode ser descartada.

VII. Outra alternativa não relacionada com o texto que pode ser eliminada é a E,
visto que o argumento não cita que o templo foi ocupado em nenhum
momento.

VIII. Logo, evidencia-se que a alternativa A é a verdadeira, já que ela é uma


justificativa do porquê associarem o templo com a constelação. Isso se dá
pois, caso a posição da constelação não não fosse a mesma, o templo não
estaria alinhado com ela quando foi construído.

Resposta correta: A

26
14ª edição do Programa de Preparação - UFRJ Consulting Club Apostila

2.3. Data Sufficiency

2.3.1. Introdução

Nessa fase mais quantitativa, são avaliadas as habilidades do candidato de


analisar o problema, reconhecer qual informação dada é relevante e
determinar em qual ponto se encontra a informação necessária para resolver
esse problema. Nesse tipo de questão, não é fundamental chegar a uma
resposta correta, mas sim compreender se o enunciado forneceu as
informações essenciais para que seja possível obter uma resposta.

A seguir, vamos apresentar uma pergunta desse tipo para tornar mais claro
como ela é estruturada e como resolvê-la de forma eficiente.

Pergunta:

How many hours would it take Pump A and Pump B working together, each at
its own constant rate, to empty a tank that was initially full?

(1) Working alone at its constant rate, Pump A would empty the full tank in 4
hours and 20 minutes.

(2) Working alone, Pump B would empty the full tank at its constant rate of 72
liters per minute.

a) Statement (1) is alone sufficient, but Statement (2) alone is not sufficient.

b) Statement (2) is alone sufficient, but Statement (1) alone is not sufficient.

c) Both statements together are sufficient, but neither statement alone is


sufficient.

d) Each statement alone is sufficient.

e) Statements (1) and (2) together are not sufficient

As perguntas seguem a seguinte estrutura: um questionamento, acompanhado


de duas declarações, que contém informações adicionais. Com isso em mente, o
candidato deve decidir se alguma das informações apresentadas são suficientes
para resolver a problemática e se são necessárias em conjunto ou se nenhuma é.
As alternativas são padronizadas, seguindo sempre a ordem.

27
14ª edição do Programa de Preparação - UFRJ Consulting Club Apostila

2.3.2. Estratégias

Dicas para resolução desse tipo de questão em específico são:

● Leia a informação inicial e a pergunta, em seguida, a primeira declaração e


verifique se ela possui a informação necessária para responder a pergunta.
Depois disso, independentemente da resposta, é preciso analisar a
segunda declaração sem levar em consideração as análises concluídas da
primeira. Por fim, deve-se pensar se essas duas declarações, em conjunto,
respondem o problema. Assim, para identificar a resposta correta é
possível seguir a linha de raciocínio apresentada no fluxograma abaixo:

● Não gaste tempo tentando solucionar a questão. Só é necessário


determinar se os dados são suficientes para responder à pergunta. Se
encontrar a resposta do problema não for indispensável para verificar
se as informações bastam para chegar a ela, fazer o cálculo será
desperdício de tempo.
● Responda à pergunta feita, se necessário. Apesar de termos
recomendado no ponto anterior que você evite solucionar a questão,
algumas vezes fazer isso pode ser o melhor caminho para avaliar se as
informações são ou não suficientes. Quando optar por essa estratégia,
assegure-se de que chegar à resposta é a opção mais fácil e
interrompa sua resolução assim que tiver certeza sobre a suficiência

28
14ª edição do Programa de Preparação - UFRJ Consulting Club Apostila

das afirmações.

Após as dicas, a seguir está o passo a passo de como resolver o exemplo


mostrado previamente:

Pergunta:

How many hours would it take Pump A and Pump B working together, each at
its own constant rate, to empty a tank that was initially full?

(1) Working alone at its constant rate, Pump A would empty the full tank in 4
hours and 20 minutes.
(2) Working alone, Pump B would empty the full tank at its constant rate of 72
liters per minute.
a) Statement (1) is alone sufficient, but Statement (2) alone is not sufficient.
b) Statement (2) is alone sufficient, but Statement (1) alone is not sufficient.
c) Both statements together are sufficient, but neither statement alone is
sufficient.
d) Each statement alone is sufficient.
e) Statements (1) and (2) together are not sufficient.

Resolução:

I. Da primeira afirmação, podemos perceber que somente os dados da bomba


A não respondem em quanto tempo as bombas, trabalhando em conjunto,
esvaziam o tanque, interpretando como um não no fluxograma.
II. A segunda afirmação também não é suficiente sozinha, pois só é fornecido
dados da bomba B isolada, interpretando como um não no fluxograma.
III. Avaliando se as duas afirmações, em conjunto, são suficientes para
determinar o tempo de esvaziamento, chega-se à conclusão que não são,
visto que não possuímos o dado do volume do tanque no momento inicial.
IV. Dessa forma, a resposta é que a afirmação (1) e (2) não são suficientes para
validar a questão, indicando que o gabarito é a letra E.

Resposta correta: E

29
14ª edição do Programa de Preparação - UFRJ Consulting Club Apostila

2.4. Problem Solving

2.4.1. Introdução

As perguntas de Problem Solving são responsáveis por testar suas habilidades


em matemática básica, resolução de problemas com números e interpretação
de gráficos. A seguir, será ilustrada uma pergunta desse tipo para tornar mais
claro como ela é estruturada e como resolvê-la de forma eficiente.

Pergunta:

In the figure above, point O is the center of the circle and OC = AC = AB. What is
the value of x ?

a) 40
b) 36
c) 34
d) 32
e) 30

2.4.2. Estratégias

Nessas questões, o essencial é resolver o que o comando pede e marcar uma


das cinco alternativas possíveis, lembrando que toda informação necessária está
no enunciado. Dicas para resolução desse tipo de questão em específico são:

● Examine as alternativas antes de responder à questão. Isso permite


entender onde o candidato quer chegar, podendo identificar aspectos
como a ordem de grandeza buscada, o formato numérico (fração, decimal,

30
14ª edição do Programa de Preparação - UFRJ Consulting Club Apostila

etc.), o grau de precisão exigido nos seus cálculos, entre outros. Dessa
maneira, percebe-se mais cedo se está no caminho certo ou errado.
● Estime para poupar tempo. Frequentemente, a questão não vai apenas
testar a sua capacidade de fazer contas, mas também de fazer
aproximações. Se as alternativas têm valores muito distantes, é possível,
por exemplo, aproximar 48% de um valor para sua metade (50%).
● Substitua para facilitar o problema. Em algumas situações é útil arbitrar
números para tornar o problema mais palpável. Esses números devem ser
simples de se trabalhar com e devem também satisfazer às restrições da
questão (por exemplo, se o problema informar que 𝑥 deve ser par, você
pode substituí-lo pelo número 2).
● Utilize backsolving. Esta técnica é parecida com a anterior, mas, neste
caso, o procedimento consiste em testar as alternativas da questão até
encontrar a resposta correta. Você pode ainda complementar o
backsolving com outras estratégias: colocando, por exemplo, as
alternativas em ordem crescente e substituindo a do meio, você pode
rapidamente identificar se a alternativa correta será maior ou menor do
que a última utilizada.

Após as dicas, a seguir está o passo a passo de como resolver o exemplo


mostrado previamente:

Pergunta:

In the figure above, point O is the center of the circle and OC = AC = AB. What is
the value of x ?

a) 40

31
14ª edição do Programa de Preparação - UFRJ Consulting Club Apostila

b) 36
c) 34
d) 32
e) 30

Resolução:

I. Após a leitura da pergunta e das alternativas, o problema é analisado. Como


OC = AC, o triângulo formado por OCA é isósceles, indicando que o ângulo
oposto a x também é x.
II. Como esse triângulo tem que somar 180°, o ângulo no vértice C no triângulo
OCA é igual a (180 - 2x).
III. Já o ângulo C do triângulo ACB, para ser suplementar ao ângulo (180-2x), é
igual a 2x.
IV. Como AC = AB, o ângulo B do triângulo ACB também é 2x.
V. Como OA e OB são raios da mesma circunferência - e, portanto, têm a mesma
medida -, sabemos que o triângulo OAB é isósceles, significando que o
ângulo em A é igual a B, que é 2x.
VI. Somando todos os ângulos desse triângulo, 2x + 2x + x e igualando a 180°,
concluímos que x é igual a 36°, que corresponde à letra B.

Resposta correta: B

2.5. Dicas gerais


A seguir, serão apresentadas boas práticas, complementares às dicas referentes a cada
formato de questão, para melhorar o desempenho na prova. Vale ressaltar que muitas
técnicas são bastante individuais, então é essencial cada candidato aplicá-las na prática
para entender o que o ajudará a resolver as questões de forma mais eficaz, ou seja,
rápida e corretamente.

2.5.1. Antes da prova


É interessante estar familiarizado com os formatos das questões para não ter
nenhuma surpresa. Assim, entender como elas são estruturadas e o passo a passo da
resolução, podem otimizar o tempo de prova. Além do mais, o conhecimento de

32
14ª edição do Programa de Preparação - UFRJ Consulting Club Apostila

matemática básica, como aritmética, álgebra e geometria, são essenciais para a


realização das questões de Problem Solving e Data Sufficiency do GMAT.

É indicado realizar diversos simulados como forma de preparação, pois a prática


é a melhor forma de entender o funcionamento da prova. Após a realização
destas provas, a correção de erros para evitá-los na hora da avaliação é essencial.

A gestão de tempo é algo que também deve ser treinado com antecedência,
pois certas provas têm, em média, apenas um minuto e meio para cada questão,
de forma que o tempo curto é um fator prejudicial para muitos candidatos. Além
disso, como, na prova, o uso de calculadoras é proibido, é indicado que o
candidato seja capaz de realizar contas sem auxílio delas uma calculadora, de
modo que treinar sem esse recurso é primordial para economizar tempo.

2.5.2. Durante a prova

É fortemente recomendado que, durante a prova, as questões sejam lidas com


calma e atenção, focando nas palavras-chaves, assim como nas informações e
nos dados principais. A fim de otimizar o tempo, evita-se demorar muito em
questões muito difíceis. Então, caso o candidato perceba que está a mais que o
esperado em uma questão da prova que está fazendo, ele deve eliminar as
opções aparentemente erradas e marcar a que faz mais sentido. É importante
ressaltar que não é indicado deixar nenhuma questão em branco.

2.6. Solve (Imbellus)


O McKinsey Digital Assessment é um jogo virtual customizado que não necessita de
conhecimentos prévios. Nessa simulação, é analisada a capacidade de resolver
problemas de uma maneira interativa, simulando as habilidades necessárias para ser
um consultor na McKinsey, como o comportamento diante da mudança de cenários e
tomada de decisão. Para analisar se o candidato possui essas habilidades, consultores
da própria firma resolvem o jogo e o programa identifica similaridades na resolução
deles com a do candidato. No teste, o candidato terá dois cenários, e deverá finalizar o
jogo em até 60 minutos.

33
14ª edição do Programa de Preparação - UFRJ Consulting Club Apostila

Nessas tarefas, o candidato deverá cumprir um objetivo dado a partir de informações


apresentadas que o guiarão para tomar decisões adequadamente. O grande diferencial
dessa prova, é que, ela avalia não só o resultado final, como também o processo como
um todo. Nesse sentido, o software identifica toda vez que o candidato clica ou mexe
no mouse e quanto tempo ele fica parado, analisando a dinâmica cognitiva do mesmo.

Dentre os cenários possíveis, podemos encontrar:

● Ecossystem creation: nesse cenário, é preciso criar um ecossistema


estável, sendo ele marinho ou terrestre. Nele, é necessário escolher oito
espécies distintas para habitá-lo, de forma a produzir a melhor chance de
sobrevivência ao local, levando em conta as condições ambientais.

● Organism Protection: nesse cenário, é necessário proteger uma planta


nativa contra espécies invasoras e o objetivo é sobreviver a uma
quantidade de rodadas. Para impedir que as espécies cheguem na planta,
podem ser usadas 2 formas de defesa: os predadores, que atacam os
invasores, ou as barreiras geográficas, que atrasam ou mudam a rota dos
predadores.

● Disaster Management: nesse cenário, o tipo de desastre que está


acontecendo deve ser identificado com base em um conjunto de
indicativos. Os desastres podem ser tsunamis e erupções vulcânicas,
enquanto os indicativos podem ser temperatura, pressão e chuva. Após o
primeiro objetivo, será necessário mover os animais para um local com
maior probabilidade de sobrevivência, com base em suas características e
nas do local.

● Disease Management: nesse cenário, é preciso descobrir qual doença


está afetando uma população animal com base num conjunto de
sintomas. Depois, deverá recomendar o melhor tratamento a partir das
características da doença, população animal e opções de tratamento. O
objetivo final é otimizar a taxa de sobrevivência para a população animal.

Boas práticas que podem ser utilizadas para resolver esse jogo são:

● Entenda as informações dadas, sejam elas em forma de texto, gráficos ou


tabelas.

34
14ª edição do Programa de Preparação - UFRJ Consulting Club Apostila

● Não replique a solução de outros candidatos, visto que todas as provas


são geradas individualmente para cada candidato, logo, os resultados
nunca serão iguais e não existe um gabarito único.

● Entenda que é normal tomar decisões com informações incompletas


devido ao tempo curto, logo, é necessário ter controle do mesmo.

● Foque no objetivo do jogo, retirando as informações mais relevantes dos


dados, não se prendendo a detalhes.

● Procurar a solução com melhores indicativos.

2.7. Revisão de conceitos matemáticos


Este tópico traz alguns conceitos matemáticos básicos que podem ajudar na hora de
resolver as provas, seja GMAT ou Business Case. A maioria dos itens abaixo são assuntos
de Ensino Fundamental e Médio, enquanto outros não serão aprofundados por serem
mais bem abordados no glossário da apostila (exemplo: profits). É importante
relembrar alguns conceitos para garantir uma nota alta na prova

Aritmética: frações, decimais, números reais e inteiros, racionalização e proporção,


porcentagem, potências e raízes dos números, estatística descritiva, métodos de
contagem, probabilidade.

Álgebra: simplificação de expressões algébricas, resolução de equações de primeiro e


segundo grau, fatoração, exponenciação, inequações, valores absolutos e funções.

Geometria: interseções entre retas e ângulos, retas perpendiculares, retas paralelas,


polígonos convexos, triângulos, quadriláteros, círculos, sólidos retangulares e cilindros,
geometria de coordenadas.

2.7.1. Fórmulas

● Teorema de Pitágoras: 𝑎2 = 𝑏2 + 𝑐2

● Fórmula da área do quadrado: 𝑙2

𝑏𝑎𝑠𝑒 × 𝑎𝑙𝑡𝑢𝑟𝑎
● Fórmula da área do triângulo: 2

35
14ª edição do Programa de Preparação - UFRJ Consulting Club Apostila

(𝑏𝑎𝑠𝑒 𝑚𝑎𝑖𝑜𝑟 + 𝑏𝑎𝑠𝑒 𝑚𝑒𝑛𝑜𝑟) × 𝑎𝑙𝑡𝑢𝑟𝑎


● Fórmula da área do trapézio: 2

● Fórmula da área do círculo: 𝜋𝑟2

● Fórmula geral do volume: Á𝑟𝑒𝑎 𝑑𝑎 𝑏𝑎𝑠𝑒 × 𝐴𝑙𝑡𝑢𝑟𝑎

á𝑟𝑒𝑎 𝑑𝑎 𝑏𝑎𝑠𝑒 × 𝑎𝑙𝑡𝑢𝑟𝑎


● Fórmula do volume de pirâmides e cones: 3

● Equação da reta: 𝑎𝑥 + 𝑏, em que 𝑎 = coeficiente angular e 𝑏 = coeficiente


linear

● Retas paralelas apresentam mesmo coeficiente angular.

● O produto dos coeficientes angulares de retas perpendiculares é -1.

2.7.2. Tipos de problemas

● Problemas de velocidade (rate problems): a velocidade média é igual a


∆𝑠
razão entre o deslocamento e o tempo transcorrido → 𝑣 = ∆𝑡
𝑚

● Problemas de trabalho (work problems): por exemplo, se uma máquina


X pode produzir mil garrafas em quatro horas e a máquina Y pode
produzir mil garrafas em cinco horas, em quantas horas as máquinas X e Y
trabalhando juntas em velocidade constante, produzem 1.000 garrafas,

● Problemas de mistura (mixture problems): diferentes substâncias são


misturadas, e a questão pede alguma informação sobre a mistura
resultante, como a quantidade, fração ou porcentagem de cada
substância na mistura ou o total resultante.

● Problemas de juros e rendimentos (interest problems)

○ Juros simples: 𝐽 = 𝐶 · 𝑖 · 𝑡

J = juros

C = capital

i = taxa de juros

t = tempo

𝑛
○ Juros compostos: 𝑆 = 𝑃 · (1 + 𝐽)

36
14ª edição do Programa de Preparação - UFRJ Consulting Club Apostila

S = montante

P = principal

i = taxa de juros

n = períodos em que o principal foi aplicado

● Desconto (discount)

● Lucratividade (profitability): o lucro é calculado a partir da diferença


entre os valores de receita e custos ou por meio da fórmula:

○ Lucro = (preço de venda × quantidade vendida) – [(custo de


venda × quantidade vendida) + custos fixos].

● Conjuntos numéricos (sets): operações com conjuntos (união e


interseção).

● Problemas com medidas (measurement problems): algumas questões


da prova envolvem unidades métricas enquanto outras envolvem
unidades estadunidenses. No entanto, a não ser com unidades de tempo,
se uma questão pedir para o candidato transformar uma unidade para a
outra, a relação entre as duas será dada no enunciado.

● Interpretação de dados (data interpretation)

2.7.3. Pontos de atenção

Como as provas são em inglês, é válido comentar o significado de alguns termos


frequentes em questões, porém, com tradução não tão óbvia:

● average/mean (média)
● ratio (razão)
● standard deviation (desvio padrão)
● integer (número inteiro)
● even number (número par)
● odd number (número ímpar)
● prime number (número primo)
● tens digit (algarismo da dezena)
● units digit (algarismo da unidade)

37
14ª edição do Programa de Preparação - UFRJ Consulting Club Apostila

● radius (raio)
● height (altura)
● length (comprimento)
● width (largura)

O candidato também deve se atentar para a diferença entre o uso do ponto e da


vírgula em números. Diferentemente do Brasil, nos EUA, o ponto é o separador
decimal padrão, isto é, separa a parte inteira da parte decimal do número (ex:
2.489 = dois inteiros e quatrocentos e oitenta e nove milésimos). Já a vírgula é o
separador de milhar padrão, ou seja, é utilizada em números grandes, a cada três
algarismos, para facilitar a leitura (ex: 2,489 = dois mil quatrocentos e oitenta e
nove).

2.8. Exemplos com resolução comentada

2.8.1. Critical Reasoning

1) The committee on sexual discrimination in the workplace has highlighted


Supremo Company as a chief offender. Of the twenty senior executives in the
firm, only one is a woman. And of the forty junior executives, only five are
female.

Supremo could best defend itself against the charges by showing that:

a) male and female executives at the same level have the same qualifications.

b) they pay the same salary to senior men and senior women.

c) ten times more men than women apply for jobs with the company.

d) the work pressures and long hours make jobs with the company
unattractive to married women.

e) all job applicants who were rejected had fewer qualifications than those
accepted.

Resolução:

I. Vamos construir uma resposta hipotética antes de olhar as alternativas. Para


se defender de uma acusação de discriminação, a companhia terá que

38
14ª edição do Programa de Preparação - UFRJ Consulting Club Apostila

mostrar que as suas seleções foram feitas com base no mérito e não no
gênero. A melhor maneira de fazê-lo é mostrar que nenhuma das mulheres
que foram rejeitadas tinham qualificações superiores às dos homens
promovidos;

II. Analisando, agora, as alternativas, aquela que mais se aproxima da nossa


solução é a E.

Resposta correta: E

2) It is not unusual to see the ball fall into a black slot on a roulette wheel four
times in a row. But for it to fall five or six times in a row into the same color is
very unusual. Therefore, you can win money by waiting for a run of five of the
same color and then betting against that color.

If the roulette wheel in question is a fair wheel, which of the following


observations or facts, if it were true, would best reveal a fallacy in the logic?

a) If there were a reliable way to win at roulette it would be well-known by now.

b) It is hard for a player to keep track of what went before for the time required.

c) The probability of getting a particular color decreases with the number of


times the color has appeared.

d) The probability of getting a particular color is always the same no matter


what has gone before.

e) A person who makes money this way once or twice, will carry on losing that
money after a few more times.

Resolução:

I. A questão nos pede para encontrar a observação que refuta a lógica do


enunciado. Devemos primeiro, portanto, identificar a conclusão do
enunciado;

II. O enunciado conclui que, se você esperar uma sequência de cinco da mesma
cor e então apostar contra essa cor, você irá ganhar, como se a probabilidade
da mesma cor se repetir outra vez fosse muito pequena;

III. Sabemos, do estudo de probabilidade, que a chance de uma cor ser tirada

39
14ª edição do Programa de Preparação - UFRJ Consulting Club Apostila

em uma nova jogada é sempre a mesma, não sendo dependente dos eventos
passados. Por isso, o fato de esperarmos uma sequência de cinco ou seis
jogadas na mesma cor não influencia em nada o resultado da próxima
jogada, o que invalida o argumento apresentado;

IV. A letra D apresenta exatamente isso. As outras alternativas, mesmo que


algumas sejam observações factíveis, não desconstroem a lógica da
conclusão do enunciado e, por isso, não são opções corretas para a questão.

Resposta correta: D

3) It has been suggested that long-term prisoners, on release from jail, be given
a reasonable state pension to reduce the likelihood of their resorting to
crime. Most people instinctively reject the suggestion as they feel it would be
like rewarding criminal activity. The supporters of the prisoners’ pension
scheme have criticized those who reject this possibility, by claiming that for
the critics...

Which of the following is the most logical completion of the sentence above?

a) emotion is more important than justice.

b) punishment for criminals is more important than crime prevention.

c) crime prevention is not an important issue.

d) money has too high a value.

e) the law should not be concerned with what happens after jail.

Resolução:

I. A questão pede que identifiquemos a premissa que completa o raciocínio


dos apoiadores da pensão. Para isso, vamos, antes, entender o que eles
acreditam que os críticos pensam para embasar sua posição;

II. Os críticos rejeitam o esquema de pensão instintivamente, por não quererem


“recompensar o crime”. Eles não param para refletir sobre o possível benefício
de prevenir a recorrência de crime, julgando ser mais importante a imagem
de estar punindo o criminoso. Vemos, portanto, que a alternativa B é a
correta;

40
14ª edição do Programa de Preparação - UFRJ Consulting Club Apostila

III. Analisaremos, para ter certeza, as outras alternativas. O raciocínio dos críticos
tem embasamento racional, então a emoção não tem influência e, por isso, a
letra A está incorreta. O fato de os críticos darem mais importância à punição
não implica necessariamente que eles julgam a prevenção do crime como
não importante, logo a C é descartada. Como a posição se baseia em
princípios, e não na quantia paga, descartamos a letra D. Por último, a E
simplesmente não tem conexão com o que foi apresentado no enunciado.

Resposta correta: B

4) Sue: Commercial flights currently contribute more carbon dioxide to the


atmosphere in one year than does the whole of Africa. If we want to reduce
global warming, we need to restrict the number of flights we take.

Dave: Did you know that by taking one inter-continental flight you cause
more pollution than you would in twelve months of car travel?

Dave’s response to Sue’s comment serves to:

a) Reinforce Sue’s contention that flights are a major contributor to increased


carbon dioxide levels.

b) Add more weight to her contention that we should reduce the number of
flights we take.

c) Mitigate the force of her argument by suggesting that there is an alternative


approach.

d) Suggest an alternative that will reduce the effect of pollution.

e) Question whether she really understands the severity of global warming.

Resolução:

I. A questão pede para identificar o papel da resposta de Dave no contexto.


Vamos, então, identificar a tese de Sue e de Dave;

II. A fala de Sue defende que voos comerciais são muito prejudiciais para o meio
ambiente e que devem, por isso, ser evitados. A resposta de Dave mostra que
voos causam muito mais poluição do que viagens de carro;

41
14ª edição do Programa de Preparação - UFRJ Consulting Club Apostila

III. Percebemos, portanto, que a tese de Dave reforça o que foi defendido por
Sue. Analisando as alternativas, a que melhor retrata essa relação observada
entre as falas é a letra B;

IV. Para garantir o acerto, avaliamos as opções restantes. Como Dave não apenas
reafirma a tese de Sue, mas sim adiciona outra informação que a fortalece, a
opção A não é a ideal. Como Dave não confronta o argumento de Sue,
eliminamos também as letras C e. Por fim, foi Sue quem apresentou uma
alternativa para diminuir a poluição, e não Dave, o que descarta a letra D.

Resposta correta: B

● Note que essa questão ilustra bem a necessidade de se atentar aos


termos utilizados e escolher sempre a melhor opção possível, por
mais que possa existir outra parcialmente correta.

5) Anton: I sold my house on an internet site last year and was happy with the
price. I got a speedy sale, and the cost of advertising was insignificant. I
would advise you to avoid real estate agents. Barbie: It is in the interest of the
real estate agent to get me the best price for my property because he gets a
commission based on the selling price. Therefore, when selling my house, I
will certainly use an agent rather than trying to sell the house by word of
mouth, or by advertising in newspapers or on the internet.

Barbie’s could strengthen her position by pointing out all of the following
except:

a) Houses of comparable value often obtain a lower price when sold on the
internet.

b) Very few houses are sold on the internet at the moment and so a valid
comparison is difficult.

c) The agent’s service includes many add-on benefits in terms of legal fees,
surveyor’s reports and advice that are not available on internet sites.

d) Some buyers pay the agent to find them a cheap house.

e) The agent’s commission is usually less than the difference between the

42
14ª edição do Programa de Preparação - UFRJ Consulting Club Apostila

internet price and the higher price the agent obtains for you.

Resolução:

I. A questão pede para identificarmos a única informação que não fortalece a


sua posição;

II. Barbie defende a venda de imóveis por meio de agentes, argumentando que
os mesmos conseguem um pagamento mais alto que o obtido pela venda
autônoma. Vamos avaliar, portanto, que informações dão respaldo a essa
tese;

III. As alternativas A e B apontam fatos negativos sobre a venda de casas pela


internet, o que dá força à ideia de contratar um agente. Já C e tratam
diretamente de benefícios adquiridos pelo agenciamento;

IV. A informação D diz respeito à utilização do profissional por compradores, com


consequências negativas para o vendedor da casa. Logo, é a exceção que não
embasa a posição da Barbie.

Resposta correta: D

6) French cuisine is highly regarded all over the world. Yet in Paris there are
more American restaurants selling burgers and fries (which many people
now class as “junk food”) than there are in any other European capital city.
Obviously, the French are very fond of “junk food” and are not too proud to
eat it.

Which of the following, if true, would most weaken the author’s contention?

a) There are also a larger number of Lebanese restaurants in Paris than there
are in other European capital cities.

b) French Cordon Bleu cuisine is very expensive.

c) The number of French tourists eating in New York burger restaurants is very
low.

d) Junk food actually has high nutritional value when eaten in moderation.

e) There are an unusually large number of American tourists in Paris who eat

43
14ª edição do Programa de Preparação - UFRJ Consulting Club Apostila

at burger joints.

Resolução:

I. Precisamos identificar a observação que refuta a conclusão apresentada. Para


isso, é preciso, antes, encontrar a conclusão propriamente dita;

II. O autor conclui, a partir do fato de haver muitos restaurantes americanos em


Paris, que os franceses adoram junk food;

III. A melhor maneira de refutar essa conclusão é provar que os franceses não
comem nesses restaurantes, que devem, portanto, servir a outro público que
não o local;

IV. Analisando as alternativas, vemos que a letra E nos entrega um fato que
prova exatamente isso. Descartamos as outras opções pois elas não
desconstroem diretamente a conclusão apresentada.

Resposta correta: E

7) A fruit known as amla in certain parts of Asia is an excellent source of


vitamin C. A small quantity of the fruit grated and added to salads provides
almost all the daily requirement of this vitamin. However, the fruit is very
sour. A new process designed to remove most of the sour taste will make the
fruit acceptable to American tastes. We are therefore starting to grow this
fruit for sale in the United States.

The argument above assumes all of the following except:

a) Americans generally won’t eat very sour foods.

b) The new process does not remove a significant part of the vitamin content.

c) That a market exists for a new source of vitamin C.

d) The fruit can be used only in salads.

e) Apart from being sour there are no other objections to eating this fruit.

Resolução:

I. Temos que identificar as premissas que foram utilizadas para sustentar o


argumento, e, então, selecionar a única alternativa que não está entre elas. É

44
14ª edição do Programa de Preparação - UFRJ Consulting Club Apostila

necessário ter atenção, pois essa é uma questão que pede a alternativa
incorreta, o que por vezes pode passar despercebido e resultar em erro;

II. O enunciado conclui que eles devem, e estão, cultivando a fruta para venda
nos EUA. Primeiro, ele argumenta que a fruta é uma excelente fonte de
vitamina C, o que significa que ele julga existir uma demanda por fontes
dessa vitamina. Depois, ele coloca a acidez da fruta como uma característica
negativa, o que pressupõe que os americanos não costumam comer
alimentos ácidos. Sugere, então, um processo que desacidifica a fruta,
assumindo implicitamente que esse processo não irá, no entanto, afetar a
concentração de vitamina C. Como nenhuma outra modificação é pensada,
há, por fim, a premissa de que a acidez é a única objeção ao consumo da
amla;

III. As premissas identificadas são representadas nas alternativas A, B, C e E. A


alternativa D não é uma premissa do enunciado, já que, embora ele aborde o
consumo da fruta em saladas, em nenhum momento ele diz que esse é o seu
único uso.

Resposta correta: D

8) On the basis of the Big Bang theory scientists predicted levels of Helium-3 in
the universe that are ten times greater than the levels actually observed.
According to the original model, Helium-3 is produced when low-mass stars
burn up hydrogen and become red giants, as well as being produced in the
Big Bang itself. Researchers have now produced a new model in which the
Helium-3 produced by a red giant is pushed to the stars interior and burnt
up. Hence the Big Bang theory is no longer undermined by Helium-3 data.

The two portions underlined are related to each other in which of the
following ways?

a) The first highlights an observation that tends to undermine a particular


theory; the second is that theory.

b) The first is a fact that undermines a theory; the second is context for
accepting that theory.

45
14ª edição do Programa de Preparação - UFRJ Consulting Club Apostila

c) The first points to an inconsistency in a particular model; the second is the


author’s main conclusion.

d) The first is a challenge to a classic theory; the second resolves that challenge.

e) The first is a position that the author does not accept; the second is the
author’s position.

Resolução:

I. A questão nos pede a relação entre os dois trechos. Para isso, precisamos
identificar o papel de cada uma delas no raciocínio apresentado;

II. A primeira informação apresenta o fato de haver discrepância entre o que foi
previsto e o que é hoje observado. Já o segundo fragmento apresenta uma
conclusão causada pela reformulação da teoria, o que é até ilustrado pelo
termo “hence”;

III. Analisando as alternativas, vemos que a alternativa C é a que melhor


apresenta os papéis identificados.

Resposta correta: C

9) Recent studies have highlighted the harmful effects of additives in food


(colors, preservatives, flavor enhancers etc.). There are no synthetic
substances in the foods we produce at Munchon Foods – we use only natural
ingredients. Hence you can be sure you are safeguarding your family’s
health when you buy our products.

Which of the following, if true, would most weaken the contention of


Munchon Foods?

a) Some synthetic substances are not harmful.

b) Some natural substances found in foods can be harmful.

c) Food without additives is unlikely to taste good.

d) Munchon Foods produces only breakfast cereals.

e) Without preservatives some foods could cause harm.

46
14ª edição do Programa de Preparação - UFRJ Consulting Club Apostila

Resolução:

I. Mais uma vez, queremos encontrar a observação que refuta a conclusão


apresentada. Sendo assim, primeiro é preciso identificar qual é essa
conclusão;

II. O enunciado argumenta que, por não possuir aditivos em seus produtos,
podemos ter certeza de que os alimentos da Munchon Foods não farão mal à
saúde;

III. Antes de olhar as alternativas, vamos pensar na solução ideal. Para


desconstruir a conclusão, deveríamos mostrar que substâncias sintéticas não
são a única ameaça à nossa saúde;

IV. Olhando agora as alternativas, vemos que a B apresenta um argumento


compatível com o nosso raciocínio, sendo, portanto, a opção correta;

V. As outras alternativas, mesmo que algumas sejam por si só observações


factíveis, não desconstroem a lógica da conclusão do enunciado e, por isso,
não são opções corretas para a questão.

Resposta correta: B

10) Josh has twenty years of typing experience behind him; therefore, if you are
looking for an efficient typist to enter your data into the new system, you
need look no further.

The speaker assumes that:

a) Twenty years of practice ensures typing efficiency.

b) The type of typing required for the new system is identical to what Josh has
been doing.

c) Josh’s job profile is the best that the new employer is going to get.

d) Josh is an outstandingly fast and accurate typist.

e) Josh will fit well into the new office.

Resolução:

I. A questão pergunta sobre o que o que enunciado está assumindo, ou seja,

47
14ª edição do Programa de Preparação - UFRJ Consulting Club Apostila

qual é a premissa que está sendo utilizada para a conclusão apresentada;

II. Estamos procurando, portanto, a premissa do argumento, que, se estivesse


errada, iria invalidá-lo;

III. A única premissa utilizada no argumento é a de que o tempo de prática


implica na eficiência de Josh. A alternativa que a apresenta é A;

IV. Analisaremos as outras opções, a fim de ter certeza da resposta. Eliminamos


B, porque nada é dito sobre o tipo de digitação necessária. Descartamos C
porque a questão não fala do perfil de trabalho como um todo, mas apenas
da habilidade de digitação. Como o enunciado fala de eficiência, sem, no
entanto, dizer que isso significa velocidade e precisão, a D também é
inadequada. Por fim, nada é dito sobre Josh se encaixar bem no escritório, o
que invalida a E.

Resposta correta: A

2.8.2. Data Sufficiency

1) If 𝑥 is negative, can we say that 𝑥 < −3?

(1) 𝑥2 > 9

(2) 𝑥3 < −9

a) Statement (1) ALONE is sufficient, but statement (2) alone is not sufficient.

b) Statement (2) ALONE is sufficient, but statement (1) alone is not sufficient.

c) BOTH statements TOGETHER are sufficient, but NEITHER statement ALONE


is sufficient.

d) EACH statement ALONE is sufficient.

e) Statements (1) and (2) TOGETHER are not sufficient.

Resolução:

I. Da afirmação (1), podemos tirar que 𝑥 > 3 ou 𝑥 < −3; mas como sabemos que 𝑥
é negativo, concluímos que 𝑥 < −3. Logo, conseguimos responder à pergunta
“Podemos dizer que 𝑥 < −3?” com um SIM;

II. Da afirmação (2), tiramos que 𝑥 < −2,08, o que NÃO nos permite responder à

48
14ª edição do Programa de Preparação - UFRJ Consulting Club Apostila

questão, pois 𝑥 pode ser menor ou maior que −3;

III. Concluímos, portanto, que a afirmação (1) sozinha nos permite responder à
pergunta, enquanto a afirmação (2) sozinha não nos traz qualquer resposta.

Resposta correta: A

● É importante apontar que o que levamos em conta é a capacidade da


afirmação de nos levar a uma resposta para a pergunta feita, seja essa
resposta positiva ou negativa. Caso a pergunta feita fosse “𝑥 é maior que
−3?”, por exemplo, a afirmação (1) continuaria suficiente para solucionar a
questão, desta vez nos mostrando que “𝑥 NÃO é menor que 3”. Atente-se a
esse detalhe para não se equivocar em questões semelhantes.

2) What percent of a group of people are women with red hair?

(1) Of the women in the group, 5% have red hair.

(2) Of the men in the group, 10% have red hair.

a) Statement (1) ALONE is sufficient, but statement (2) alone is not sufficient.

b) Statement (2) ALONE is sufficient, but statement (1) alone is not sufficient.

c) BOTH statements TOGETHER are sufficient, but NEITHER statement ALONE


is sufficient.

d) EACH statement ALONE is sufficient.

e) Statements (1) and (2) TOGETHER are not sufficient.

Resolução:

I. Para chegar à porcentagem de mulheres ruivas no grupo de pessoas,


precisaremos ter o número de mulheres ruivas e o número total de pessoas
no grupo;

II. Analisando a afirmação (1), percebemos que temos a porcentagem de


mulheres ruivas dentre as mulheres do grupo. Contudo, não sabemos nem o
número de mulheres, nem o número total de pessoas no grupo. Portanto, a
afirmação (1) sozinha é insuficiente para resolver a questão;

III. A informação (2) nos dá apenas a porcentagem de homens ruivos dentre os

49
14ª edição do Programa de Preparação - UFRJ Consulting Club Apostila

homens do grupo. Analogamente à afirmação (1), a afirmação (2) sozinha


também é insuficiente;

IV. Juntando as afirmações (1) e (2), temos a porcentagem de homens ruivos,


dentre os homens, e a porcentagem de mulheres ruivas, dentre as mulheres.
Ainda assim, não conseguimos obter os dados necessários para encontrar a
porcentagem de mulheres ruivas no grupo;

V. Concluímos, portanto, que, mesmo unidas, as afirmações são insuficientes.

Resposta correta: E

3) A citrus fruit grower receives $15 for each crate of oranges shipped and $18
for each crate of grapefruit shipped. How many crates of oranges did the
grower ship last week?

(1) Last week the number of crates of orange that the grower shipped was 20
more than twice the number of crates of grapefruit shipped.

(2) Last week the grower received a total of $38,700 from the crates of oranges
and grapefruit shipped.

a) Statement (1) ALONE is sufficient, but statement (2) alone is not sufficient.

b) Statement (2) ALONE is sufficient, but statement (1) alone is not sufficient.

c) BOTH statements TOGETHER are sufficient, but NEITHER statement ALONE


is sufficient.

d) EACH statement ALONE is sufficient.

e) Statements (1) and (2) TOGETHER are not sufficient.

Resolução:

I. Considerando que 𝑥 é o número de caixas de laranja e 𝑦 é o número de caixas


de grapefruit, devemos achar o valor de 𝑥;

II. Analisando a afirmação (1), tiramos que 𝑥 = 2𝑦 + 20. Como temos uma equação
e duas variáveis, essa informação é insuficiente;

III. Analisando a afirmação (2), tiramos que 15𝑥 + 18𝑦 = 38.700. Assim como
afirmação (1), essa informação, sozinha, é insuficiente;

50
14ª edição do Programa de Preparação - UFRJ Consulting Club Apostila

IV. Juntando as duas afirmações, teremos enfim duas equações para as duas
variáveis, um sistema resolvível, o que torna possível encontrar os valores de 𝑥
e 𝑦. Perceba que não precisamos efetivamente calcular os valores, mas tão
somente saber ser viável determiná-los.

Resposta correta: C

4) What is the value of 𝑧 in the following triangle?

(1) 𝑥 + 𝑦 = 139

(2) 𝑦 + 𝑧 = 108

a) Statement (1) ALONE is sufficient, but statement (2) alone is not sufficient.

b) Statement (2) ALONE is sufficient, but statement (1) alone is not sufficient.

c) BOTH statements TOGETHER are sufficient, but NEITHER statement ALONE


is sufficient.

d) EACH statement ALONE is sufficient.

e) Statements (1) and (2) TOGETHER are not sufficient.

Resolução:

^ ^ ^
I. Em qualquer triângulo, a soma dos ângulos internos 𝑥, 𝑦 𝑒 𝑧 é igual a 180º;

II. Analisando a alternativa (1), vemos que 𝑥 + 𝑦 = 139. Substituindo na soma dos
ângulos internos, calculamos 𝑧 = 41º. Logo, a alternativa (1) é suficiente;

III. Na alternativa (2), temos que 𝑦 + 𝑧 = 108. Só conseguiríamos, com essa


equação, determinar o valor de 𝑥. Nada podemos concluir quanto ao valor de
𝑧. Portanto, a alternativa (2) é insuficiente.

Resposta correta: A

51
14ª edição do Programa de Preparação - UFRJ Consulting Club Apostila

5) If 𝐴 and 𝐵 are positive integers, is the product 𝐴 × 𝐵 even?

(1) The sum 𝐴 + 𝐵 is odd.

(2) 𝐴 is even.

a) Statement (1) ALONE is sufficient, but statement (2) alone is not sufficient.

b) Statement (2) ALONE is sufficient, but statement (1) alone is not sufficient.

c) BOTH statements TOGETHER are sufficient, but NEITHER statement ALONE


is sufficient.

d) EACH statement ALONE is sufficient.

e) Statements (1) and (2) TOGETHER are not sufficient.

Resolução:

I. Analisando a afirmação (1), sabemos que se 𝐴 é par, 𝐵 é ímpar e vice-versa,


uma vez só assim sua soma pode ter resultado ímpar. E, como o produto de
um par e um ímpar é sempre par, a informação (1) é suficiente para
solucionar a questão;

II. Analisando a afirmação (2), sabemos que 𝐴 é par e 𝐵 pode ser par ou ímpar. O
produto de um número par com qualquer outro número inteiro positivo
sempre é par. Logo, a afirmação (2) também basta para responder à
pergunta.

Resposta correta: D

6) A scientist recorded the number of eggs in each of ten birds’ nests. What was
the standard deviation of the number of eggs in the ten nests?

(1) The average (arithmetic mean) number of eggs for the ten nests was 4.

(2) Each of the ten nests contained the same number of eggs.

a) Statement (1) ALONE is sufficient, but statement (2) alone is not sufficient.

b) Statement (2) ALONE is sufficient, but statement (1) alone is not sufficient.

c) BOTH statements TOGETHER are sufficient, but NEITHER statement ALONE


is sufficient.

52
14ª edição do Programa de Preparação - UFRJ Consulting Club Apostila

d) EACH statement ALONE is sufficient.

e) Statements (1) and (2) TOGETHER are not sufficient.

Resolução:

I. Primeiro, devemos lembrar que o desvio padrão é a raiz quadrada da


𝑁
variância e a variância é dada pelo somatório descrito por ∑ (𝑥𝑖 − 𝑥𝑚), onde 𝑥𝑚
𝑖=1

é a média aritmética e 𝑥𝑖 é um dos valores obtidos;

II. Agora analisando a afirmação (1), podemos perceber que temos o valor da
média aritmética, mas não temos a quantidade de ovos em cada ninho.
Portanto, a afirmação (1) é insuficiente;

III. Analisando a afirmação (2), sabemos que o número de ovos em cada ninho é
igual. Então temos n ovos em cada ninho. Se temos n ovos em cada ninho, a
média aritmética é n. Portanto, a variância é igual a zero, e,
consequentemente, o desvio padrão também é igual a zero. Portanto, a
afirmação (2) é suficiente.

Resposta correta: B

7) Consider 𝑑 = 0.43𝑡7. If 𝑡 denotes the thousandths digit in the decimal


representation of 𝑑 above, what digit is 𝑡?

(1) If 𝑑 were rounded to the nearest hundredth, the result would be 0.44.

(2) If 𝑑 were rounded to the nearest thousandth, the result would be 0.436.

a) Statement (1) ALONE is sufficient, but statement (2) alone is not sufficient.

b) Statement (2) ALONE is sufficient, but statement (1) alone is not sufficient.

c) BOTH statements TOGETHER are sufficient, but NEITHER statement ALONE


is sufficient.

d) EACH statement ALONE is sufficient.

e) Statements (1) and (2) TOGETHER are not sufficient.

Resolução:

I. Analisaremos primeiro a afirmação (1) isoladamente. Como o número é

53
14ª edição do Programa de Preparação - UFRJ Consulting Club Apostila

arredondado para cima, sabemos que 𝑡 ≥ 5. Essa informação é insuficiente


sozinha para encontrarmos o valor de 𝑡;

II. Analisando a afirmação (2), vemos que, como o último dígito, 7, causa um
arredondamento para cima na casa do milhar, que chega a 6, podemos
concluir que 𝑡 era anteriormente 5. A afirmação é, portanto, suficiente por si
só.

Resposta correta: B

8) If the sequence 𝑆 has 300 terms, what is the 293𝑟𝑑 term of 𝑆?

(1) The 298𝑡ℎ term of 𝑆 is −616, and each term of S after the first is 2 less than the
preceding term.

(2) The first term of S is −22.

a) Statement (1) ALONE is sufficient, but statement (2) alone is not sufficient.

b) Statement (2) ALONE is sufficient, but statement (1) alone is not sufficient.

c) BOTH statements TOGETHER are sufficient, but NEITHER statement ALONE


is sufficient.

d) EACH statement ALONE is sufficient.

e) Statements (1) and (2) TOGETHER are not sufficient.

Resolução:

I. Analisaremos primeiro a afirmação (1) sozinha. Temos o valor do 298º termo e


sabemos que cada termo antecedente é duas unidades maior do que o
seguinte. Para chegar ao 293º termo dessa progressão aritmética, portanto,
cinco posições antes do 298º, somamos cinco vezes as duas unidades.
Provamos, assim, que apenas a afirmação (1) já basta para responder à
pergunta;

II. Olharemos agora a afirmação (2). Temos o valor do 1º termo da sequência.


Sabemos, no entanto, que o enésimo termo de uma progressão aritmética é
dado por 𝑎𝑛 = 𝑎1 + (𝑛 − 1) * 𝑅. Como não temos o valor da razão R, a

afirmação (2) sozinha é insuficiente para calcular o 293º termo.

54
14ª edição do Programa de Preparação - UFRJ Consulting Club Apostila

Resposta correta: A

9) What is the value of |𝑥|?

(1) 𝑥 = −|𝑥|

(2) 𝑥2 = 4

a) Statement (1) ALONE is sufficient, but statement (2) alone is not sufficient.

b) Statement (2) ALONE is sufficient, but statement (1) alone is not sufficient.

c) BOTH statements TOGETHER are sufficient, but NEITHER statement ALONE


is sufficient.

d) EACH statement ALONE is sufficient.

e) Statements (1) and (2) TOGETHER are not sufficient.

Resolução:

I. Analisamos inicialmente a afirmação (1) isolada. Ela apenas nos mostra que 𝑥
é um número negativo, ou zero, não sendo suficiente sozinha, portanto, para
responder o valor pedido;

II. Estudamos agora a afirmação (2). Tirando a raiz quadrada dos dois lados da
equação, encontraremos 𝑥 = ±2. Como o valor procurado é o do módulo, o
sinal não importa e conseguimos responder à pergunta apenas com essa
informação.

Resposta correta: B

10) In College X the number of students enrolled in both a chemistry course and
a biology course is how much less than the number of students enrolled in
neither?

(1) In College X there are 60 students enrolled in a chemistry course.

(2) In College X there are 85 students enrolled in a biology course.

a) Statement (1) ALONE is sufficient, but statement (2) alone is not sufficient.

b) Statement (2) ALONE is sufficient, but statement (1) alone is not sufficient.

55
14ª edição do Programa de Preparação - UFRJ Consulting Club Apostila

c) BOTH statements TOGETHER are sufficient, but NEITHER statement ALONE


is sufficient.

d) EACH statement ALONE is sufficient.

e) Statements (1) and (2) TOGETHER are not sufficient.

Resolução:

I. Como se trata de um problema de interseção de conjuntos, vamos primeiro


desenhar o diagrama abaixo, em que 𝑥 representa o número de estudantes
apenas de química, 𝑦 representa o número de estudantes das duas matérias,
𝑧 representa o número de estudantes só de biologia e 𝑤 representa o número
de estudantes que não cursam nenhuma das duas matérias;

II. Queremos encontrar, portanto, o valor de 𝑤 − 𝑦. Começamos avaliando a


afirmação (1) isoladamente. Ela nos diz que 𝑥 + 𝑦 = 60. Essa única equação é
insuficiente para chegar ao valor de 𝑤 e de 𝑦 e, portanto, a (1) sozinha não
responde à pergunta;

III. Analogamente, a (2) nos fala que 𝑧 + 𝑦 = 85, sendo também insuficiente para
encontrar o valor que desejamos;

IV. Unindo as duas afirmações, teremos um sistema com duas equações e três
variáveis, o que já inviabiliza a sua solução. Além disso, a variável 𝑤 nem está
envolvida no sistema, o que confirma ainda mais que as informações juntas
continuam insuficientes.

Resposta correta: E

56
14ª edição do Programa de Preparação - UFRJ Consulting Club Apostila

2.8.3. Problem Solving

1) What is 30% of the square root of 85?

a) 62.4

b) 35.7

c) 15.6

d) 2.7

e) 0.23

Resolução:

I. Analisando as alternativas, é possível perceber que os valores estão bastante


distantes, o que nos garante uma faixa de erro aceitável. Vamos aproximar os
1
números, portanto, calculando 3
da raiz quadrada de 81, pois o resultado será

suficientemente próximo do real;

II. Facilmente calculamos a raiz quadrada de 81, que é 9;

1
III. Por fim, encontramos 3
de 9, que é 3;

IV. Concluímos que a única alternativa possível, próxima do valor encontrado


pelas aproximações, é 2,7.

Resposta correta: D

● Note que poderíamos, ainda, ter resolvido esse mesmo problema de


outras maneiras, como, por exemplo, aproximando a raiz quadrada de 85
para 9 e calculando 30% sobre esse resultado → 30% de 9 = 3 × (10% de 9) =
3 × 0,9 = 2,7.
● Alguns problemas de GMAT parecem requerer cálculos e computações
longas e complexas. Entretanto, raramente esse é o caso. Assim, nesse tipo
de questão, sempre opte por fazer uma aproximação. Isso é válido
essencialmente se as opções de resposta têm valores muito distantes.

2) A rectangular door to a bank's safe is twice as long as it is wide. If its


perimeter is 20 feet, then the dimensions of the bank door are?

57
14ª edição do Programa de Preparação - UFRJ Consulting Club Apostila

16 7
a) 2
𝑏𝑦 2

20 10
b) 2
𝑏𝑦 2

10 5
c) 2
𝑏𝑦 2

20 10
d) 3
𝑏𝑦 3

e) 6 by 4

Resolução:

I. Sabemos que o perímetro é dado por: 2×(𝑎𝑙𝑡𝑢𝑟𝑎) + 2×(𝑙𝑎𝑟𝑔𝑢𝑟𝑎);

É possível observar que a resposta deve satisfazer a equação acima,


fornecendo um perímetro igual a 20 e respeitando a proporção de 2:1, como
informado no enunciado;

II. Assim, podemos eliminar logo as alternativas (A) e (E), pois os valores
apresentados para os lados não respeitam a proporção de 2:1.

III. Por fim, substituímos os valores das demais alternativas na fórmula do


perímetro para encontrar a opção com valores que resultem em um
perímetro igual a 20. Desse modo, eliminamos as alternativas (B) e (C), pois
20 10 10 5
(2× 2
+ 2× 2
≠20) e (2× 2
+ 2× 2
≠20).

Resposta correta: D

3) The sum of four consecutive positive integers is 𝑧. What is the sum of the next
four consecutive integers in terms of 𝑧?

a) 10𝑧

b) 5𝑧

c) 𝑧 + 4

d) 2𝑧 + 4

e) 𝑧 + 16

Resolução:

I. Para facilitar a resolução, aplicamos a técnica de substituição: 𝑧 = 1 + 2 + 3 + 4;

58
14ª edição do Programa de Preparação - UFRJ Consulting Club Apostila

II. Concluímos, assim, que 𝑧 = 10;

III. Calculamos a soma dos próximos quatro inteiros consecutivos → 5 + 6 + 7 + 8 =


26;

IV. Em termos de 𝑧, equivalente a 10, a nova soma é igual a z + 16 (10 + 16 = 26).

Resposta correta: E

4) The cost of a coat increased 20%, only then to decrease 25%. What was the
final change in the price of the coat relative to its original price?

a) 10% increase

b) 10% decrease

c) 5% increase

d) 5% decrease

e) No change

Resolução:

I. Em questões de porcentagem, em que o valor original não é definido, é


sempre uma boa alternativa definir um número arbitrário para facilitar os
cálculos. Considerando o custo original do casaco como $100, teremos um
problema muito mais simples de resolver;

II. O aumento de 20% (1,20 × $100) leva o preço do casaco para $120;

III. A redução de 25% (0,75 × $120) leva o preço do casaco para $90;

IV. Já que o preço final é $90, podemos concluir que houve uma redução de 10%
sobre o preço inicial ($100 − $90 = $10 = 10% de $100).

Resposta correta: B

● Note que poderíamos ainda resolver diretamente, chamando o preço


original de p. O aumento levaria o preço para 1,2𝑝 e para encontrar o valor
final, após o decréscimo, faríamos 0,75 × (1,2𝑝) = 0,9𝑝, o que mostraria a
redução total de 10% sobre o preço original.

59
14ª edição do Programa de Preparação - UFRJ Consulting Club Apostila

5) What is the smallest integer 𝑥 for which 25𝑥 > 510?

a) 0

b) 5

c) 6

d) 15

e) 100

Resolução:

𝑥 2 𝑥 2𝑥
I. Fatorando, temos 25 = (5 ) = 5 ;

II. Agora, devemos encontrar o menor inteiro de x para o qual 52𝑥 > 510;

III. Pelas propriedades de inequação, precisamos encontrar 2𝑥 > 10. O menor


valor de 𝑥 que satisfaz essa condição é 6.

Resposta correta: C

6) The sequence 𝑎1, 𝑎2, 𝑎3, 𝑎4, 𝑎5 is such that 𝑎𝑛 = 𝑎𝑛−1 + 5, for 2 ≤ 𝑛 ≤ 5. If 𝑎5 = 31,

what is the value of 𝑎1?

a) 1

b) 6

c) 11

d) 16

e) 21

Resolução:

I. Do enunciado, sabemos que 𝑎𝑛 = 𝑎𝑛−1 + 5, logo 𝑎𝑛 − 𝑎𝑛−1 = 5, desde que 2 ≤ 𝑛 ≤

5. Então:

𝑎5 − 𝑎4 = 5

𝑎4 − 𝑎3 = 5

𝑎3 − 𝑎2 = 5

60
14ª edição do Programa de Preparação - UFRJ Consulting Club Apostila

𝑎2 − 𝑎1 = 5

II. Somando as equações, concluímos que 𝑎5 − 𝑎1 = 20;

III. Como o enunciado nos diz que 𝑎5 = 31, substituindo na equação anterior

encontramos que 𝑎1 = 11.

Resposta correta: C

● Note que a questão acima trata de uma progressão aritmética (P.A.), já


que um termo qualquer dessa sequência é obtido a partir da soma do
termo anterior com um valor R. Assim, para solucionar o problema,
poderíamos ter utilizado a fórmula 𝑎𝑛 = 𝑎1 + (𝑛 − 1) * 𝑅, em que n seria 5,

pois temos o valor de 𝑎5 e R seria igual a 5. Logo, teríamos que:

𝑎5 = 𝑎1 + (5 − 1) * 5 ∴ 31 = 𝑎1 + 20 ∴ 𝑎1 = 11.

6) In the figure above, what is the area of triangular region 𝐵𝐶𝐷?

a) 4 2

b) 8

c) 8 2

d) 16

e) 16 2

Resolução:

2 2
I. Pelo Teorema de Pitágoras, 𝐵𝐷 = 4 + 4 = 4 2;

61
14ª edição do Programa de Preparação - UFRJ Consulting Club Apostila

1
II. A área do ∆BCD, então, é igual a 2
× 4 × 4 2 = 8 2.

Resposta correta: C

7) What is the sum of the odd integers from 35 to 85, inclusive?

a) 1,560

b) 1,500

c) 1,240

d) 1,120

e) 1,100

Resolução:

I. Os números inteiros ímpares de 35 a 85 formam uma sequência tal que o 1º


termo é igual a 35, e os termos subsequentes têm sempre duas unidades a
mais. Temos, assim, uma progressão aritmética (P.A.). Logo, para qualquer
termo dessa sequência, seu valor é dado por: 35 + (𝑛 − 1) × 2, que segue a
fórmula geral de uma P.A. (𝑎𝑛 = 𝑎1 + (𝑛 − 1) × 𝑅). Então, a soma

35 + 37 + 39 + … + 85 pode ser escrita como:

1º termo: 35 = 35 + 0 × 2

2º termo: 37 = 35 + 1 × 2

3º termo: 39 = 35 + 2 × 2

4º termo: 41 = 35 + 3 × 2

...

+ 26º termo: 85 = 35 + 25 × 2 .;

——————————————————

SOMA = 35 × 26 + (1+2+3+...+25) × 2;

II. Se 𝑠 = 1 + 2 + 3 + … + 25, então 2𝑠 = (1 + 2 + 3 + … + 25) + (25 + 24 + 23 + … + 1);


III. Reorganizando, temos que 2𝑠 = (1 + 25) + (2 + 24) + (3 + 23) + … + (25 + 1) = 25 ×

62
14ª edição do Programa de Preparação - UFRJ Consulting Club Apostila

26;
25×26
IV. Logo, 𝑠 = 2
;
25×26
V. Temos, portanto: SOMA = 35 × 26 + 2
× 2 = 910 + 325 × 2 = 1560

Resposta correta: A

8) A couple decides to have four children. If they succeed in having four children
and each child is equally likely to be a boy or a girl, what is the probability
that they will have exactly two girls and two boys?

3
a) 8

1
b) 4

3
c) 16

1
d) 8

1
e) 16

Resolução:

I. Representaremos a ordem de nascimento das crianças com uma sequência


de quatro letras, H para homens e M para mulheres;

II. Realizando uma contagem simples, podemos concluir que há duas


possibilidades para o primeiro nascimento (H ou M), assim como para o
4
segundo, o terceiro e o último, totalizando 2 = 16 sequências possíveis;

III. Em uma tabela, representamos as sequências que não correspondem ao que


é pedido pelo problema (nascimentos diferentes de exatamente dois homens
e duas mulheres):

63
14ª edição do Programa de Preparação - UFRJ Consulting Club Apostila

Homens Mulheres Sequências Nº de sequências

0 4 MMMM 1

1 3 HMMM, MHMM, 4
MMHM, MMMH

3 1 MHHH, HMHH, HHMH, 1


HHHM

4 0 HHHH 4

IV. Existem dez casos não favoráveis à situação do problema. Logo, restam
seis casos favoráveis. Logo, restam seis casos favoráveis. A probabilidade de o
6 3
casal ter exatamente dois filhos e duas filhas é, portanto, igual a 16
= 8
;

Resposta correta: A

2.9. Exercícios extras

2.9.1. Critical Reasoning

1) Eating beets significantly lowers the risk of cancer, according to an article in


a nutritional magazine. The article refers to a study that found that people
who consumed one or more beets per day were half as likely to be
diagnosed with the disease as people who did not.

Which of the following, if true, most weakens the argument in the magazine
article?

a) Another study found that people who consumed one tablespoon of flax seed
oil per day were more than four times less likely to be diagnosed with cancer
as those who did not.

b) Participants in the study reported consuming no vegetables other than

64
14ª edição do Programa de Preparação - UFRJ Consulting Club Apostila

beets.

c) The study was only conducted in one city.

d) In another experiment, cancer patients who ate one or more beets per day
were no more likely to recover than those who ate no beets.

e) The participants in the study who ate beets were more likely to exercise
regularly than those who did not eat beets.

2) Wine Company Representative: The corks of red wine bottles pose a threat to
the environment because they are treated with chemicals that are especially
toxic in landfills. However, the new cork that our company developed, which
will be adopted by the entire red wine industry, represents a solution. Since
the new cork is natural and not treated with chemicals, when the industry
completes its transition to the new cork, there will no longer be any threat to
landfills from red wine corks.

Which of the following, if true, most weakens the argument above?

a) The industry's transition to the new red wine corks will take years, allowing
thousands of old corks to pollute landfills.

b) Even after the industry's transition to new corks, a large number of wine
bottles with old corks will continue to be consumed.

c) The new corks take considerably longer to produce.

d) Production of the new cork emits more toxic fumes than were emitted in the
production of the old cork.

e) The new corks are more expensive than the old corks.

3) Most scientists agree that new lines of interdisciplinary research are the
need of the hour. Even government committees on science have stressed the
need for more interdisciplinary projects. Yet, of ten proposals for new
interdisciplinary projects last year, only one was successfully funded. Some
have suggested this means that as yet researchers are not coming up with
sufficiently persuasive projects, or that their proposals are not of high

65
14ª edição do Programa de Preparação - UFRJ Consulting Club Apostila

enough quality, or even that the reputation of these researchers is not high
enough. However, the real reason probably lies in the way funding is
organized. Funding is still allocated according to the old categories and
there are no funds specifically for research that overlaps different subject
areas.

The two parts underlined are related to each other in which of the following
ways?

a) The first is a finding that the author finds unacceptable; the second is the
author’s own position.

b) The first is a finding that the author attempts to account for; the second is a
finding that contradicts the author’s main conclusion.

c) The first is a fact that the author attempts to account for. The second is data
that explicitly supports the author’s main conclusion.

d) The first is a position that the author opposes; the second is the author’s
main position.

e) The first is a situation that the author finds paradoxical; the second is an
assumption that the author uses to reinforce the paradox.

4) Early data on seat-belt use showed that seat-belt wearers were less likely to
be killed in road accidents. Hence, it was initially believed that wearing a
seat-belt increased survival chances in an accident. But what the early
analysts had failed to see was that cautious drivers were more likely to wear
the belts and were also less likely to cause big accidents, while reckless
drivers were more likely to be involved in big accidents and were less likely to
wear the belts.

Which of the following, if true, could an opponent of the view presented


above best cite as a reason for recommending continued use of seatbelts?

a) Careful drivers who are involved in accidents caused by reckless drivers,


would be more likely to survive if wearing a belt.

b) All drivers should be required by law to wear a belt.

66
14ª edição do Programa de Preparação - UFRJ Consulting Club Apostila

c) The ratio of big to small road accidents is very small.

d) In fatal accidents seat-belt wearers in the front seat are less likely to survive
than those wearing seat belts in the back seat.

e) On average, careful drivers pay lower insurance premiums than do drivers


who have been involved in accidents.

5) Scientists investigating a rare metabolic disorder hypothesized that obesity


was a predisposing factor in the development of the disease. A study of
twenty patients found that, on average, the patients were close to the
normal weight for their height.

Before concluding that obesity is not a predisposing factor, the researchers


would find the answer to which of the following questions most useful?

a) Are the patients above or below normal height?

b) Were any of the patients underweight when the disorder was diagnosed?

c) Does weight loss reduce the severity of the symptoms?

d) Have the patients always been close to the normal weight for their heights?

e) How many of the patients had obese parents?

6) In research designed to investigate the possibility of animals developing


friendship with other, unrelated, members of their species, a group of 29
chimpanzees were reared together for 15 years. At the end of that time the
chimps were presented with two options for obtaining food: press a lever and
feed themselves, or press another identical lever and feed themselves, and at
the same time deliver food to the chimp next door. (The chimps were able to
see each other). The researchers found that the chimps were no more likely
to choose the lever that fed a neighbor. The researchers concluded that the
chimps had no concept of friendship. However, one critic has suggested that
the animals were in an artificial environment from which little can be
concluded, and that, at the least, the test ought to have involved the animals
being able to touch.

67
14ª edição do Programa de Preparação - UFRJ Consulting Club Apostila

What role do the parts underlined play in the argument above?

a) The first is a position that the critic opposes. The second is a position that the
critic supports.

b) The first is an observation that supports the researchers’ position. The second
is an observation that opposes the researchers’ position.

c) The first is a finding on which the researchers base their conclusion. The
second is a suggestion that might cast doubt on that finding.

d) The first is an observation that supports the critic’s conclusion. The second is
the critic’s conclusion.

e) The first is part of the evidence that the critic disputes. The second is a
suggestion that the researchers do not accept.

7) Jay: We have too many people working on each of our computers in the
office. The high frequency of breakdowns is due to too many people handling
the same hardware.

Ada: We have just as many people working in our office, yet we hardly need
any repairs to our systems. Our systems must be more robust than yours.

Ada’s argument would be most strengthened by providing data on the:

a) actual number of people in the two offices.

b) type of computers that are in both offices.

c) ratio of computers to users in her office.

d) number of visits by computer engineers to service the computers in Jay’s


office.

e) number of computers in Jay’s office.

8) Refer to the extract in the previous question.

Jay apparently believes that:

a) he has the best available hardware.

68
14ª edição do Programa de Preparação - UFRJ Consulting Club Apostila

b) the frequency of breakdowns in his office is above average.

c) software specifications are not important in his office.

d) no other office has a similar ratio of computers to users.

e) he does not need more people working in his office.

9) Thousands of people have tonsillectomies every year and all live normal lives
after the operation. We can conclude, from this observation, that the tonsils
have no function in the body.

The argument would be most weakened by which of the following, if it were


true?

a) People live normal lives after appendectomies, but the appendix is known to
be part of the digestive system.

b) Another part of the body can take over the function of the tonsils if they are
removed.

c) The tonsils have been shown to have a vital role to play in the physiology of
laboratory rabbits and guinea pigs.

d) The human tonsil develops as part of the immune system, a system of vital
importance in defense against disease.

e) Tonsillectomies are performed only when the tonsils become seriously


infected.

10) Photography is no longer an art form. Nowadays everyone has access to


digital cameras that only need to be pointed at the subject in order to
generate a perfect image.

The writer of the argument apparently assumes that:

a) the selection of the subject is not an important artistic factor in


photography.

b) digital cameras will continue to improve in quality.

69
14ª edição do Programa de Preparação - UFRJ Consulting Club Apostila

c) digital cameras can never go wrong.

d) photography with all other types of camera is an art form.

e) art is not perfect.

11) Studies in restaurants show that the tips left by customers who pay their bill
in cash tend to be larger when the bill is presented on a tray that bears a
credit card logo. Consumer psychologists hypothesize that simply seeing a
credit-card logo makes many credit card holders willing to spend more
because it reminds them that their spending power exceeds the cash they
have immediately available.

Which of the following, if true, most strongly supports the psychologists'


interpretation of the studies?

a) The effect noted in the studies is not limited to patrons who have credit
cards.

b) Patrons who are under financial pressure from their credit-card obligations
tend to tip less when presented with a restaurant bill on a tray with a
credit-card logo than when the tray has no logo.

c) In virtually all of the cases in the studies, the patrons who paid bills in cash
did not possess credit cards.

d) In general, restaurant patrons who pay their bills in cash leave larger tips
than do those who pay by credit card.

e) The percentage of restaurant bills paid with a given brand of credit card
increases when that credit card's logo is displayed on the tray with which the
bill is presented.

12) Although parapsychology is often considered a pseudoscience, it is in fact a


genuine scientific enterprise, for it uses scientific methods such as controlled
experiments and statistical tests of clearly stated hypotheses to examine the
questions it raises.

The conclusion above is properly drawn if which of the following is assumed?

70
14ª edição do Programa de Preparação - UFRJ Consulting Club Apostila

a) If a field of study can conclusively answer the questions it raises, then it is a


genuine science.

b) Since parapsychology uses scientific methods, it will produce credible results.

c) Any enterprise that does not use controlled experiments and statistical tests
is not genuine science.

d) Any field of study that employs scientific methods is a genuine scientific


enterprise.

e) Since parapsychology raises clearly statable questions, they can be tested in


controlled experiments.

13) Delta Products Inc. has recently switched at least partly from older
technologies using fossil fuels to new technologies powered by electricity. The
question has been raised whether it can be concluded that for a given level
of output Delta's operation now causes less fossil fuel to be consumed than it
did formerly. The answer, clearly, is yes, since the amount of fossil fuel used to
generate the electricity needed to power the new technologies is less than
the amount needed to power the older technologies, provided level of output
is held constant.

In the argument given, the two underlined portions play which of the
following roles?

a) The first identifies the content of the conclusion of the argument; the second
provides support for that conclusion.

b) The first provides support for the conclusion of the argument; the second
identifies the content of that conclusion.

c) The first states the conclusion of the argument; the second calls that
conclusion into question.

d) The first provides support for the conclusion of the argument; the second
calls that conclusion into question.

e) Each provides support for the conclusion of the argument,

71
14ª edição do Programa de Preparação - UFRJ Consulting Club Apostila

14) The irradiation of food kills bacteria and thus retards spoilage. However, it
also lowers the nutritional value of many foods. For example, irradiation
destroys a significant percentage of whatever vitamin B1 a food may
contain. Proponents of irradiation point out that irradiation is no worse in
this respect than cooking. However, this fact is either beside the point, since
much irradiated food is eaten raw, or else misleading, since .

Which of the following most logically completes the argument?

a) Many of the proponents of irradiation are food distributors who gain from
foods' having a longer shelf life.

b) It is clear that killing bacteria that may be present on food is not the only
effect that irradiation has.

c) Cooking is usually the final step in preparing food for consumption, whereas
irradiation serves to ensure a longer shelf life for perishable foods.

d) Certain kinds of cooking are, in fact, even more destructive of vitamin B1


than carefully controlled irradiation is.

e) For food that is both irradiated and cooked, the reduction of vitamin B1
associated with either process individually is compounded.

15) Scientists typically do their most creative work before the age of forty. It is
commonly thought that this happens because aging by itself brings about a
loss of creative capacity. However, studies show that of scientists who
produce highly creative work beyond the age of forty, a disproportionately
large number entered their field at an older age than is usual. Since by the
age of forty the large majority of scientists have been working in their field
for at least fifteen years, the studies' finding strongly suggests that the real
reason why scientists over forty rarely produce highly creative work is not
that they have aged but rather that scientists over forty have generally spent
too long in their field.

In the argument given, the two portions underlined play which of the
following roles?

a) The first is a claim, the accuracy of which is at issue in the argument; the

72
14ª edição do Programa de Preparação - UFRJ Consulting Club Apostila

second is a conclusion drawn on the basis of that claim.

b) The first is an objection that has been raised against a position defended in
the argument; the second is that position.

c) The first is evidence that has been used to support an explanation that the
argument challenges; the second is that explanation.

d) The first is evidence that has been used to support an explanation that the
argument challenges; the second is a competing explanation that the
argument favors.

e) The first provides evidence to support an explanation that the argument


favors; the second is that explanation.

Gabarito:

1. E 6. C 11. B
2. B 7. C 12. D
3. C 8. B 13. B
4. A 9. B 14. E
5. D 10. A 15. E

2.9.2. Data Sufficiency


𝑥 −𝑥
1) If x is an integer, 9 − 9 = 𝑏?

𝑥 −𝑥
(1) 3 − 3 = 𝑏 + 2

(2) 𝑥 > 0

a) Statement (1) ALONE is sufficient, but statement (2) alone is not sufficient.

b) Statement (2) ALONE is sufficient, but statement (1) alone is not sufficient.

c) BOTH statements TOGETHER are sufficient, but NEITHER statement ALONE


is sufficient.

d) EACH statement ALONE is sufficient.

e) Statements (1) and (2) TOGETHER are not sufficient.

73
14ª edição do Programa de Preparação - UFRJ Consulting Club Apostila

2) What is the tens digit of a positive integer 𝑥?

(1) 𝑥 divided by 100 has a remainder of 30.

(2) 𝑥 divided by 110 has a remainder of 30.

a) Statement (1) ALONE is sufficient, but statement (2) alone is not sufficient.

b) Statement (2) ALONE is sufficient, but statement (1) alone is not sufficient.

c) BOTH statements TOGETHER are sufficient, but NEITHER statement ALONE


is sufficient.

d) EACH statement ALONE is sufficient.

e) Statements (1) and (2) TOGETHER are not sufficient.

3) In a survey of 200 college graduates, 30 percent said they had received


student loans during their college careers, and 40 percent said they had
received scholarships. What percent of those surveyed said that they had
received neither student loans nor scholarships during their college careers?

(1) 25 percent of those surveyed said that they had received scholarships but no
loans.

(2) 50 percent of those surveyed who said that they had received loans also said
that they had received scholarship.

a) Statement (1) ALONE is sufficient, but statement (2) alone is not sufficient.

b) Statement (2) ALONE is sufficient, but statement (1) alone is not sufficient.

c) BOTH statements TOGETHER are sufficient, but NEITHER statement ALONE


is sufficient.

d) EACH statement ALONE is sufficient.

e) Statements (1) and (2) TOGETHER are not sufficient.

74
14ª edição do Programa de Preparação - UFRJ Consulting Club Apostila

4) What is the area of triangular region 𝐴𝐵𝐶 above?

(1) The product of 𝐵𝐷 and 𝐴𝐶 is 20.

(2) 𝑥 = 45

a) Statement (1) ALONE is sufficient, but statement (2) alone is not sufficient.

b) Statement (2) ALONE is sufficient, but statement (1) alone is not sufficient.

c) BOTH statements TOGETHER are sufficient, but NEITHER statement ALONE


is sufficient.

d) EACH statement ALONE is sufficient.

e) Statements (1) and (2) TOGETHER are not sufficient.

5) What is the value of integer 𝑛?

(1) 𝑛 × (𝑛 + 1) = 6

2𝑛
(2) 2 = 16

a) Statement (1) ALONE is sufficient, but statement (2) alone is not sufficient.

b) Statement (2) ALONE is sufficient, but statement (1) alone is not sufficient.

c) BOTH statements TOGETHER are sufficient, but NEITHER statement ALONE


is sufficient.

d) EACH statement ALONE is sufficient.

e) Statements (1) and (2) TOGETHER are not sufficient.

75
14ª edição do Programa de Preparação - UFRJ Consulting Club Apostila

6) The figure above represents a circle graph of Company H’s total expenses
broken down by the expenses for each of its five divisions. If 𝑂 is the center of
the circle and if Company H’s total expenses are $5,400,000, what are the
expenses for division 𝑅?

(1) 𝑥 = 94ᵒ

(2) The total expenses for division 𝑆 and 𝑇 are twice as much as the expenses for
division 𝑅.

a) Statement (1) ALONE is sufficient, but statement (2) alone is not sufficient.

b) Statement (2) ALONE is sufficient, but statement (1) alone is not sufficient.

c) BOTH statements TOGETHER are sufficient, but NEITHER statement ALONE


is sufficient.

d) EACH statement ALONE is sufficient.

e) Statements (1) and (2) TOGETHER are not sufficient.

7) A box contains only red chips, white chips, and blue chips. If a chip is
randomly selected from the box, what is the probability that the chip will be
either white or blue?

1
(1) The probability that the chip will be blue is 5
.

1
(2) The probability that the chip will be red is 3
.

a) Statement (1) ALONE is sufficient, but statement (2) alone is not sufficient.

b) Statement (2) ALONE is sufficient, but statement (1) alone is not sufficient.

c) BOTH statements TOGETHER are sufficient, but NEITHER statement ALONE

76
14ª edição do Programa de Preparação - UFRJ Consulting Club Apostila

is sufficient.

d) EACH statement ALONE is sufficient.

e) Statements (1) and (2) TOGETHER are not sufficient.

8) In the following rectangular coordinate system, if 𝑂𝑃 < 𝑃𝑄, is the area of


region 𝑂𝑃𝑄 greater than 48?

(1) The coordinates of point 𝑃 are (6,8).

(2) The coordinates of point 𝑄 are (13,0).

a) Statement (1) ALONE is sufficient, but statement (2) alone is not sufficient.

b) Statement (2) ALONE is sufficient, but statement (1) alone is not sufficient.

c) BOTH statements TOGETHER are sufficient, but NEITHER statement ALONE


is sufficient.

d) EACH statement ALONE is sufficient.

e) Statements (1) and (2) TOGETHER are not sufficient.

9) In the following expression, 𝑥 ≠ 0 and 𝑛 ≠ 0, what is the value of 𝑆?

2
𝑛
𝑆= 1 2
𝑥
+ 3𝑥

(1) 𝑥 = 2𝑛

77
14ª edição do Programa de Preparação - UFRJ Consulting Club Apostila

1
(2) 𝑛 = 2

a) Statement (1) ALONE is sufficient, but statement (2) alone is not sufficient.

b) Statement (2) ALONE is sufficient, but statement (1) alone is not sufficient.

c) BOTH statements TOGETHER are sufficient, but NEITHER statement ALONE


is sufficient.

d) EACH statement ALONE is sufficient.

e) Statements (1) and (2) TOGETHER are not sufficient.

10) If [𝑥] denotes the greatest integer less than or equal to 𝑥, is [𝑥] = 0?

(1) 5𝑥 + 1 = 3 + 2𝑥

(2) 0 < 𝑥 < 1

a) Statement (1) ALONE is sufficient, but statement (2) alone is not sufficient.

b) Statement (2) ALONE is sufficient, but statement (1) alone is not sufficient.

c) BOTH statements TOGETHER are sufficient, but NEITHER statement ALONE


is sufficient.

d) EACH statement ALONE is sufficient.

e) Statements (1) and (2) TOGETHER are not sufficient.

11) Is 𝑟 × 𝑠 × 𝑡 = 1?

(1) 𝑟 × 𝑠 = 1

(2) 𝑠 × 𝑡 = 1

a) Statement (1) ALONE is sufficient, but statement (2) alone is not sufficient.

b) Statement (2) ALONE is sufficient, but statement (1) alone is not sufficient.

c) BOTH statements TOGETHER are sufficient, but NEITHER statement ALONE


is sufficient.

d) EACH statement ALONE is sufficient.

e) Statements (1) and (2) TOGETHER are not sufficient.

78
14ª edição do Programa de Preparação - UFRJ Consulting Club Apostila

12) What is the value of 𝑛 in the following list?

𝑘, 𝑛, 12, 6, 17

(1) 𝑘 < 𝑛

(2) The median of the numbers in the list is 10.

a) Statement (1) ALONE is sufficient, but statement (2) alone is not sufficient.

b) Statement (2) ALONE is sufficient, but statement (1) alone is not sufficient.

c) BOTH statements TOGETHER are sufficient, but NEITHER statement ALONE


is sufficient.

d) EACH statement ALONE is sufficient.

e) Statements (1) and (2) TOGETHER are not sufficient.

13) Three machines, 𝐾, 𝑀, and 𝑃, working simultaneously and independently at


their respective constant rates, can complete a certain task in 24 minutes.
How long does it take Machine 𝐾, working alone at its constant rate, to
complete the task?

(1) Machines 𝑀 and 𝑃, working simultaneously and independently at their


respective constant rates, can complete the task in 36 minutes.

(2) Machines 𝐾 and 𝑃, working simultaneously and independently at their


respective constant rates, can complete the task in 48 minutes.

a) Statement (1) ALONE is sufficient, but statement (2) alone is not sufficient.

b) Statement (2) ALONE is sufficient, but statement (1) alone is not sufficient.

c) BOTH statements TOGETHER are sufficient, but NEITHER statement ALONE


is sufficient.

d) EACH statement ALONE is sufficient.

e) Statements (1) and (2) TOGETHER are not sufficient.

14) What is the value of 𝑛 in the equation −25 + 19 + 𝑛 = 𝑠?

(1) 𝑠 = 2

79
14ª edição do Programa de Preparação - UFRJ Consulting Club Apostila

𝑛
(2) 𝑠
=4

a) Statement (1) ALONE is sufficient, but statement (2) alone is not sufficient.

b) Statement (2) ALONE is sufficient, but statement (1) alone is not sufficient.

c) BOTH statements TOGETHER are sufficient, but NEITHER statement ALONE


is sufficient.

d) EACH statement ALONE is sufficient.

e) Statements (1) and (2) TOGETHER are not sufficient.

15) The table above shows the distance, in kilometers, by the most direct route,
between any two of the four cities, 𝑅, 𝑆, 𝑇, and 𝑈. For example, the distance
between City 𝑅 and City 𝑈 is 62 kilometers. What is the value of 𝑥?

(1) By the most direct route, the distance between 𝑆 and 𝑇 is twice the distance
between 𝑆 and 𝑅.

(2) By the most direct route, the distance between 𝑇 and 𝑈 is 1.5 times the
distance between 𝑅 and 𝑇.

a) Statement (1) ALONE is sufficient, but statement (2) alone is not sufficient.

b) Statement (2) ALONE is sufficient, but statement (1) alone is not sufficient.

c) BOTH statements TOGETHER are sufficient, but NEITHER statement ALONE


is sufficient.

d) EACH statement ALONE is sufficient.

e) Statements (1) and (2) TOGETHER are not sufficient.

80
14ª edição do Programa de Preparação - UFRJ Consulting Club Apostila

Gabarito:

1. A 6. A 11. E
2. A 7. B 12. C
3. D 8. A 13. A
4. A 9. A 14. D
5. B 10. D 15. B

2.9.3 Problem Solving

1) 2𝑥 + 3𝑦 = 16 and 𝑦 = −6𝑥. What −𝑥 equals to?

a) −1

4
b) 5

4
c) − 5

5
d) − 4

e) 1

(70+𝑥)
2) For which of the following values of 𝑥 is the fraction 𝑥
an integer?

a) 0

b) 6

c) 7

d) 8

e) 9

3) Points 𝐴, 𝐵, 𝐶, and 𝐷, in that order, lie on a line. If 𝐴𝐵 = 3 𝑐𝑚, 𝐴𝐶 = 4 𝑐𝑚, and 𝐵𝐷 =


6 𝑐𝑚, what is 𝐶𝐷, in centimeters?

a) 1

b) 2

81
14ª edição do Programa de Preparação - UFRJ Consulting Club Apostila

c) 3

d) 4

e) 5

4) Company C produces toy trucks at a cost of $5.00 each for the first 100 trucks
and $3.50 for each additional truck. If 500 toy trucks were produced by
Company C and sold for $10.00 each, what was Company C’s gross profit?

a) $2,250

b) $2,500

c) $3,100

d) $3,250

e) $3,500

5) During the past week, a local medical clinic tested 𝑁 individuals for two
1
infections. If 3
of those tested had infection A and, of those with infection A,
1
5
also had infection B, how many individuals did not have both infection A

and B?

𝑁
a) 15

4𝑁
b) 15

𝑁
c) 5

14𝑁
d) 15

4𝑁
e) 15

6) In a local intramural basketball league, there are ten teams, and each team
plays every other team exactly one time. Assuming that each game is played
by only two teams, how many games are played in total?

a) 100

82
14ª edição do Programa de Preparação - UFRJ Consulting Club Apostila

b) 90

c) 50

d) 45

e) 10

𝑥 𝑥
7) If 4
is 2 more than 8
, then 𝑥 equals:

a) 4

b) 8

c) 16

d) 32

e) 64

5,98×601,5
8) Of the following, the closest approximation to 15,79
is:

a) 5

b) 15

c) 20

d) 25

e) 225

9) What is the perimeter, in meters, of a rectangular garden 6 meters wide that


has the same area as a rectangular playground 16 meters long and 12
meters wide?

a) 48

b) 56

c) 60

d) 76

83
14ª edição do Programa de Preparação - UFRJ Consulting Club Apostila

e) 192

10) A project manager needs a group of five people from a total of five men and
five women. How many possible group combinations exist such that no
group is homogeneous?

a) 72

b) 68

c) 48

d) 82

e) 250

11) In a classroom of eight, there are also eight desks. They can sit in whichever
way they want, except that two students are preselected to sit in either end
seats. In how many ways can this occur?

a) 490

b) 1000

c) 1200

d) 1440

e) 1690

12) If a square region has area 𝑛, what is the length of the diagonal of the
square in terms of 𝑛?

a) 2𝑛

b) 𝑛

c) 2 𝑛

d) 2𝑛

2
e) 2𝑛

84
14ª edição do Programa de Preparação - UFRJ Consulting Club Apostila

13) One inlet pipe fills an empty tank in 5 hours. A second inlet pipe fills the same
2
tank in 3 hours. If both pipes are used together, how long will it take to fill 3

of the tank?

8
a) 15
hr

3
b) 4
hr

5
c) 4
hr

15
d) 8
hr

8
e) 3
hr

14) In the figure, what is the value of 𝑎?

a) 30

b) 45

c) 60

d) 72

e) 90

85
14ª edição do Programa de Preparação - UFRJ Consulting Club Apostila

15) In the rectangular coordinate system above, the area of ∆RST is:

𝑏𝑐
a) 2

𝑏(𝑐−1)
b) 2

𝑐(𝑏−1)
c) 2

𝑎(𝑐−1)
d) 2

𝑐(𝑎−1)
e) 2

Gabarito:

1. E 6. D 11. D
2. C 7. C 12. A
3. E 8. B 13. C
4. C 9. D 14. D
5. D 10. E 15. B

2.10. Conclusão
Tendo em vista todas as etapas da prova e como solucionar questões, há formas de
treinar cada vez mais para o GMAT por meio do The Official Guide for GMAT e
simulados oferecidos pelas próprias firmas de consultoria.

Além disso, nessa apostila, nas seções acima, são disponibilizadas inúmeras questões a
serem treinadas e suas respectivas resoluções, a fim de preparar os candidatos para
passar nessa etapa dos processos seletivos.

86
14ª edição do Programa de Preparação - UFRJ Consulting Club Apostila

3. Business Case

3.1 O que é?

3.1.1. Definição

O business case é uma prova de múltipla escolha de caráter eliminatório,


presente na maioria dos processos seletivos das firmas de consultoria estratégica.
Ela constitui uma etapa muito importante, na medida que busca avaliar a
capacidade de raciocínio lógico aplicada ao mundo dos negócios. Portanto, é
demandado que o candidato analise gráficos e outras informações
disponibilizadas pelo enunciado, identifique variáveis-chave e realize cálculos
matemáticos eficientemente, de modo a gerar insights sobre a situação em
questão.

3.1.2. O que ela busca avaliar

O business case é uma avaliação cujos pontos centrais são: habilidade para
manusear e filtrar dados, capacidade de gestão de tempo e aptidão para realizar
cálculos matemáticos. Nessa perspectiva, é inegável que tais habilidades são
cruciais no cotidiano de uma pessoa, seja ela um consultor(a), empreendedor(a)
ou ainda estudante. Em suma, essas são qualidades altamente demandadas no
mercado.

Agora, após entender o que é um business case e quais são as habilidades


centrais desenvolvidas por ele, é possível focar em sua aplicabilidade dentro dos
processos seletivos de consultorias. De forma geral, as provas são realizadas em
inglês, prezando pela análise gráfica e formulação de insights conclusivos,
capazes de transmitir para os avaliadores o quão aguçado é o business sense1 do
candidato.

1
É a capacidade de lidar, resolver e definir questões relacionadas ao mundo dos negócios,
levando em conta os riscos e as oportunidades da análise.

87
14ª edição do Programa de Preparação - UFRJ Consulting Club Apostila

3.1.3. Como se preparar?

Pelo seu caráter lógico, o estudo prévio é extremamente útil para as provas de
business case - afinal, com a prática, é possível perceber a repetição de padrões
de raciocínio e perguntas. Portanto, para uma preparação eficiente, é indicada a
realização de simulados, disponíveis nos sites das próprias empresas e, também,
no material disponibilizado pelo UFRJ Consulting Club. Com isso, torna-se
possível se familiarizar com os gráficos e termos em inglês, assim como obter
uma melhor gestão de tempo, o que é essencial para a prova.

3.1.4. Como ela é aplicada em cada uma das grandes empresas de


consultoria estratégica?

Apesar de possuírem o mesmo propósito e estrutura central, é válido aprofundar


algumas características específicas do business case do BCG e da Bain. Além
disso, vale ressaltar que, atualmente, a McKinsey não conta com uma prova de
business case.

● BCG: a prova de business case faz parte da primeira etapa de avaliação do


processo de seleção. Para isso, ela é aplicada em conjunto a uma prova em
inglês de GMAT, totalizando 20 perguntas - 10 de GMAT e 10 de business
case, para serem respondidas em 45 minutos. Idealmente, recomenda-se
que sejam separados 25 minutos para a parte de business case. Além
disso, no BCG, os gráficos de cada questão costumam vir junto ao
enunciado, facilitando a etapa de filtragem das informações.

● Bain: a prova de business case é a segunda etapa do processo seletivo.


Nela, serão realizadas 15 perguntas em inglês com um prazo de 45
minutos, totalizando em média 3 minutos por questão. Além disso, o
formato da prova é um pouco diferente do adotado pelo BCG. Geralmente,
a avaliação conta com arquivos complementares nos quais estão expostos
os gráficos necessários para um grupo de questões e, também, algumas
informações iniciais de contextualização da empresa, como setor
econômico, principal produto ou serviço fornecido, objetivos da
companhia, entre outros. Nesse sentido, existe uma dificuldade a mais na

88
14ª edição do Programa de Preparação - UFRJ Consulting Club Apostila

prova: identificar qual ou quais gráficos contêm os dados capazes de


fornecer os insights que respondem corretamente às questões.

3.2. Como resolver a prova


Na avaliação de business case, a capacidade de gestão de tempo é fundamental para
um bom desempenho na prova. Contudo, a necessidade de se atentar ao tempo não
pode se traduzir em pressa ou falta de atenção. Nesse sentido, é interessante ter em
mente o método mais convencional para a resolução das questões. Vale ressaltar que
ele não é o único correto ou válido, e sim apenas uma opção. Por isso, desenvolver uma
estratégia própria pode ser importante para a prova, o que é possível apenas por meio
da prática.

● Assim como em provas de interpretação de texto do ENEM ou mesmo do


Ensino Médio, uma tática interessante é ler o enunciado antes de realizar
as suas análises, porque, assim, o candidato poderá focar apenas nas
informações necessárias para atender ao comando. Após fazer a sua
leitura, é importante ter claro quais são as informações chaves para
responder as perguntas.

● Dando sequência ao passo a passo, ciente da pergunta realizada no


enunciado e também do raciocínio necessário, o candidato deve seguir
para a análise gráfica, na qual serão fornecidos os dados necessários para
formular insights.

● Por fim, após chegar a uma resposta, o candidato deve conferir se o seu
raciocínio de fato atingiu a resposta desejada, levando em consideração
todos os pontos expostos no enunciado e nos gráficos. Tenha certeza de
que as informações dos gráficos foram exploradas ao máximo, de forma a
não desconsiderar nenhum dado importante.

● Caso a resposta encontrada não esteja em nenhuma das opções de


resposta não se desespere, pois isso irá provocar ainda mais gasto de
tempo! Nessa hora, é importante revisar o raciocínio e o comando:
provavelmente, o erro se encontra em alguma conta, unidade incorreta ou
até mesmo em informações desconsideradas.

89
14ª edição do Programa de Preparação - UFRJ Consulting Club Apostila

3.2.1. Entender o que de fato foi perguntado

Apesar de parecer uma ação trivial, o entendimento exato da pergunta realizada


no enunciado é fundamental para a formulação do raciocínio e, principalmente,
para a gestão do tempo. Uma estratégia útil para a boa compreensão do
enunciado é anotar as palavras chaves. Isso permite que o candidato se
mantenha atento às informações necessárias e, também, ao que é esperado
como resposta.

Para resolver com eficiência a prova, é importante focar em 2 aspectos durante a


preparação: desenvolver familiaridade com termos de business e aprender a
analisar gráficos corretamente e rapidamente.

3.2.2. Termos de business

Durante a prova, termos comuns no mundo dos negócios são abordados. Nesse
sentido, é importante saber o que eles significam para compreender as
informações passadas, assim como o que foi perguntado. Portanto, seguem as
definições de alguns deles:

● Market size é o tamanho de algum mercado em questão. Ele pode ser


quantificado em número de consumidores, unidades vendidas ou
faturamento, portanto, é sempre válido estar atento à unidade na qual a
informação é passada.

Exemplo:

90
14ª edição do Programa de Preparação - UFRJ Consulting Club Apostila

Ao analisarmos as informações dispostas no gráfico, observamos que o


mercado brasileiro de C&T (Cosmetics and Toiletries) era de 22 bilhões de
dólares em 2007. Portanto, podemos afirmar que o market size brasileiro
para os produtos cosméticos e de higiene pessoal era de 22 bilhões de
dólares.

● Market share é um termo em inglês que expressa a participação da


empresa no mercado em questão, ou seja, dentro do total de vendas de
um setor, qual é a parcela referente às vendas dessa firma.

Exemplo:

Ao analisarmos o gráfico, observamos a participação absoluta de cada


uma das empresas no setor de transmissão de energia elétrica no Brasil
em 2020. Nesse sentido, o cálculo do market share absoluto de uma
empresa é feito pela razão entre o seu volume de vendas e o volume total
daquele mercado.

● Relative Market Share (Participação de mercado relativa): é um


indicador que compara o market share de um player com o de outro,
normalmente com o líder de mercado, sendo abreviado pela sigla RMS.

É calculado pela razão entre o market share da empresa em questão e o


do líder do mercado. Caso estejamos tratando do RMS do próprio líder,
calculamos pela razão entre o market share do líder e o do segundo maior
player, que será a participação de mercado do seu principal competidor.

91
14ª edição do Programa de Preparação - UFRJ Consulting Club Apostila

Desse modo, o RMS do maior player do mercado será sempre acima de 1,


enquanto dos outros será sempre abaixo de 1.

Exemplo:

Imagine que uma empresa vendeu 100 mil dólares em bolas de futebol
ano passado, enquanto seu principal competidor, e maior player do
mercado, vendeu 400 mil dólares em bolas. Dessa forma, seu RMS será
0,25.

Para calcular o relative market share do líder e de outro player do


mercado, respectivamente, são utilizadas as seguintes fórmulas:

𝑀𝑎𝑟𝑘𝑒𝑡 𝑠ℎ𝑎𝑟𝑒 𝑑𝑜 𝑙í𝑑𝑒𝑟 𝑀𝑎𝑟𝑘𝑒𝑡 𝑠ℎ𝑎𝑟𝑒 𝑑𝑜 𝑝𝑙𝑎𝑦𝑒𝑟


𝑀𝑎𝑟𝑘𝑒𝑡 𝑠ℎ𝑎𝑟𝑒 𝑑𝑜 𝑠𝑒𝑔𝑢𝑛𝑑𝑜 𝑚𝑎𝑖𝑜𝑟 𝑝𝑙𝑎𝑦𝑒𝑟
𝑜𝑢 𝑀𝑎𝑟𝑘𝑒𝑡 𝑠ℎ𝑎𝑟𝑒 𝑑𝑜 𝑙í𝑑𝑒𝑟

● Compound Annual Growth Rate (CAGR): a taxa de crescimento anual


composta é a taxa de retorno necessária para um investimento crescer de
seu saldo inicial para o final. Também podendo ser utilizado para calcular
variações da taxa de crescimento de algum dado (geralmente, receita,
custos ou lucros). É calculado a partir da fórmula:
1

( )
𝑉𝐹
𝑉𝐼
𝑁
−1

Sendo n o intervalo de tempo analisado, VF o valor final e VI o valor inicial.

Exemplo:

92
14ª edição do Programa de Preparação - UFRJ Consulting Club Apostila

O gráfico demonstra a evolução da receita líquida da Natura durante o


período de 2002 e 2007. No eixo das abcissas, é possível ver a alteração
anual dessa métrica, cujas variações não são constantes, enquanto, no eixo
das ordenadas, é possível obter o CARG.

Essa estatística tem, por objetivo, informar que os crescimentos relativos


anuais de 34%, 33%, 29%, 21% e 11% ao longo dos seis anos são equivalentes
a um crescimento anual total de 25% no mesmo período.

● EBITDA: (Earnings before interest, taxes, depreciation and amortization)


significa os lucros antes dos juros, impostos, depreciações e amortizações.
Essa é uma métrica muito utilizada pelas empresas para medir o quão
eficientes são suas operações, já que mede apenas os ganhos que a
empresa teve a partir de si própria, sem contar fatores externos como
impostos por não as influenciarem de forma direta.

Exemplo:

● Costs (Custos) ou Expenses (Despesas): é todo o dinheiro gasto pela


empresa. Dividem-se em duas categorias: custos fixos e custos variáveis.

o Fixed costs (Custos fixos): custos que independem do volume de


vendas, ou seja, que não são diretamente afetados pelo aumento ou
redução da quantidade de produtos e/ou serviços comercializados.

93
14ª edição do Programa de Preparação - UFRJ Consulting Club Apostila

Um exemplo é o aluguel de uma fábrica. Seu valor deverá ser sempre


pago pela empresa, independente do volume produzido pela fábrica.

o Variable costs (Custos variáveis): custos que variam de acordo com


a quantidade de bens e serviços vendidos pela empresa.

Imagine que você está vendendo bolos de chocolate feitos por um


cozinheiro contratado. Caso o número de bolos produzidos aumente,
naturalmente será necessário comprar uma maior quantidade dos
ingredientes, como farinha e cacau, caracterizando-se, portanto,
como custos variáveis.

o Direct costs (Custos diretos): custos diretamente ligados aos serviços


ou produtos oferecidos pela empresa.

Alguns exemplos são os custos com a matéria prima de um produto e


a mão de obra direta, como a de funcionários relacionados à
prestação de serviço.

o Indirect costs (Custos indiretos): custos não ligados diretamente à


cadeia de produção ou de prestação de serviço.

Alguns exemplos são o aluguel do espaço, o seguro de


equipamentos e a mão de obra indireta, como a de funcionários
ligados à administração.

● Cost of Goods Sold (Custo de bens vendidos): é o custo dos materiais e


serviços necessários para a produção dos bens vendidos, abreviado,
frequentemente, como COGS, em português CMV.

Retomando o exemplo do bolo, a energia, a compra de equipamentos e


outros custos relacionados à produção dos bolos seriam computados
como COGS.

● SG&A (Selling, General & Administrative expenses): são todos os custos


de uma empresa não diretamente ligados à produção do produto da
companhia, ou seja, são gastos administrativos de uma firma.

Ainda no exemplo do bolo, o gasto com um entregador no serviço de


delivery seria computado como SG&A.

● Revenues (Receitas) ou Sales (Vendas): é todo o dinheiro arrecadado

94
14ª edição do Programa de Preparação - UFRJ Consulting Club Apostila

pela empresa oriundo da venda de seus produtos e/ou serviços ao


mercado.

● Gross Revenue (Receita Bruta): é o total obtido pelas vendas dos


produtos sem sofrer quaisquer deduções, como descontos ou devoluções.
Ao computar esses aspectos, obtém-se a receita líquida.

● Net Revenue or Net Sales (Receita Líquida): é o total de receita obtido


após o desconto dos retornos ou devoluções das vendas.

● Average Ticket/Price (Ticket Médio): refere-se ao valor médio gasto pelos


clientes por cada compra feita dentro do estabelecimento. Pode ser
calculado pela receita dividida pela quantidade de vendas efetuadas.

● Break Even Point: é o ponto onde o custo da operação é igual à receita


proveniente da mesma.

● Payback/Break-even Time: é a previsão do tempo que um investimento


irá demorar para cobrir seu custo inicial.

● Profit (Lucro) ou Earnings (Ganhos): lucro é o benefício financeiro,


descontados os custos, resultante das vendas de uma empresa. É
calculado, portanto, pela seguinte fórmula:

Profit = Revenue - costs

● Profit Margin/Net Margin (Margem de lucro): é a taxa que quanto do


dinheiro captado pela empresa é convertido em lucro.

A margem de lucro é calculada pela razão entre o lucro e a receita, sendo


normalmente expressa em termos de porcentagem.

● Depreciation (Depreciação): é a parcela de valor perdida sobre algum


bem ao longo do tempo. Assim como dinheiro, bens também perdem
valor ao longo do tempo e a depreciação é a contabilização de tal efeito.

● EBIT (Earnings Before Interests and Taxes): demonstra apenas o lucro


operacional da empresa, sem incluir despesas ou receitas financeiras.
Portanto, contabiliza apenas as atividades operacionais da empresa como
forma de geração de recursos, descartando possíveis aplicações paralelas,
como investimentos financeiros, juros sobre capital próprio, etc.

95
14ª edição do Programa de Preparação - UFRJ Consulting Club Apostila

● ROI (Return on Investment): é a métrica utilizada para determinar o


quanto a companhia terá de retorno sobre o custo do investimento. Um
ROI positivo, de forma geral, significa que o investimento é rentável. Pode
ser calculado de formas diferentes, sendo a mais popular:

𝐺𝑎𝑛ℎ𝑜𝑠 𝑜𝑏𝑡𝑖𝑑𝑜𝑠−𝐼𝑛𝑣𝑒𝑠𝑡𝑖𝑚𝑒𝑛𝑡𝑜 𝑖𝑛𝑖𝑐𝑖𝑎𝑙


𝐼𝑛𝑣𝑒𝑠𝑡𝑖𝑚𝑒𝑛𝑡𝑜 𝑖𝑛𝑖𝑐𝑖𝑎𝑙

● Stakeholder: em português, “parte interessada”, é aquele que possui


algum tipo de interesse nas atividades de uma empresa. Alguns exemplos
são: donos de um negócio, funcionários e acionistas.

● Player: empresa que atua em algum mercado.

● CAPEX: é ogasto de um negócio que visa gerar benefício futuro. Nesta


categoria, entram investimentos em ativos que possuem vida útil.

● OPEX: é o gasto direcionado ao funcionamento diário do negócio, como


salários​, manutenção​, reparos​, custos administrativos (luz, água, telefone,
etc), utilidades e etc. Em linhas gerais, o OPEX é o dinheiro que um
negócio despende para transformar seu inventário em receita e, tendo isso
em vista, também inclui a depreciação​ ​de​ ​máquinas​ ​e​ ​imóveis​
diretamente ligados à produção.

● Research and Development (R&D): em português, Pesquisa e


Desenvolvimento (P&D), é a área de uma companhia que tem como
função a inovação, seja ela em novos produtos, ou em processos.

● Customer Aquisition Cost (CAC): em português custo de aquisição de


clientes, é a parcela da receita que deve ser direcionada para que a
companhia adquira um novo cliente. Essa parcela pode incluir, por
exemplo, custos de propaganda e custos da equipe de vendas. É calculado
a partir da fórmula:

𝐼𝑛𝑣𝑒𝑠𝑡𝑖𝑚𝑒𝑛𝑡𝑜 𝑒𝑚 𝑚𝑎𝑟𝑘𝑒𝑡𝑖𝑛𝑔 + 𝐼𝑛𝑣𝑒𝑠𝑡𝑖𝑚𝑒𝑛𝑡𝑜 𝑒𝑚 𝑣𝑒𝑛𝑑𝑎𝑠


𝑁ú𝑚𝑒𝑟𝑜 𝑑𝑒 𝑐𝑙𝑖𝑒𝑛𝑡𝑒𝑠 𝑛𝑜𝑣𝑜𝑠

● B2B: Business-to-Business é o comércio entre empresas (sell-in), sem a


participação do consumidor final.

● B2C: Business-to-Consumer é o comércio direto da empresa com o


consumidor final (sell-out).

96
14ª edição do Programa de Preparação - UFRJ Consulting Club Apostila

● Branding: é a gestão da marca de uma companhia em aspectos como:


seu nome, valores, identidade e posicionamento, com o objetivo trazê-la
para mais perto do consumidor e alinhada com os valores internos da
companhia..

Já Rebranding é o mesmo processo aplicado a empresas já estabelecidas,


que desejam mudar sua imagem de alguma forma.

● Quick-Wins: é o conceito utilizado para definir situações nas quais a


companhia implementa algum tipo de estratégia que a permita conseguir
rápidos retornos.

● Liability: são as responsabilidades legais, obrigações de uma empresa,


como impostos, certificações a serem obtidas e dívidas.

● Markup: é um índice utilizado como ponto inicial para se determinar o


preço de venda de um produto, a partir dos custos para a sua produção.

● Breakdown: é o processo de dividir algum operador em todas as


subpartes que o compõem.

● Breakdown of cost: é a decomposição do valor do custo total em todas as


diferentes formas de custos envolvidas naquela empresa, com seus
respectivos valores, de forma a, se somadas, retornarem os custos totais.

3.2.3. Análise gráfica

A análise gráfica compõe parte das habilidades mais centrais da prova de


business case - afinal, será com base neles que os candidatos irão coletar os
dados e formular os insights para as perguntas. Portanto, dada a sua
importância, é interessante trazer um método capaz de permitir o entendimento
completo das informações. Para isso, vamos dividir a análise em 3 passos:

● Mapear o gráfico

Nessa etapa, é recomendado que o candidato não fique preso aos dados. É
necessário apenas entender o que o gráfico buscará representar, e,
consequentemente, quais insights podem ser formulados a partir dele.

Ao observarmos o gráfico, e, por enquanto sem nos atentarmos aos


números, podemos concluir através da análise dos eixos, que X representa

97
14ª edição do Programa de Preparação - UFRJ Consulting Club Apostila

o transcorrer dos ano e Y o lucro da empresa. Nesse caso, o título também


traz de maneira bem clara o seu objetivo.

● Ler as nuances:

Nessa segunda fase, recomenda-se que o candidato leia as informações


extras, como notas de rodapé, legendas e rótulos, garantindo que nada
passe despercebido.

98
14ª edição do Programa de Preparação - UFRJ Consulting Club Apostila

Nesse caso, a nota de rodapé nos forneceu a taxa de conversão de dólares


para reais, o que pode ser bem útil para calcular as receitas na moeda
brasileira. Além disso, ela também trouxe a definição do que é o CAGR,
algo importante para aqueles que não sabem o significado dessa métrica.
Entretanto, nem sempre as definições desses termos estão presentes nos
gráficos, deixando clara a importância de dominá-los.

Outro dado interessante é a legenda, que nos fornece a informação que a


firma A terá o seu gráfico desenhado em vermelho e a firma B terá o seu
em azul.

Por fim, no canto superior direito, o gráfico ainda nos forneceu


informações sobre o CAGR de cada uma das empresas.

● Ler os dados:

Nessa etapa o candidato deve ler o que realmente os dados mostram,


analisando em qual firma o lucro é maior, ou então se alguma apresenta
prejuízo… Essas observações serão úteis para que, posteriormente, seja
feito o insight necessário para realizar a questão.

99
14ª edição do Programa de Preparação - UFRJ Consulting Club Apostila

Por exemplo, é possível formular um insight sobre a velocidade de


crescimento - afinal a linha de A é mais inclinada que a de B, ou seja, o
lucro de A cresce de forma mais acelerada em comparação ao da empresa
B. Portanto, é possível concluir que, em um período de alguns poucos
anos (não é possível dizer quando), a firma A será mais lucrativa que a B,
caso a tendência se mantenha.

3.2.4. Tipos de gráfico

Após descrevermos um método para a análise gráfica, é interessante conhecer


melhor os tipos de gráfico - afinal, a familiaridade com os mesmos pode agilizar o
entendimento das informações.

● Gráfico de coluna

No gráfico de coluna, as barras geralmente representam a


correspondência de algum valor do eixo das abscissas com o eixo das
ordenadas, compondo de forma geral, um tipo de gráfico de fácil análise.

100
14ª edição do Programa de Preparação - UFRJ Consulting Club Apostila

Nessa representação, o eixo horizontal dispõe da sequência de anos, e, em


cada coluna desses anos, é apresentado o valor referente à quantidade de
consultores da Natura.

Entretanto, há um conjunto de gráficos de colunas um pouco mais


complicados, nos quais a coluna se apresenta de forma segmentada,
indicando que dentro de um mesmo valor de X, há diferentes
correspondências em Y.

Exemplo:

101
14ª edição do Programa de Preparação - UFRJ Consulting Club Apostila

Ao analisarmos este gráfico, torna-se possível interpretar por onde os


consultores da Natura estão distribuídos em um mesmo ano. Além disso,
espera-se que no final de 2012, a empresa atinja a marca de 1,785 milhões
de representantes mundialmente.

● Gráfico de colunas 100% empilhadas

O gráfico de colunas 100% empilhadas possui a mesma ideia que o gráfico


anterior, contudo as informações segmentadas representam uma
porcentagem do total daquela coluna, com isso, todas elas apresentam a
mesma altura e o que as difere é a informação do valor absoluto que vem
em cima da coluna. Dessa forma, as colunas devem ser analisadas
individualmente, pois segmentos do mesmo tamanho em colunas
diferentes não necessariamente apresentam o mesmo valor absoluto.

No gráfico acima, é possível perceber que houve um aumento do market


share do competidor 3, porém, pela diminuição do market size com o
decorrer dos anos, esse aumento não se mostrou absoluto. Além disso, é
possível observar que o market share do competidor 2 se manteve mas,
por outro lado, em valores absolutos a empresa apresentou uma redução
no rendimento.

102
14ª edição do Programa de Preparação - UFRJ Consulting Club Apostila

● Gráfico mekko

Dentro das provas de business case, os gráficos de área são extremamente


comuns, sobretudo no detalhamento comparativo do market size e do
market share de diferentes setores e empresas. Dessa forma, eles se
apoiam na diferença entre as áreas fornecidas para ressaltar as diferenças
entre os valores e definir os competidores nesses segmentos.

Exemplo:

Esse é um gráfico de áreas referente a participação de cada um dos


principais competidores dentro dos subsegmentos do setor de jogos. Para
analisá-lo, é crucial considerar a legenda, capaz de informar a
correspondência das cores e relacioná-las com os valores escritos no
interior da área. Caso o candidato deseje obter o market share dentro do
segmento de “RPG” basta que analise a coluna na abscissa
correspondente. Por fim, caso ainda deseje uma empresa em específico,
basta identificar a sua cor na legenda e localizá-la dentro da coluna de
“RPG”.

● Gráfico de bolhas

O gráfico de bolhas é comum quando se deseja representar 3 variáveis em


uma só imagem. Nesse caso, por exemplo, o eixo dos X representa a renda
de cada família e o eixo dos Y qual o comprometimento da renda com o
financiamento. Por sua vez, o tamanho da bolha é a percepção de risco
sobre o empréstimo realizado.

103
14ª edição do Programa de Preparação - UFRJ Consulting Club Apostila

3.3. Tipos de pergunta


De forma geral, as perguntas de business case podem ser divididas em quatro padrões:
as questões de recommendation, as de fact-based, as de reading facts e as de
root-cause reason. Entretanto, a classificação de cada uma não é primordial para a
resolução de questões, portanto, não se preocupe em decorar os diferentes tipos de
enunciado.

● Recommendation: são questões que pedem para o candidato marcar a


melhor solução ou escolha a ser tomada pela empresa diante de um
cenário hipotético.

Exemplo:

104
14ª edição do Programa de Preparação - UFRJ Consulting Club Apostila

(BCG) What ticket price should Olympian Airways charge to maximize


gross profit per day?

a. U$100

b. U$125

c. U$150

d. None of the above/we lack sufficient information at this stage

Resolução:

Após ler o enunciado e realizar a marcação dos pontos importantes,


sabemos que é demandado que o candidato escolha a opção capaz de
maximizar o lucro bruto da Olympian Airways. Com isso, iniciamos a
análise dos dados:

Cenário A

Receita

Segundo a tabela, serão vendidas 200 passagens pelo preço de U$ 100,


totalizando U$ 20.000. Como são 30 voos por dia, a receita com passagens
é de U$ 600.000.

Além das passagens, ainda há a receita com os snacks, que devem ser
levados em conta. Pela tabela, temos:

Dos 200 passageiros, 50% são do padrão C1, os quais costumam gastar em
média U$ 5 por pessoa.

200 X 50% X 5 = 500 U$

Dos 200 passageiros, 30% são do padrão C2, os quais costumam gastar U$
10 por pessoa,

200 X 30% X 10 = 600 U$

Dos 200 passageiros há ainda 20% do padrão C3, os quais costumam


gastar U$ 15 por pessoa.

200 X 20% X 15 = U$ 600

105
14ª edição do Programa de Preparação - UFRJ Consulting Club Apostila

Somando as 3 segmentações, temos U$ 1.700 em snacks por voo. Como


são 30 viagens por dia, temos 51.000 em snacks.

Somando todas as receitas (voo e snacks), temos U$ 651.000.

Custos

Cada voo custa U$ 15.000. Como são 30 voos diários, tem-se como custo
U$ 450.000 por dia.

Lucro bruto

Receita - custos= 651.000 - 450.00 = 201.000 U$/dia.

Cenário B

Receita

Segundo a tabela, serão vendidos 150 passagens pelo preço de U$ 150,


totalizando U$ 22.250. Como são 20 voos por dia, a receita com passagens
é de 450.000 U$/dia.

Além das passagens, ainda há a receita com os snacks que devem ser
levados em conta. Pela tabela temos:

Dos 150 passageiros, 30% são do padrão C1, os quais costumam gastar em
média U$ 5 por pessoa.

150 X 30% X 5 = 225 U$.

Dos 1500 passageiros, 30% são do padrão C2, os quais costumam gastar
U$ 10 por pessoa.

1500 X 30% X 10 = U$ 450.

Dos 150 passageiros há ainda 40% do padrão C3, os quais costumam


gastar U$ 15 por pessoa.

150 X 40% X 15 = U$ 900.

Somando as 3 segmentações, temos U$ 1.575 em snacks por voo. Como


são 30 viagens por dia, temos 31.500 em snacks.

Somando todas as receitas (voo e snacks), temos U$ 481.500.

106
14ª edição do Programa de Preparação - UFRJ Consulting Club Apostila

Custos

Cada voo custa U$ 15.000. Como são 20 voos diários, tem-se como custo
U$ 300.000 por dia.

Lucro bruto

481.500 - 300.00 = 181.50000 U$/dia.

Não há informações para calcular o lucro bruto para U$ 125, portanto essa
opção é desconsiderada.

Logo, o preço que maximiza o lucro é U$ 100.

Resposta correta: A

● Fact-based conclusion: Nas questões de Fact-based Conclusion, são


entregues alguns fatos e/ou dados, e é perguntado qual das alternativas
seria uma conclusão coerente ao que foi fornecido.

Exemplo:

(BCG) Which of the factors listed below would not affect the gross profit

difference between scenarios A and B?

a. Ticket prices

b. Aircrafts’ seat capacity

c. Fuel costs

d. None of the above

Resolução:

Ao lermos o enunciado, vemos que foi pedido para assinalar o fator que
não influenciaria o lucro bruto da empresa nos cenários A e B. Após
destacar os pontos importantes, podemos iniciar a testagem de cada uma
das alternativas.

a. Ticket prices

Como demonstrado no raciocínio na questão anterior, o preço das


passagens afeta o lucro bruto da companhia

b. Aircrafts’ seat capacity

107
14ª edição do Programa de Preparação - UFRJ Consulting Club Apostila

Caso a aeronave no cenário A apresentasse capacidade de apenas 150


lugares, isso significaria uma redução de 150.00 U$ só na receita referente
às passagens, o que demonstra a sua influência no lucro bruto.

c. Fuel costs

O enunciado forneceu que os custos da viagem são de U$ 15.000 por voo.


Supondo, hipoteticamente, que o querosene componha 10% desse custo -
ou seja, U$ 1.500, caso o seu preço aumentasse em 10%, a sua participação
aumentaria para U$ 1.650 e o custo total do voo iria para U$ 15.150.
Portanto, fica claro que o combustível aéreo é um fator que influencia no
lucro bruto da empresa.

d. None of the above

Portanto, sobra a letra D, que é o gabarito, visto que todas as opções


citadas influenciam o lucro.

Resposta correta: D

● Root-cause reason: são as perguntas que demandam que o candidato


forneça a causa efetiva de algum fenômeno. Assim como nas provas de
GMAT, o comando “if true” é bem comum nos comandos desse tipo de
questão. Portanto, é importante lembrar que não cabe ao candidato
avaliar se as hipóteses apresentadas são verídicas, mas sim qual delas qual
delas provaria o fato se fosse verdadeira.

Exemplo:

108
14ª edição do Programa de Preparação - UFRJ Consulting Club Apostila

(BCG) Based on Doc 1, which of the statements below, if true, could


explain the EBITDA margin decline?

(Note: EBITDA = Revenue - COGS - Operating Expenses (except for


Depreciation and Amortization))

i) Monthly average volume of cement sold on the 1st semester of 2018 was
lower than the monthly average sold on the 1st semester of 2017

ii) Cement price has increased from 2017 to 2018, but average volume of
cement sold per month remained constant, considering only the 1st
semester of 2017

iii) Cia. Daska invested € 10 million on a new heat exchanger in January


2018, which is not operating yet

a. Only I is correct

b. Only III is correct

c. Only II and III are correct

d. Only I and III are correct

109
14ª edição do Programa de Preparação - UFRJ Consulting Club Apostila

Resolução:

Após ler o enunciado e destacar os pontos chaves, é possível começar o


exercício. Primeiramente, o enunciado deixa bem claro para se basear no
documento 1, portanto, faça o uso dessa dica!

i) Monthly average volume of cement sold on the 1st semester of 2018

was lower than the monthly average sold on the 1st semester of 2017

Caso o volume de cimento vendido no primeiro semestre de 2018 seja


menor do que o do primeiro semestre de 2017, esse pode ser um fator que
explica o motivo da redução da margem EBITDA.

Supondo uma manutenção do preço do cimento nos dois períodos, o total


arrecadado pela empresa seria menor. Com isso, em paralelo à redução na
receita, provavelmente, haveria uma redução na margem EBITDA.

Portanto, essa é uma explicação válida.

ii) Cement price has increased from 2017 to 2018, but average volume of
cement sold per month remained constant, considering only the 1st
semester of 2017

Caso o preço do cimento tenha aumentado com a troca de ano e as


vendas se mantido iguais no período, a receita da empresa aumentaria, o
que dificilmente explicaria a redução da margem EBITDA.

Portanto, a alternativa é falsa.

iii) Cia. Daska invested € 10 million on a new heat exchanger in January


2018, which is not operating yet.

No cálculo do EBITDA não são contabilizados gastos com aquisições de


maquinários. Dessa forma, a alternativa não explicaria uma redução da
margem EBITDA.

Resposta correta: A

● Reading facts: são as questões numéricas, onde geralmente é


demandado que o candidato traga o lucro de uma empresa, a sua receita,

110
14ª edição do Programa de Preparação - UFRJ Consulting Club Apostila

entre outros. Portanto, elas apresentam como objetivo analisar se, de fato,
o candidato entendeu os dados fornecidos.

Exemplo:

(BCG) Based on the market research, what is the average ticket price
Olympian Airways should charge for a flight from London to Paris?

a. €102

b. €136

c. €125

d. €116

Resolução:

Após ler o enunciado e realizar as marcações dos pontos-chave, podemos


iniciar a análise dos dados e do gráfico.

Inicialmente, a questão fornece uma descrição do cenário de alteração do


mercado aéreo global. Diante disso, a empresa precisa readaptar o preço
das suas passagens para o novo cenário. Para calcular o novo valor, é
necessário relacionar a intenção de compra de cada pessoa diante de um
preço. O preço médio ideal será o resultado da média ponderada, onde os
preços são os valores e a intenção de compra os fatores de ponderação.

111
14ª edição do Programa de Preparação - UFRJ Consulting Club Apostila

Portanto:

Preço ideal = (200 x 1.110 + 50 X 500 + 100 X 1500 + 150 X 1900)/5.000.

O resultado dessa conta é 136 €/passagem.

Resposta correta: B

3.4. Exercícios
Por fim, seguem alguns exercícios com gabarito. Tente fazê-los como se fosse uma
prova, cronometrando tempo e sem consultas a materiais externos. Após a sequência
de exercícios estão os gabaritos.

3.4.1. WFGLA Case

Way Forward (WF) is a nonprofit organization that consists of more than 50


local offices in the United Kingdom. Way Forward Greater London (WFGLA) is
one of these local offices, based in the Greater London Area, a metropolitan
area surrounding the city of London. Typically, the local offices work together
with private and social sector organizations to pool efforts in fundraising
campaigns. These campaigns typically address pressing community issues,
usually around education, income or health. WF first seeks to educate the
population about these campaigns, then solicits donations.

Currently, there is an economic downturn in the United Kingdom. This presents


a challenge for WFGLA, because donations are decreasing when community
need is at its highest. The President of WFGLA has reached out to McKinsey to
ask for support. He tells the team: “I need your help on improving our campaign
effectiveness, which we define as the number of pounds donated per pound
spent on the campaign. We really need to focus on increasing donations in
these times!”

A campaign is usually organized by a group of people called a “campaign


cabinet”. This group includes WFGLA staff, as well as volunteers from the
general public; the actual campaign work is conducted by volunteers.

112
14ª edição do Programa de Preparação - UFRJ Consulting Club Apostila

1) Which of the following pieces of information would be LEAST helpful in


better understanding the current WFGLA situation?

a) Total amount of donations, in British pounds, collected by other WF


offices across the U.K.

b) Comparison of donations made to WFGLA by one-time donors versus


regular donors over the past 5 years.

c) Market research on the public awareness generated by WFGLA


campaigns over the past 5 years.

d) Comparison of campaign effectiveness with other local WF offices.

The team gathers more data on WFGLA and on other WF offices. The
following exhibit shows the total donations and total campaign costs, in
million British pounds (£), for various WF offices last year.

2) How should Regions A to E in Exhibit 5 be ranked according to their


campaign effectiveness from highest to lowest?

a) C, E, D, B, A

b) C, A, D, E, B

c) C, E, D, A, B

d) A, B, D, E, C

113
14ª edição do Programa de Preparação - UFRJ Consulting Club Apostila

3) Assuming WFGLA could reach the highest campaign effectiveness of


all other regions on Exhibit 5, by what percentage could their current
campaign effectiveness be increased?

a) 196%

b) 235%

c) 296%

d) 335%

3.4.2. Challenge Airlines Case

Year Challenge Airlines' total sales ($M) Net profits ($M)

2008 1650 297

2009 1815 309

2010 1997 329

2011 2196 351

2012 2416 362

4) Based on the analysis of Challenge Airlines situation, what is correct to


say?

a) Net profits, as a percentage of sales, have remained steady between


2008 and 2012.

b) Considering that in 2013 the revenue and net profit will follow the
CAGR, 2013’s net margin would be 15.3%.

c) Challenge Airlines’ total sales showed the greatest growth in 2010.

d) Challenge Airlines’ total sales are increasing approximately 10% per


year on average since 2008. However, net profit margins (net profit /

114
14ª edição do Programa de Preparação - UFRJ Consulting Club Apostila

sales) have decreased.

Net profits, as a percentage of sales, were the same in 2008 and 2012.

Break-down of Challenge Airlines Total


Sales ($M)
Year

Domestic International

2008 371 1279

2012 1357 1059

Type RASK CASK

Domestic $0,18 $0,16

International $0,10 $0,08

5) What can be concluded from Challenge Airlines’ breakdown of sales,


RASK and CASK?

ASK: Available seat kilometers (ASK = number of available seats X


distance flown)

RASK: Revenue (sales) per available seat kilometer (e.g., Domestic


RASK = Domestic Sales/Domestic ASK)

CASK: Cost per available seat kilometer (e.g., Domestic CASK =


Domestic Costs/Domestic ASK)

Note: There wasn’t any perceived growth in RASK and CASK between
2008 and 2012.

a) The increase in Domestic sales as a percentage of total sales

115
14ª edição do Programa de Preparação - UFRJ Consulting Club Apostila

contributes to the change in profit margins from 2008 to 2012.

b) Challenge Airlines’ total sales in domestic flights have increased by


over 40% CAGR from 2008 to 2012.

c) Challenge Airlines’ available seat kilometers (ASK) increased


approximately 27% from 2008 to 2012.

d) Domestic sales increased approximately 265% from 2008 to 2012, while


International sales decreased 5% in the same period.

e) Since the difference in RASK and CASK is the same in Domestic and
International flights, profit margins were not affected by the increase in
Domestic sales.

Airline RMS

Challenge Airlines 2

Cheap Air 0,5

Blue Sky 0,33

Good Flight 0,2

South Airlines 0,1

6) What is Blue Sky’s market share, considering its RMS?

RMS: relative market share.

If company is market leader: RMS = company’s market share/second


player’s market share.

If company isn’t market leader: RMS = company’s market share/leader’s


market share.

a) 9%

b) 15%

116
14ª edição do Programa de Preparação - UFRJ Consulting Club Apostila

c) 23%

d) 33%

e) 47%

7) Cheap Air is considering acquiring South Airlines. Should that happen,


what would Challenge Airline’s expected RMS approximately be?

a) 0.33

b) 0.5

c) 0.6

d) 1.67

e) 2

CASK Break-down Challenge Airlines' ($ Cheap Air's ($


cents) cents)

Food Services 0,3 0,0

Landing 0,6 0,5

Sales & Marketing 1,6 0,4

AC Fuel 1,4 1,1

Salaries & Benefits 3,8 2,2

Other 3,6 1,8

8) Benchmarking efforts were made to show the cost breakdown


comparison between Challenge Airlines and Cheap Air. What we can
conclude about it?

a) Cheap Air’s cost position demonstrates why it has been the most

117
14ª edição do Programa de Preparação - UFRJ Consulting Club Apostila

profitable in the industry.

b) The investment in Sales & Marketing made by Challenge Airlines is the


reason why it is the market leader.

c) Challenge Airlines offers the best compensation plan in the industry.

d) Challenge Airlines has a better customer satisfaction than Cheap Air.

e) Cheap Air has a better cost position than Challenge Airlines even
though it is a smaller company.

3.4.3. Imagine Learning Case

Imagine Learning is an English learning software provider for children. All the
instructions, texts, and audio are translated into many different languages and
inserted into the software.

A market study compared the cost of the Spanish product line versus that of
Korean.

We propose an idea to self-translate Korean products by hiring new Korean


translators, replacing both the Translation Company and the programmers.
Assume that the average wage for programmers and translators are $25 and
$15 per hour, respectively.

Market Study

Total Cost Cost from Translation Nº of defect Cost of fixing


($US) Company ($US) found Defect* ($US)

Korean
Line
60,00 10,000 230 45,000

Spanish
Line
25,000 15,000 17 2,000

*by programmers

118
14ª edição do Programa de Preparação - UFRJ Consulting Club Apostila

9) What is the maximum amount of time that translators have to finish


translating to make it worth it to switch to our idea?

a) 3650 hours

b) 3500 hours

c) 4000 hours

d) 3000 hours

3.4.4. Basketball League Case

Our client is a professional basketball league in Germany. The league operates


on a sporting business franchising model. Each team in the league has to pay
an annual fee to the league and will manage itself by hiring players, selling
tickets, managing administration costs…

Some direct and indirect competitors of the Client League are:

- US professional basketball league (NBA)

- Euro Basketball league (EBL)

- Country’s Soccer league (CSL)

The next chart measures revenue and its components on the average-per-team
basis.

119
14ª edição do Programa de Preparação - UFRJ Consulting Club
Apostila

10) If the Client League can manage its Other Costs as well as the NBA
can, given that Players’ Salary stays constant (measured by the
per-dollar-of-revenue- generated basis), what would their Profit be?

a) $3 million

b) $5 million

c) $7 million

d) $9 million

3.4.5. Comfort Taxi Case

11) Comfort Taxi is one of the biggest taxi companies in Singapore.


Following are some information on this pricing structure:

● The first 1 km or less: $ 3.50

● Every 400 m thereafter or less up to the 11th km (10 km after the


first km): $ 0.25

● Every 500 m thereafter or less after 11 km: $ 0.25

● Peak Period Surcharge: 25% of all cost between 6pm-midnight

● If a taxi ride is continuous through non-peak and peak hours, Peak


Period Surcharge only apply partially to cost incurred within the
Peak time frame

Suppose Singapore road speed on peak hours is 30 km/h and 50 km/h for
non-peak hours. A rider has $ 23.5 left to travel a distance of 30 km after
work.

Which of the following is the minimum minutes the rider has to leave
before 6 pm to be certain that he can cover the distance within the left
budget?

a) 0 minutes

b) 15 minutes

c) 30 minutes

120
14ª edição do Programa de Preparação - UFRJ Consulting Club
Apostila

d) 45 minutes

3.4.6. TravelCo Case

12) It is possible to segment the market in three customer groups. Travel


operators offer product lines for all segments. After analyzing the market
segmentation and the product lines offered by the main players in the
market, which is the best alternative?

a) Flamingo tours is a good acquisition option due to its strong position in


the promotional packages market.

b) TravelCo should merge with flamingo in order to improve its presence


in the luxury segment.

c) Super holidays is a good acquisition option due to its strong position in


the mid-price market.

d) TravelCo should merge with the Cool travel to increase its presence in
the economic segment.

121
14ª edição do Programa de Preparação - UFRJ Consulting Club
Apostila

13) Based on the estimate of TravelCo revenues and margins, what is


correct to say?

a) In terms of %, gross margins were the same in Y1 an Y3

b) Gross margins decreased 2% basis points between Y1 and Y2, mainly


due to lower penetration of domestic long-haul packages

c) TravelCo’s total sales are increasing approximately 5% per year on


average between Y1 and Y5. However, total gross margins have not
changed.

d) TravelCo reached its highest gross margins (in %) in Y1.

122
14ª edição do Programa de Preparação - UFRJ Consulting Club
Apostila

14) What is possible to conclude about TravelCo’s product portfolio?

a) TravelCo should get out of cruises and focus its efforts on more
growth-promising products such as domestic packages and bus
packages.

b) TravelCo should get out of Air Tickets and Bus Packages, due to its low
market share in these segments.

c) Long Haul packages are the best investment option since it is where
TravelCo has its highest market share.

d) TravelCo should get out of Bus Packages and focus efforts on gaining
market share on more growth-promising products such as Cruises and
Long-Haul packages.

123
14ª edição do Programa de Preparação - UFRJ Consulting Club
Apostila

3.4.7. Kosher Franks

15) Which of the following statements, if true, would NOT help support the
assumption that most of Kosher Franks’ hot dog consumers are Jewish?

a) The trends in Kosher Franks’ hot dog sales in the last five years are very
similar to those of other kosher food products

b) All of Kosher Franks’ major grocery chain customers have shopping


aisles dedicated to kosher foods

c) In a recent consumer survey, the awareness of the Kosher Franks’ label


was three times as high among Jewish respondents than among other
respondents

d) All stores who stock Kosher Franks’ hot dogs are in areas with above
average concentrations of Jewish households.

16) Which of the following values is the best estimate of Kosher Franks’
revenue in Year 4 under Scenario C in Exhibit 1?

124
14ª edição do Programa de Preparação - UFRJ Consulting Club
Apostila

a) $441M

b) $495M

c) $549M

d) $603M

3.4.8. Expensive Oil

17) What is, approximately, the average gross margin?

a) 5,9%

b) 6,3%

c) 6,9%

d) None of above/We lack sufficient information at this stage.

125
14ª edição do Programa de Preparação - UFRJ Consulting Club
Apostila

18) What is the price per litre the company should charge to maximize
profits on gasoline sales?

a) € 1,50

b) € 1,60

c) € 1,70

d) € 1,80

3.5. Gabaritos

3.5.1. WFGLA Case

1) Resposta correta: A

Resolução:

● A pergunta trata de identificar informações que melhor explicam a


situação atual do escritório. É, portanto, uma questão de Root-cause
Reason;
● Buscaremos informações que expliquem o fato de o escritório estar
arrecadando menos doações ultimamente;

126
14ª edição do Programa de Preparação - UFRJ Consulting Club Apostila

● A letra A nos fornece quanto de doação, em libras, cada um dos outros


escritórios do Reino Unido arrecada atualmente. Como cada região tem
seu tamanho, sua realidade e atende a um público distinto, conhecer
esses valores absolutos não é de grande ajuda para entender o cenário do
WFGLA.

● A pergunta B fornece dados sobre dois diferentes perfis de doadores, o


que permite determinar em qual deles focar. Já as alternativas C e D
trazem informações importantes sobre a eficácia das campanhas atuais,
exatamente o instrumento de captação com que o presidente mais se
preocupa.

2) Resposta correta: A

Resolução:

● A questão demanda um ranqueamento baseado no indicador de eficácia


da campanha, cuja fórmula foi dada no texto inicial. Como precisamos
apenas ler as informações do gráfico e aplicar a fórmula do indicador,
trata-se de um exercício de Reading Facts;

● A eficácia é dada pela razão entre o volume de doações captadas e o custo


da campanha. Calculando para as regiões pedidas, temos, já em ordem, C

= = 20,5, E = = 13,5, D = = 11,2, B = = 8,3 e A = = 8,1.

3) Resposta correta: B

Resolução:

● Assim como na anterior, basta computarmos para as regiões o indicador


de eficácia de campanha. Temos, portanto, mais uma vez uma questão de
Reading Facts;

● Para não perder tempo, eliminaremos as regiões mais óbvias. Vemos que
G tem o maior volume de doações, com custo maior apenas que os das
regiões E e H. Assim, já descartamos todas as outras regiões;

● E e H tem uma arrecadação menor que a metade da apresentada por G,

127
14ª edição do Programa de Preparação - UFRJ Consulting Club Apostila

enquanto seus custos são maiores que a metade dos custos de G, o que
confirma essa região como a de maior eficácia;

● Calcularemos as eficácias das regiões G e WFGLA. G = 43,8/1,9 = 23,1. Para


calcular quanto significa em termos de porcentagem de WFGLA, a
diferença entre a eficácia de G e de WFGLA, fazemos (G - WFGLA)/WFGLA
= (23,1 - 6,9)/6,9=2,35. Vemos, portanto, que o aumento de eficácia seria de
235%.

3.5.2. Challenge Airlines Case

4) Resposta correta: D

Resolução:

● A questão pede para fornecermos a resposta correta de acordo com os


dados existentes na tabela. Ela se trata, portanto, de uma questão de
Reading Facts;

● A alternativa A fala sobre lucro como percentual em função da receita, ou


seja, a margem de lucro, que abordamos no estudo de conceitos mais
recorrentes. Assim, para descobrir se a lucratividade foi mantida durantes
os anos de 2008 a 2012, devemos calcular a margem de lucro de cada ano:

2008: = 0.18

2009: = 0.17

2010: = 0.16

2011: = 0.15

2012: = 0.14

Portanto, podemos ver que os lucros não foram mantidos durante esse
tempo, mas sim, diminuíram. Logo, a afirmação da A é falsa;

● A letra B fala de CAGR, um conceito mais complexo de taxa de


crescimento que sequer foi explicado no enunciado. Já é um indício de

128
14ª edição do Programa de Preparação - UFRJ Consulting Club Apostila

que o mais prudente seria resolver solucionando as outras alternativas


antes. Faremos aqui a conta, no entanto:

CAGR = ( )–1

CAGR receita = 10%

CAGR lucro = 5%

Receita 2013 = 2658

Lucro 2013 = 380

Margem 2013 = 14.3%

● Na opção C, devemos calcular o crescimento da receita de um ano em


relação ao ano anterior, para então compará-los. Para agilizar as contas,
arredondaremos as unidades das receitas, que, em ordem, serão 1650, 1815,
1995, 2195 e 2415. Também arredondaremos antes das divisões, quando
necessário. Fazendo isso para todos os anos, teremos:

Vemos que a receita aumenta aproximadamente, portanto, 10% todos os


anos. Dessa forma, 2010 não apresentou maior crescimento que os outros
anos;

● Na alternativa D, utilizamos as contas já feitas nas letras A e C. Vimos que


de fato as receitas tiveram crescimento constante, enquanto a margem de
lucro decresceu ao longo dos anos. Essa opção é, portanto, a correta;

● Na letra E, lembramos as contas da alternativa A, que mostram que a


margem de 2012 é menor que a de 2008. Concluímos assim que a letra E
está incorreta.

129
14ª edição do Programa de Preparação - UFRJ Consulting Club Apostila

5) Resposta correta: A

Resolução:

● Mais uma vez, devemos analisar as informações das tabelas para


responder a pergunta. Temos, portanto, uma questão de Reading Facts;

● A opção A pergunta sobre o efeito do aumento de voos domésticos na


margem de lucro total da empresa. Vimos na questão passada, em sua
letra A, que a margem de lucro diminuiu com o passar dos anos. Para que
essa redução tenha sido causada pelo aumento de voos domésticos, eles
devem apresentar uma margem de lucro menor do que os voos
internacionais. Temos informações por ASK, então calcularemos a margem
de lucro como a razão entre lucro por ASK e receita por ASK. Como lucro é
receita menos custo, temos:

Margem de lucro por ASK:

Doméstico: = 11,1%

Internacional: = 20%

Assim, confirmamos que a margem de lucro dos voos domésticos é menor


e a opção está correta;

● Na letra B, mais uma vez encontramos o CAGR. Assim como na questão


anterior, na prova para valer deveríamos tentar evitar o cálculo dessa
alternativa. Faremos a conta, no entanto, para fins didáticos. Temos:

𝑉𝑎𝑙𝑜𝑟 𝑓𝑖𝑛𝑎𝑙 1
CAGR = 𝑉𝑎𝑙𝑜𝑟 𝑖𝑛𝑖𝑐𝑖𝑎𝑙
× 𝑛º 𝑑𝑒 𝑎𝑙𝑢𝑛𝑜𝑠
–1

CAGR𝑠𝑎𝑙𝑒𝑠 = – 1 = 38,2%

A opção está, assim, incorreta;

● A opção C pede o valor do ASK, que pode ser calculado pela definição
dada do RASK ou do CASK. Como possuem RASKs e CASKs diferentes,
calculamos os ASKs de voos domésticos e internacionais separados para
depois somá-los e obter o total. Fazendo as contas, teremos:

130
14ª edição do Programa de Preparação - UFRJ Consulting Club Apostila

𝑅𝐴𝑆𝐾 = ∴ 𝐴𝑆𝐾 =

Em 2008: 𝐴𝑆𝐾𝑑𝑜𝑚 = = 2061 e 𝐴𝑆𝐾𝑖𝑛𝑡 = = 12790 ∴ 𝐴𝑆𝐾𝑡𝑜𝑡𝑎𝑙 =


14851

Em 2012: 𝐴𝑆𝐾𝑑𝑜𝑚 = = 7539 e 𝐴𝑆𝐾𝑖𝑛𝑡 = = 10590 ∴ 𝐴𝑆𝐾𝑡𝑜𝑡𝑎𝑙 =


18129

Crescimento: = 22%

Como o crescimento foi de 22%, a alternativa C é falsa;

● Na alternativa D, basta calcularmos os valores mencionados. Para ganhar


tempo, olhamos rapidamente a variação das vendas de voos
internacionais, que intuitivamente parece ter decrescido bem mais que
5%. Ao calculá-la, veremos que seu verdadeiro valor é 17%. Mesmo que, se
fizermos os cálculos veremos, a taxa de crescimento dos voos domésticos
está certa, o decrescimento de voos internacionais está errado e a
afirmação está incorreta;

● Na letra E, precisamos analisar a definição de margem de lucro em função


de RASK e CASK, que já fizemos na opção A. Lá, vemos que de fato o
numerador não se altera para as margens de voos domésticos e
internacionais. O denominador, no entanto, será diferente para cada um
dos dois tipos, o que resulta em valores distintos de margem de lucro. A
opção é, assim, falsa.

6) Resposta correta: B

Resolução:

● Sabemos que a soma de todos os market shares tem que ser igual a 100%,
ou seja, a 1, como descrito abaixo:

𝑀𝑆𝑐ℎ𝑎𝑙𝑙𝑒𝑛𝑔𝑒 + 𝑀𝑆𝑐ℎ𝑒𝑎𝑝 + 𝑀𝑆𝑏𝑙𝑢𝑒 + 𝑀𝑆𝑔𝑜𝑜𝑑 + 𝑀𝑆𝑠𝑜𝑢𝑡ℎ = 1

● Temos, no entanto, apenas os dados de RMS. Para escrever a função em

131
14ª edição do Programa de Preparação - UFRJ Consulting Club Apostila

termos desses valores, dividiremos todos pelo market share do líder:

+ + + + =

1 + RMScheap + RMSblue + RMSgood + RMSsouth =

● Substituindo os valores da tabela, encontraremos o market share do líder:

1 + 0,5 + 0,33 + 0,2 + 0,1 =

MSchallenge = 0,47

● Podemos agora, a partir do RMS da Blue Sky e do MS da Challenge


Airlines, calcular o MS da Blue Sky:

𝑅𝑀𝑆𝑏𝑙𝑢𝑒 = ∴ 𝑀𝑆𝑏𝑙𝑢𝑒 = 0, 33×0, 47≈15%

7) Resposta correta: D

Resolução:

● Esta questão se assemelha à anterior, envolvendo apenas cálculos em


cima de market share e RMS. Utilizaremos ainda alguns valores já
encontrados na questão anterior. Iremos calcular o MS da South Airlines e
o da Cheap Air para somá-los e, então, calcular o novo RMS da Challenge
Airlines:

𝑅𝑀𝑆south = ∴ 𝑀𝑆south = 0, 1×0, 47≈4, 7%

𝑅𝑀𝑆cheap = ∴ 𝑀𝑆cheap = 0, 5×0, 47≈23, 5%

𝑀𝑆south + 𝑀𝑆cheap = 7 + 23, 5 = 28, 2%

● Como a nova empresa South+Cheap assumiria a posição de segundo


maior market share, teríamos uma alteração no RMS da Challenge
Airlines:

𝑅𝑀𝑆cℎ𝑎𝑙𝑙𝑒𝑛𝑔𝑒 = ∴ 𝑅𝑀𝑆challenge = ≈ 1,67

132
14ª edição do Programa de Preparação - UFRJ Consulting Club Apostila

8) Resposta correta: E

Resolução:

● Nessa questão, precisamos olhar para cada uma das alternativas e avaliar
qual delas nos mostra uma conclusão que pode ser tirada com base nas
informações mostradas na tabela. Temos, portanto, uma questão de
Fact-based Conclusion;

● Como não temos informações sobre a receita da companhia Cheap Air,


nada podemos concluir com certeza a respeito de seus lucros. Por isso, a
opção A é incorreta. Apesar de os investimentos em Sales & Marketing da
Challenge Airlines serem maiores do que os da Cheap Air, nada nos prova
que investir nesse setor é uma garantia de maior sucesso para a empresa.
Portanto, a alternativa B também está errada. Descartamos a letra C ao
notar que a tabela só compara duas empresas, e não a indústria toda. Por
mais que a compensação (salário e benefícios) seja mais alta na Challenge
Airlines que na Cheap Air, não sabemos se há outras empresas no setor
que oferecem compensações ainda melhores aos seus empregados.
Como a tabela não fornece indicadores de satisfação dos clientes, a
conclusão da D não tem base;

● A partir dos dados sobre os custos das companhias, podemos afirmar que
os gastos da Cheap Air são menores do que os da Challenge Airlines. Além
disso, da tabela anterior, sabemos que o MS da Challenge é
aproximadamente quatro vezes maior que o da Cheap Air, o que
comprova que a Cheap é uma empresa menor do que a Challenge. A letra
E, portanto, apresenta uma conclusão embasada nos fatos fornecidos e é a
opção correta.

3.5.3. Imagine Learning Case

9) Resposta correta: A

Resolução:

133
14ª edição do Programa de Preparação - UFRJ Consulting Club Apostila

● Temos mais uma questão de Reading Facts, pois só precisamos fazer


contas a partir dos dados fornecidos;

● A nova proposta substituiria a empresa de tradução e os programmers,


eliminando seus respectivos custos. É essencial notar que todo o custo dos
programadores é representado na tabela como cost of fixing defect e não
precisamos, por isso, da informação do salário de $25 para programadores.
Para que a troca valha a pena, o gasto com o salário dos novos tradutores
tem que ser menor do que a economia com o corte dos custos citados
anteriormente. O número máximo de horas que os novos tradutores
podem trabalhar, portanto, deve ser aquele que gera um pagamento de
salário igual aos antigos custos. Sabendo que o salário por hora dos novos
tradutores é $15, calculamos:

A resposta é 3650 horas, quantidade apontada na alternativa A.

3.5.4. Basketball League Case

10) Resposta correta: C

Resolução:

● A questão nos fala que o cliente passará a gerenciar seus custos tão bem
como a NBA. Para saber como ficará a situação dos seus lucros,
precisamos comparar a relação entre receita e custos dessas duas ligas;

● A NBA tem hoje os mesmos other costs que o cliente, em termos


absolutos. No entanto, a receita da NBA, $65 milhões, é praticamente o
dobro da receita do cliente, $31 milhões. Se o cliente passar a ser tão
eficiente quanto a NBA em sua gestão de custos, espera-se que seu custo
passe a ter o mesmo valor relativo à receita que a NBA. Como a receita do

134
14ª edição do Programa de Preparação - UFRJ Consulting Club Apostila

cliente é metade da receita da NBA, seus custos deverão também ser a


metade, ou seja, $5 milhões;

● Essa alteração, já que todas as outras variáveis são mantidas constantes de


acordo com o enunciado, causa uma economia de $5 milhões, que se
tornarão lucro. Somando essa nova quantia ao valor antigo do lucro, de $2
milhões, o cliente passará a ter o lucro de $7 milhões e a resposta correta é,
portanto, a letra C.

3.5.5. Comfort Taxi Case

11) Resposta correta: B

Resolução:

● Como a questão pede o tempo mínimo, é interessante começar testando


as alternativas menores primeiro, uma vez que achando a que se adequa,
teremos já resolvido a questão;

● Começando com a alternativa A, de 0 minutos. Isso significa que toda a


viagem receberá 25% a mais por estar no horário de pico. Portanto, a soma
dos gastos da viagem será:

1º km – $3,50

𝐷𝑜 1 𝑎𝑜 11 𝑘𝑚 – $6,25

𝐷𝑜 11 𝑎𝑜 30 𝑘𝑚 − $9,50

𝑇𝑜𝑡𝑎𝑙 𝑎𝑛𝑡𝑒𝑠 𝑑𝑎 𝑡𝑎𝑥𝑎: $19,25

𝑇𝑜𝑡𝑎𝑙 𝑑𝑒𝑝𝑜𝑖𝑠 𝑑𝑎 𝑡𝑎𝑥𝑎 $24,06

Então a alternativa está incorreta;

● Partindo 15 minutos antes, o táxi percorrerá 12,5 km antes de entrar na


tarifa de pico. Logo, a soma dos gastos será:

1º 𝑘𝑚 − $3,50

𝐷𝑜 1 𝑎𝑜 11 𝑘𝑚 − $6,25

𝐷𝑜 11 𝑎𝑜 12,5 𝑘𝑚 − $0,75

135
14ª edição do Programa de Preparação - UFRJ Consulting Club Apostila

𝐷𝑜 12,5 𝑎𝑜 30 𝑘𝑚, 𝑠𝑒𝑚 𝑡𝑎𝑥𝑎 − $8,75

𝐷𝑜 12,5 𝑎𝑜 30 𝑘𝑚, 𝑐𝑜𝑚 𝑡𝑎𝑥𝑎 − $10,94

𝑇𝑜𝑡𝑎𝑙: $21,44

Como o valor está abaixo de $23,50, a opção B é a correta.

3.5.6. TravelCo Case

12) Resposta correta: B

Resolução:

● Do primeiro gráfico (da esquerda) temos que:

○ O que está sendo retratado é a disposição dos shares de mercado


de cada um dos competidores baseado em três nichos de cliente:
Economic (10% de lucratividade), Mid-Price (12% de lucratividade),
Luxury (16% de lucratividade).
○ A TravelCo é focada no Mid-Price (com 30% do Share) e sua grande
defasagem está no nicho mais lucrativo, já que ela possui somente
10% do share na Luxury.

● Do segundo gráfico (da direita) temos que:

○ O nicho com maior projeção de crescimento é o nicho Luxury


(justamente onde a TravelCo está mais fraca).
○ O nicho Economic não apresenta boas projeções de crescimento ao
se comparadas com as demais (além de apresentar uma
lucratividade baixa).
○ O nicho Mid-Price é um meio termo, mas ainda não alcança a força
do nicho Luxury.
● Analisando as alternativas, temos que:
○ A alternativa I não é condizente já que a Flamingo Tours é mais
forte no nicho Luxury e não no nicho Economic.
○ A alternativa II é condizente já que a Flamingo Tours é bem forte no
nicho Luxury e fraca no nicho Mid-Price (logo uma fusão seria muito
benéfica para ambas).

136
14ª edição do Programa de Preparação - UFRJ Consulting Club Apostila

○ A alternativa III não é condizente já que a Super Holidays não é


forte no nicho Mid-Price (inclusive, a TravelCo a supera).
○ A alternativa IV não é a melhor escolha em termos de business já
que o nicho Economic não apresenta uma taxa de crescimento tão
alta ao se comparado ao nicho Luxury.
○ Ou seja, ter 40% do nicho Luxury (pela fusão com a Flamingo Tours)
é melhor do que ter 40% do nicho Economic (pela fusão com a Cool
Travel) em termos de lucratividade.

13) Resposta correta: D

Resolução:

● No gráfico da esquerda percebemos a divisão (breakdown) da receita da


TravelCo de acordo com o tipo de serviço. No topo de cada uma das barras
temos a quantidade em milhões de receita.

● No gráfico da direita temos a Gross Margin (Margem Bruta) do intervalo do


ano 1 ao ano 5 de cada serviço.

● A margem bruta é o lucro operacional da empresa:

(Receita Operacional - Custo Operacional)/Receita

● Para resolver a questão, devemos assumir que a tabela de Gross Margin é


correta para todos os anos. O que significa que no ano 1 (Y1), por exemplo,
para o Domestic, a TravelCo ganhou 47% * 50% * 255, ou que, para o
LonGhaul, ela ganhou 16% * 70% * 255.

● Como precisamos unicamente da margem total, não é necessário


multiplicar pela receita total daquele ano.

● Então a margem total de um ano será o somatório das margens de cada


produto multiplicado pela % da receita do respectivo produto no
respectivo ano. Assim, ficamos com:

137
14ª edição do Programa de Preparação - UFRJ Consulting Club Apostila

● Dessa forma, inferimos que o ano 1 teve a maior margem de todos os


anos.

14) Resposta correta: D

Resolução:

● Como a questão nos pergunta “o que é possível concluir” sobre algum


tema, já temos a certeza de que se trata de uma Fact-Based Conclusion.
Lembrando que as alternativas de uma questão desse tipo podem ser

○ Comprovadamente correta: as informações dão certeza quanto à


veracidade de uma conclusão.
○ Comprovadamente incorreta: as informações dão certeza quanto à
falsidade de uma conclusão.
○ Não comprovada: as informações não conseguem dar certeza
sobre a falsidade ou a veracidade da conclusão.
● Passando por cada uma das alternativas:
○ A primeira alternativa é comprovadamente falsa, uma vez que os
cruzeiros têm uma expectativa maior de crescimento e as margens
são maiores do que ônibus e domestic packages.
○ A segunda alternativa não é comprovada, uma vez que não temos
mais informações sobre competidores (o que está sendo analisado
é o market share relativo, não o market share em si ou a
concentração do mercado).
○ A terceira alternativa novamente não é comprovada, já que
estamos tratando de market share relativo.
○ A quarta alternativa é comprovadamente verdadeira já que bus

138
14ª edição do Programa de Preparação - UFRJ Consulting Club Apostila

packages não se trata de um mercado com boa previsão de


crescimento, ao passo que Longhaul e Cruises são os mais
promissores.

3.5.7. Kosher Franks

15) Resposta correta: B

Resolução:

● O “If True” do enunciado, nos revela que se trata de uma questão de


hipóteses, isto é, uma questão de Root-Cause Reason.

● Atenção que devemos nos atentar à frase que não condiz com a assertiva
de que “a maioria dos clientes da Kosher Franks são judeus”. Você não
precisa concordar com a premissa do autor!

● Passando pelas alternativas:

○ A primeira alternativa nos indica que, a Kosher Franks é voltada para


o consumidor judeu e quem come os cachorros quentes tem o
mesmo padrão de consumo do que quem compra as outras
comidas da Kosher Franks. Logo, ela suporta a hipótese, já que, por
indução, quem consome os cachorros quentes segue a tendência
de consumo do público judeu.

○ A segunda alternativa não traz nenhum estudo de padrões ou de


inferência de dados quanto ao perfil dos consumidores.

○ A terceira alternativa nos faz crer que a Kosher Franks é altamente


voltada para o consumidor judeu já que foi feita uma pesquisa para
confirmar tal fato.

○ A quarta alternativa suporta a hipótese justamente por trazer dados


que reforcem a ideia de que quanto mais judeus há numa área mais
empresas estocam e vendem os cachorros quentes da Kosher
Franks.

139
14ª edição do Programa de Preparação - UFRJ Consulting Club Apostila

16) Resposta correta: D

Resolução:

● A primeira tabela nos traz informações sobre a receita da empresa


destrinchada por cada um dos produtos vendidos pela empresa.

● O gráfico da direita estipula quatro cenários para o crescimento da


empresa, em receita, nos próximos cinco anos.

● Percebemos pelo gráfico que, pelo cenário C, do ano 0 (hoje) até o ano 4
(onde ele quer no enunciado) há um crescimento de 100% da receita para
exatamente 120% da receita. Isso nos dá uma projeção de crescimento de
20%.

● Multiplicando a receita total da empresa (somando todos os valores da


tabela) de 502,4 por 1,2 (20% de aumento) teremos um resultado final de
603M.

3.5.8. Expensive Oil

17) Resposta correta: B

Resolução:

● O primeiro texto nos diz que a margem de lucro da empresa é de €0,1 por
litro de combustível.

● O segundo texto nos diz que as vendas estão distribuídas da seguinte


forma:

○ 30% dos clientes compram combustível no intervalo de preço de


€1.70 por litro.

○ 40% dos clientes compram combustível no intervalo de preço de


€1.60 por litro.

○ 30% dos clientes compram combustível no intervalo de preço de


€1.50 por litro.

○ Todos consideram a margem de €0,1 por litro de combustível.

● Como ele quer a margem bruta média, devemos:

140
14ª edição do Programa de Preparação - UFRJ Consulting Club Apostila

○ Encontrar o preço médio fazendo (30%*1,7)+(40%*1,6)+(30%*1,5) = 1,6.


Isso se dá porque o preço médio é uma média ponderada dos
clientes vezes a média de preços que eles pagam.

○ Encontrar o percentual por que a margem representa dentro do


preço médio, calculado por 0,1/1,6 = 0,0625.

○ Esse valor é o mais próximo de 6,3%, e por isso a resposta é a letra C.

18) Resposta correta: C

Resolução:

● Pelo gráfico, podemos pegar algumas informações:

○ Temos quatro possibilidades (A,B,C,D) para a gasolina que nos dão


diferentes valores para o preço e para o consumo de gasolina por
carro por semana.

○ O custo da gasolina de um carro se mantém constante


independente da possibilidade (A,B,C,D) num valor de €1,50

● O cálculo que devemos fazer é o de (Preço-Custo)*Litros

○ Como o custo é fixamente igual a 1,50, as únicas variáveis que


teremos que colher do gráfico são o preço e os litros

○ Temos os seguintes pares ordenados (Preço;Litros)

■ (1, 5; 150)

● (1, 5 − 1, 5)×150 = 0

■ (1, 6; 135)

● (1, 6 − 1, 5)×135 = 13, 5

■ (1, 7; 110)

● (1, 7 − 1, 5)×110 = 22

Maior valor de lucro, e, portanto, a resposta:

■ (1, 8; 60)

● (1, 8 − 1, 5)×60 = 18

141
14ª edição do Programa de Preparação - UFRJ Consulting Club Apostila

4. Fit Interview

4.1. Introdução

4.1.1. O que é Fit Interview?

Uma das etapas mais importantes nos processos seletivos de qualquer empresa
são as de entrevistas. Em consultoria, essa avaliação é dividida em 2 partes: Fit
Interview e Case Interview. A Fit Interview, tema desse módulo, é a fase na qual é
avaliado se as qualidades, motivações e modus operandi do candidato estão
alinhadas com a cultura da empresa. Essa entrevista, geralmente realizada de
forma online, contém perguntas sobre experiências no âmbito pessoal,
acadêmico e profissional, baseadas no currículo, além de perguntas acerca do
comportamento do entrevistado em situações passadas que podem se
assemelhar ao ambiente de trabalho da consultoria estratégica.
Frequentemente, os candidatos direcionam seu foco somente para os treinos de
case, não dando a atenção necessária para essa etapa, o que é extremamente
prejudicial, uma vez que a Fit é tão importante quanto um Case, sendo uma
ótima oportunidade de se destacar em relação aos concorrentes.

4.1.2. Como Funciona?

As entrevistas, de modo geral, possuem a duração de 10-15 minutos, podendo se


estender até 30 minutos, e são compostas por dois rounds eliminatórios. Durante
esses rounds, o entrevistado terá contato com diversos membros da firma,
sendo, geralmente, o primeiro com membros juniors e o segundo com sócios.
Esses rounds são constituídos por algumas entrevistas, em média, de 2 à 3, e
cada uma delas é composta, geralmente, de 4-6 perguntas. Normalmente, não
há diferença em termos de dificuldade entre os rounds, como se pode supor.

4.1.3. As Big 3

Direcionando o foco para as 3 maiores empresas do ramo da consultoria


estratégica no Brasil, temos a Mckinsey & Company, o Boston Consulting Group

142
14ª edição do Programa de Preparação - UFRJ Consulting Club Apostila

(BCG) e a Bain & Company. Quando o candidato se inscreve para uma dessas
empresas, ou qualquer outra consultoria, é de extrema importância que ele faça
uma pesquisa prévia sobre a entrevista, entendendo a cultura da empresa e o
que ela busca em um candidato. Além disso, é imprescindível demonstrar
motivação para fazer parte daquela instituição específica. Vamos analisar
algumas particularidades de cada uma dessas 3 firmas:

● McKinsey & Company: a Fit Interview Da McKinsey é chamada de PEI,


sigla para Personal Experience Interview - em português, entrevista de
experiências pessoais. Nessa entrevista, há o foco nas perguntas
comportamentais, visando determinar se as habilidades, experiências,
motivações e personalidades do candidato são alinhadas com a cultura da
companhia, destacando a importância de estudar os valores da firma
anteriormente. No geral, as características principais buscadas são:
impacto pessoal, motivação empreendedora, capacidade de solucionar
problemas e liderança inclusiva, habilidade de liderar uma equipe com
diversas vivências, de modo a estimular o desenvolvimento e o alcance do
potencial máximo de todos. Durante as respostas, é recomendado
mencionar conquistas relevantes e demonstrar orgulho por elas. Alguns
exemplos dessas perguntas são:
○ "Conte-me sobre um momento no qual você superou uma
tarefa extremamente difícil?"
○ "Conte-me sobre uma vez que você resolveu algum conflito
dentro da sua equipe"
○ "Conte-me de um momento que você mudou a cabeça de uma
pessoa"

A cultura dessa empresa dá enorme destaque à comunicação. Tendo em


vista que essa foi a primeira das Big 3, ela possui uma grande base de dados e
informações, valorizando o seu bom uso. Uma curiosidade interessante é que
uma consultora da McKinsey é a autora do livro O Princípio da Pirâmide, que
sugere abordagens diferentes para a linearidade na comunicação, que é uma
habilidade muito valorizada junto à persuasão.

● Boston Consulting Group: o BCG possui um estilo muito semelhante à


McKinsey, optando por perguntas comportamentais, cujas respostas

143
14ª edição do Programa de Preparação - UFRJ Consulting Club Apostila

devem descrever experiências que demonstrem as características


procuradas.

A cultura dessa firma é focada em fazer sempre análises rigorosas e


meticulosas, prezando muito pela habilidade de problem solving. Ela valoriza
o pensamento lógico, e possui uma abordagem acadêmica. Além disso, um
de seus grandes valores é considerar cada situação única, acreditando em
soluções e frameworks personalizados para cada cliente. Logo, ela valoriza a
pesquisa, o desenvolvimento e a ciência de dados fortemente.

● Bain & Company: na Bain & Company, a entrevista recebe o nome de


"Experience Interviews". Essa firma é a mais nova terceira das Big3 e, em
geral, apresenta, em sua entrevista de Fit, um menor número de
perguntas comportamentais em comparação com as duas empresas
citadas anteriormente.

A cultura da Bain baseia-se na motivação empreendedora, dentro da qual são


muito valorizadas a iniciativa e a inovação. Por ser a menor das Big3, pode-se
argumentar que é uma firma que apresenta bastantes oportunidades de
protagonismo e autonomia dos membros. Além disso, a Bain possui uma
cultura descontraída e jovial, sendo menos tradicional, exemplificada pela
frase bastante conhecida entre os Bainies: work hard and play hard.

4.2. Características procuradas em um candidato

São diversas as características e habilidades buscadas em um candidato durante essa


etapa, sendo 8 delas as mais procuradas: motivação, problem solving, liderança,
criatividade, persuasão, impacto pessoal, adaptabilidade e empreendedorismo.
Portanto, deve-se tentar demonstrá-las aos avaliadores de forma natural, mantendo-se
fiel à personalidade individual, com o objetivo de passar a imagem verdadeira. Uma
forma de mostrar todas essas habilidades é trazer alguma experiência vivida
anteriormente que exemplifique-as.

● Motivação: a principal característica procurada em Fit Interviews é a


motivação. Desse modo, deve-se transparecer, naturalmente, o desejo do
candidato de alcançar a vaga em determinada empresa. Para isso, uma

144
14ª edição do Programa de Preparação - UFRJ Consulting Club Apostila

boa prática é estudar previamente a cultura da firma e, se possível, fazer


comentários que demonstrem conhecimento e identificação com os
valores durante a entrevista. Além disso, mostrar entusiasmo, busca por
aprendizado e excelência é fundamental.

● Problem Solving: outra característica muito procurada nos candidatos é a

capacidade de resolver problemas, tendo em vista que uma das principais


tarefas de um consultor é achar soluções viáveis para determinada
questão. Dessa maneira, é preciso demonstrar capacidade analítica,
business sense e clareza na estruturação das ideias.
● Liderança: a liderança é uma habilidade primordial de um consultor, o
qual necessita lidar com gestão de pessoas, delegação de tarefas, entre
outras atividades. Um bom líder é aquele que possui características como
paciência, maturidade, responsabilidade, integridade, empatia, confiança,
inteligência emocional e empoderamento. Além disso, vale ressaltar que
lideranças informais e inclusivas também são bem vistas pelo
entrevistador, destacando a importância de não assumir o lugar do líder
formal, mas sim mostrar iniciativa, proatividade e organização, além das
habilidades mais interpessoais citadas antes. Dessa forma, durante uma
entrevista, deve-se tentar transparecer alguns desses atributos.
● Criatividade: uma das características mais procuradas, não só em
consultores, como também em todo profissional, é a criatividade. A
capacidade de desenvolver algo novo é muito valorizada, e é de grande
auxílio na resolução de problemas complexos, o que colabora com outra
habilidade, o problem solving.
● Persuasão: a persuasão é muito importante na rotina de um consultor, o
qual trabalha com comunicação e clientes. É necessário, portanto, que o
candidato seja capaz de apresentar suas ideias e persuadir o público, seja
com colegas de equipe, seja na apresentação de projetos para clientes.
Logo, a argumentação e a confiança são qualidades fundamentais, sendo
assim, uma boa prática estudar técnicas de persuasão.
● Impacto pessoal: para garantir um ambiente de trabalho agradável, as
empresas levam em consideração o impacto que cada indivíduo causa na
equipe. Assim, espera-se que o candidato saiba trabalhar bem em grupo,
colaborando, respeitando seus colegas e se comunicando de forma clara e

145
14ª edição do Programa de Preparação - UFRJ Consulting Club Apostila

eficiente, de modo a gerar resultados positivos. Além disso, é avaliado se o


entrevistado se mantém confiante e controlado, se ele é agradável e se
traz credibilidade em suas respostas. Ressaltando que ser agradável é um
fator muito considerado no momento da entrevista, uma vez que as
firmas buscam alcançar um bom convívio dos funcionários.
● Adaptabilidade: tendo em vista a variedade de setores e projetos com os
quais um consultor lida no seu dia a dia, é importante que ele seja
adaptável, e consiga mudar para um novo projeto de maneira flexível.
Além disso, é esperado que ele lide de forma positiva com situações
desafiadoras. Logo, uma boa forma de demonstrar versatilidade é
trazendo experiências que envolvam grandes mudanças e momentos de
trabalho sob pressão.
● Empreendedorismo: por fim, uma característica procurada nas firmas de
consultoria é o espírito empreendedor. Superar obstáculos e atingir metas
requerem uma mentalidade inovadora e aberta para novas ideias,
acompanhada de uma vontade constante de aprendizado e crescimento.
Além de conseguir enxergar gaps como oportunidades, demonstra uma
boa capacidade em identificar quais frentes explorar para se ter os
melhores resultados. Dessa forma, demonstrar iniciativa e inovação
destaca os candidatos dos seus concorrentes.
● Inteligência emocional: demonstrar boa capacidade de lidar com
emoções e sentimentos em momentos de pressão ou frustração é visto
muito bem pelas empresas, que buscam essa inteligência para lidar com
obstáculos e buscar novos caminhos para situações cotidianas.

4.3. Técnica do questionamento

De acordo com a técnica de questionamento baseada em critérios, os avaliadores


fazem perguntas que englobam os pontos a seguir:

● Atitude:
Como o candidato percebe e dá valor às suas ações em situações de
trabalho ou em situações adversas.

Exemplo: "Por qual motivo você abriria mão de entregar um projeto com
excelência?"

146
14ª edição do Programa de Preparação - UFRJ Consulting Club Apostila

● Autoavaliação:
Como o entrevistado reflete sobre si em alguma situação.

Exemplo: "Nessa situação, o que você teria feito de diferente?" ou "Por


quais razões você acha que eu contrataria outra pessoa no seu lugar?"

● Avaliação externa:
Como as pessoas que convivem com você te enxergam em algum
ambiente.

Exemplo: "O que seus melhores amigos responderíam se eu perguntasse


a eles o que te torna excepcional?"

● Comparação:
O entrevistado deve mostrar o que tem ou não em comum com outra
pessoa.

Exemplo: "Quem é a pessoa que mais te inspira?"

● Experiências:
O candidato é induzido a mostrar situações que ajudaram ele
desenvolver alguma habilidade procurada pela empresa.

Exemplo: "Me conte sobre quando você discordou de alguém em uma


posição hierárquica superior a você".

● Exemplos:
O avaliador quer diretamente um exemplo em que foi desenvolvida uma
habilidade. Diferentemente das experiências, os exemplos não são implícitos.

4.4. Perguntas mais frequentes


Existem algumas perguntas que costumam aparecer com bastante frequência nas Fit
Interviews. Tendo em vista que o entrevistador analisa diversas respostas para essas
questões, é preciso se destacar e respondê-las de forma linear e clara para de fato
mostrar a individualidade do candidato. Assim, é indicado ter em mente uma resposta
bem estruturada para os principais questionamentos, porém com o cuidado de não

147
14ª edição do Programa de Preparação - UFRJ Consulting Club Apostila

parecer uma resposta decorada, visto que candidatos "robóticos" são mal vistos. Ao
responder as perguntas, é uma boa prática se manter aberto para a evolução da
conversa baseada nessas respostas, demonstrando habilidades interpessoais e a
capacidade de manter uma conversa agradável, já que o entrevistador pode, em certos
momentos, emendar em perguntas mais específicas.

4.4.1. Me conte mais sobre a sua trajetória

Essa pergunta é muito comum em Fit Interviews e é uma excelente


oportunidade para trazer experiências passadas que demonstrem suas
habilidades e características. Esse questionamento normalmente é feito no início
da conversa como forma de "quebrar o gelo", porém ela é tão avaliada quanto
qualquer outra. Durante essa resposta, é muito comum o entrevistado se perder
e falar em excesso ou muito pouco, então é extremamente importante ser
organizado e linear. Nesse tópico, é comum que os entrevistadores interrompam
o candidato e peçam que ele desenvolva mais algum ponto específico, podendo
acontecer com frequência até de fazer perguntas adicionais sobre ele. Dessa
forma, é preciso estar preparado para aprofundar cada parte apresentada
resumidamente e também conectar com os pontos mencionados no currículo.
Como exemplo de resposta, temos:

"Eu nasci em São Paulo, mas fui criado no Maranhão, em uma área bastante
rural. Com 18 anos, eu decidi estudar engenharia de produção na UFRJ, não só
porque o Rio de Janeiro é uma cidade muito bonita, mas principalmente pela
excelência do curso e pelas oportunidades de trabalho. Adentrando no meu
interesse por consultoria, como eu estava desde 18 anos, intrigado com essa
carreira, após o semestre X, eu decidi entrar em projeto de extensão Y da
universidade, focada em consultoria, onde participava de projetos reais e
simulados, nos quais aprendi muito sobre como resolver problemas e, vender
minhas ideias, e, claro, trabalhar em equipe. No 9° período da faculdade, decidi
entrar em um estágio de consultoria na empresa Z, onde trabalhei diretamente
em 3 projetos voltados para crescimento, M&A e otimização de custos, os quais
abriram as portas para novos projetos com esses clientes. Nesse cargo, eu
aprendi como aprimorar a estratégia da empresa e quais seriam seus próximos

148
14ª edição do Programa de Preparação - UFRJ Consulting Club Apostila

passos, o que me interessou muito. Por esse motivo, eu acredito que a


consultoria estratégica seria uma área onde essas duas coisas poderiam se
juntar. Além de ver na empresa X oportunidades de trabalhar em projetos de
otimização de custos, os quais eu me identifico, ela possui um incentivo grande
à participação dos membros em diferentes demandas, o que também me
interesso, pois me considero uma pessoa flexível."

Naturalmente, a resposta para uma pergunta ampla como trajetória requer uma
resposta maior e mais completa, mas sempre estruturada. Analisando o
exemplo, o candidato inicia com uma breve introdução sobre de onde ele veio,
para depois entrar no tópico da trajetória profissional, o que é uma ótima forma
de iniciar essa resposta. Em seguida, comenta sobre o seu interesse em
consultoria, transparecendo motivação. Após esse comentário, ele guia o
entrevistador por sua trajetória na universidade, onde menciona sua extensão e
seu estágio em uma empresa, explicando o que ele fez e aprendeu em cada um
deles. Observa-se que ele faz comentários positivos sobre sua experiência,
trazendo detalhamento e conectando com habilidades desejadas pelas firmas.
Ele também ressalta como isso impactou a empresa ao mencionar que sua
participação contribuiu para abrir portas para novos projetos, e também o que
ele achou interessante.

Além disso, percebe-se que o candidato traz todas as informações de maneira


linear, seguindo uma linha cronológica. Por fim, ele desenvolve o raciocínio de
como sua trajetória o encorajou para trabalhar em consultoria e finaliza
demonstrando sua motivação e alinhamento com a cultura da organização. Um
ponto a ser ressaltado é que, em sua resposta, o entrevistado já responde o
porquê do interesse em consultoria e o porquê do interesse nessa empresa,
prática muito bem vista e encorajada.

4.4.2. Por que consultoria?

Ao responder essa pergunta, evita-se mencionar que a consultoria pode ser uma
escolha de apenas alguns anos, e sim focar nos pontos positivos que levam
alguém a escolher essa carreira. Além disso, não é recomendável citar coisas que
o candidato gosta, mas que não seriam possíveis em consultoria, o que é um erro

149
14ª edição do Programa de Preparação - UFRJ Consulting Club Apostila

muito cometido por aqueles que buscam uma carreira no meio acadêmico e se
interessam mais por ensinar e pesquisar, por exemplo. Uma boa forma de
responder é desenvolver o porquê da sua paixão pela consultoria e mostrar que
se identifica com as características do trabalho, sendo capaz de se adaptar ao dia
a dia com muita motivação. Um bom exemplo de como responder essa
pergunta é:

"Desde do início dos meus estudos, essa carreira sempre me intrigou.


Inicialmente, o que eu tinha em mente era a oportunidade de aprendizado de
trabalhar em uma diversidade de setores e funções, tendo em vista que eu me
interesso e me adapto em diferentes situações. Então, para decidir se isso
realmente era o caminho que eu gostaria de seguir, após o meu X˚ período na
universidade, eu entrei no estágio que citei antes. Nele, fiz um projeto que foi
decisivo para meu interesse se confirmar, voltado para utilização de energia
solar em uma firma, porque foi onde tive espaço para inovar e pude aplicar um
conteúdo que eu estudei bastante a fundo em pesquisas na minha
universidade. No meu ponto de vista, conversar com colegas e clientes
experientes e ajudá-los a desenvolver estratégias que resolvam seus problemas,
ainda sem ser um funcionário das empresas deles de forma fixa, é fascinante.
Eu espero que, com a consultoria, eu possa trabalhar com tarefas mais
estratégicas, conceituais e de projetos diversos."

Analisando essa resposta, percebe-se que, desde o início, o candidato demonstra


sua motivação ao dizer que a consultoria sempre o intrigou, transparecendo uma
certa inquietação em um sentido positivo. Em seguida, ele mostra
conhecimento sobre o assunto ao citar algumas informações sobre como é o
trabalho de um consultor. Outro ponto que ele trouxe foi uma experiência
passada, demonstrando que a decisão de entrar para a carreira de consultoria foi
muito planejada, o que ressalta sua cautela na tomada de decisões.

Além disso, ele exemplifica e detalha seus aprendizados no estágio, conectando


às suas preferências pessoais e as habilidades importantes para um consultor.
Nessas experiências, ele comenta sobre a eficiência de suas análises,
demonstrando a habilidade de problem solving, e a capacidade de inovação,
transparecendo um espírito empreendedor e criativo. Além disso, ele demonstra
colaboração coletiva ao mencionar que se interessa em conversar com colegas

150
14ª edição do Programa de Preparação - UFRJ Consulting Club Apostila

experientes e pelo contato com clientes. Por fim, o entrevistado conclui


retomando a questão da motivação e esclarece o que espera encontrar em um
futuro trabalho em consultoria estratégica.

4.4.3. Por quê essa empresa?

A resposta dessa pergunta representa a grande chance do candidato de


demonstrar seus conhecimentos sobre a cultura da empresa, além de seu
interesse por ela especificamente. Em geral, as diferentes firmas valorizam
pontos distintos em relação à cultura da companhia, além do reconhecimento
de serem mais fortes em determinados setores e escopos que outras,
ressaltando que não se deve depreciar uma firma para elogiar outra. Por isso, o
conhecimento da cultura da empresa é primordial. Há, também, uma diferença
na forma de responder dependendo do tamanho da firma para qual se está
fazendo a entrevista.

● Todas as empresas: para empresas em geral, deve-se focar na cultura,


benefícios e vantagens da firma. Uma abordagem interessante é trazer
insights adquiridos através do networking do candidato. Nessa
abordagem, pode-se mencionar quais são as conexões que o entrevistado
possui dentro da firma, e desenvolver quais foram as conversas recentes
que ele teve com alguém que trabalha ou conhece a empresa. Ao
desenvolver a conversa, é bem visto enfatizar os pontos abordados pelo
membro a respeito da cultura que o candidato tenha achado interessante.
Dessa forma, demonstra-se conhecimento, e, também, o fato do
candidato já possuir conexões dentro da empresa e já estar motivado para
fazer parte dela. Além disso, pode-se mencionar eventos da organização
nos quais esteve presente, como um bate-papo com consultores, um
treinamento ou uma palestra. Falar sobre projetos que essa organização
realizou e incitaram um interesse por ela também é uma ótima forma de
responder a essa pergunta. Por fim, uma resposta a ser evitada é dizer que
aquela empresa possui um maior prestígio - afinal, essa resposta não foca
nos valores, benefícios ou vantagens oferecidos pela empresa.

151
14ª edição do Programa de Preparação - UFRJ Consulting Club Apostila

● Empresas maiores: no caso de empresas maiores, uma boa forma de


abordar essa pergunta é enfatizar as oportunidades oferecidas por essa
organização específica. Com a McKinsey, BCG ou Bain, tem-se uma
grande variedade de projetos para trabalhar, além de uma visibilidade
global.
● Empresas menores: Empresas menores frequentemente possuem um
nicho mais específico de mercado, ou uma forma específica de operar.
Dessa forma é uma boa prática enfatizar o quanto se está interessado
nessa especificidade e trazer experiências passadas que confirmam isso.
Além disso, vale ressaltar que, em empresas menores, existe uma
proximidade maior no ambiente de trabalho e mais espaço para inovação.

Para ilustrar, temos esse exemplo de resposta:

"Como eu sempre tive o interesse em consultoria, é lógico considerar a empresa


X, tendo em vista que ela é líder no seu segmento. O que realmente fez eu
tomar a decisão de me candidatar para uma vaga, e esperar trabalhar aqui no
futuro, foi minha conversa com um membro da firma, o qual trabalha aqui há
quase um ano, que conheci durante um crack the case da empresa que foi
realizado na minha faculdade. Durante nossas conversas, ele tirou dúvidas que
eu tinha sobre como a cultura da empresa X se aplica no dia a dia, mas o que
mais se destacou pra mim e me fez buscar entrar foi a valorização das
melhores ideias. Acompanhando relatos dele sobre seus primeiros projetos,
gostei muito de como cada membro da equipe, mesmo os mais recentes e
menos experientes, são muito valorizados e têm sua contribuição incentivada.
Ficou muito nítido que seus pontos de vista eram sempre bem-vindos e de fato
essenciais para o andamento do projeto, tendo impacto direto nas decisões dos
clientes, o que me pareceu uma oportunidade incrível e única de me desafiar e
sair da zona de conforto com tanta responsabilidade, sobretudo sendo ainda
tão novo. Acredito que a atmosfera da empresa tenha bastante a ver com
meus objetivos profissionais que descrevi quando comentei sobre minha
trajetória"

Analisando mais profundamente essa resposta, percebe-se que ele inicia


comentando sobre essa empresa ser líder de mercado, algo que pode parecer
semelhante ao prestígio da empresa, porém essa informação tem o foco no bom

152
14ª edição do Programa de Preparação - UFRJ Consulting Club Apostila

desempenho da organização em determinado segmento, enquanto o prestígio


traz uma ideia de status. Após esse comentário, ele adentra nas conversas que
teve, mostrando ter networking e detalhando brevemente sua história ao
mencionar a forma como conheceu o membro, de modo a trazer veracidade. Em
seguida, ele desenvolve domínio sobre a cultura da empresa e o que fez ele se
interessar pela vaga. Por fim, conclui com uma frase curta e capaz de
demonstrar novamente sua motivação de se juntar à instituição, devido ao
alinhamento com seus objetivos profissionais.

Esse tipo de resposta pode se aplicar a empresas que não fazem parte das Big3,
como, por exemplo, a Visagio, onde fazem especificamente a pergunta: "Por que
a Visagio e não as Big3?". Além dela, temos a Mirow, L.E.K. Consultoria, entre
outras firmas muito desejadas pelos candidatos, mas que não fazem parte desse
trio e que podem se encaixar nesse tipo de resposta. Além disso, é importante
saber se a empresa é de consultoria estratégica ou de gestão, como a Visagio por
exemplo é uma de gestão.

4.4.4. Você tem alguma pergunta pra mim?

Essa pergunta normalmente é feita no final da entrevista toda, ou seja, após a


Case Interview. Esse é o momento do candidato fazer perguntas para o
entrevistador, tendo em vista que, assim como a empresa precisa ter certeza da
conexão dele com sua cultura e o cargo de consultor, o próprio candidato precisa
ter bastante segurança da decisão que está tomando para sua carreira. Esse
momento é usado para sanar dúvidas, e, até mesmo, curiosidades que possam
existir sobre a firma e a consultoria estratégica. Vale ter, no entanto, o cuidado
para não fazer perguntas muito pessoais ao consultor, ou perguntas óbvias que
poderiam ser respondidas por uma rápida visita ao site da empresa. É
importante ter já algo em torno de 5 a 7 perguntas bem pensadas por entrevista
em mente, já que costumam ser feitas de 2 a 3 na hora da Fit Interview, já que
algumas delas podem vir a não fazer sentido no momento, dada alguma
resposta inicial do consultor. Essa seção da entrevista costuma durar apenas 5
minutos.

153
14ª edição do Programa de Preparação - UFRJ Consulting Club Apostila

OBS: os exemplos da aula têm como objetivo facilitar o entendimento, mas não
devem ser tomados como respostas para serem memorizadas, já que cada
candidato é único.

4.5. Recursos de comunicação - Método STAR


Durante uma entrevista, é de extrema importância que o entrevistado possua uma
comunicação clara e objetiva, a fim de demonstrar um pensamento organizado. Um
ponto importante é que muitas perguntas, por mais que não peçam diretamente
exemplos de experiências reais do candidato, podem ser melhor respondidas dessa
maneira.

Diante dessa perspectiva, o método STAR, metodologia frequentemente utilizada em


entrevistas de emprego, é de grande auxílio, uma vez que ajuda o locutor a organizar
suas respostas, fornecendo exemplos concretos que demonstram a competência e
aptidão para a vaga, de forma bastante linear e lógica. Ele é dividido em 4 partes que se
conectam bastante, e a transição entre elas deve ser feita de forma natural.

4.5.1. Situação

O primeiro ponto da metodologia é o de Situação. Nele, é preciso descrever a


situação da forma mais específica possível e com o maior detalhamento. Caso o
candidato esteja apenas no início de sua carreira profissional, poderão ser
descritas situações ocorridas na universidade ou em algum trabalho voluntário,
porém sempre dando preferência para os exemplos profissionais, quando
houver, e mais recentes. Desse modo, pode-se estruturar essa etapa com os
seguintes pontos:

● Quando: referente ao período de tempo que a situação ocorreu: ano;


período da universidade; tempo na empresa; entre outros.
● Onde: relacionado ao local, como empresa, estágio ou universidade,
podendo também ser mais específico em setor, curso, disciplina, entre
outros.
● Com quem: trata-se de pessoas relevantes para a narrativa. Vale o adendo
de que não é necessário mencionar os nomes dos envolvidos, em prol de

154
14ª edição do Programa de Preparação - UFRJ Consulting Club Apostila

manter a ética da resposta e focar sempre nas pessoas essenciais, de


modo que só mencionar o cargo da pessoa ou função dela no contexto é
suficiente.
● O que: introdução do que ocorreu e ponto de transição para a tarefa.

4.5.2. Tarefa

O segundo tópico é a Tarefa, que consiste em descrever quais eram as


responsabilidades individuais do candidato naquela ocasião. A Tarefa, é um bom
momento para comentar sobre alguma situação desafiadora, como: prazo
apertado, um imprevisto, um escopo nunca trabalhado antes, uma equipe pouco
experiente, um cliente mais exigente, um cliente que era um parceiro
fundamental da firma, entre outros desafios.

4.5.3. Ação

Em terceiro lugar, vem a Ação. Nesse momento, é preciso contar quais atitudes
foram tomadas para resolver aquela situação desafiadora e de que forma o
candidato chegou nesse caminho. Nessa hora, é importante demonstrar a linha
de raciocínio com clareza, o problem solving e o pensamento estruturado e
organizado para descrever o que exatamente foi feito com detalhes. Além disso,
esse é um bom momento para demonstrar alguma habilidade apreciada como
liderança, criatividade ou espírito empreendedor.

4.5.4. Resultado

O quarto e último ponto é o Resultado, ou seja, o que foi obtido após a ação.
Nessa parte, é recomendado citar dados reais para trazer embasamento e
credibilidade para a história, além de, assim como no currículo, quantificar a
conquista que o candidato teve, ou seja, qual foi o resultado prático gerado pelo
seu esforço e pela qualidade do seu trabalho. Por fim, é bem visto explicar o que
foi aprendido com essa situação, isto é, se houve alguma lição importante que
impactou no crescimento pessoal, profissional, ou acadêmico do candidato.

155
14ª edição do Programa de Preparação - UFRJ Consulting Club Apostila

Para ilustrar a forma correta de aplicar a metodologia STAR em uma resposta, temos a
seguinte pergunta:

"Fale sobre alguma situação em que você tenha precisado completar uma tarefa em
meio a um desafio"

"Em 2021, quando estava em meu primeiro estágio na empresa X, eu fui alocado no
meu primeiro projeto junto a uma consultora mais experiente, além de um supervisor,
e o objetivo principal era traçar estratégias para recuperar o market share do cliente
com urgência. Com uma semana de projeto, minha colega de equipe precisou
representar a firma em uma viagem de última hora, deixando somente eu e meu
supervisor a cargo do projeto, de modo que o planejamento seguiu o mesmo, mas
sem outra pessoa para dividir as tarefas como antes. Para lidar com isso, garantindo a
qualidade que o cliente precisava com a urgência necessária, busquei separar as
estratégias pensadas em 2 grupos para análise: primeiro, estudei a viabilidade de
aplicar as 5 que eu acreditava que teriam maior impacto para o cliente e validei esse
pensamento com meu supervisor, que concordou, de forma que o cliente já pôde,
durante o projeto, começar a colocá-las em prática. Em seguida, foquei nas 10
restantes no mês de projeto. Com isso, a empresa do cliente conseguiu retomar o
market share que teve no ano anterior, atingindo seu objetivo, e eu aprendi bastante
sobre segmentação de tarefas e priorização baseada em impacto e agilidade."

Nesse exemplo percebemos que o entrevistado inicia a resposta descrevendo a


situação falando quando ocorreu - em 2021 -, onde - no 1° estágio na empresa X -, com
quem - um consultor mais experiente -, e o que - foi alocado em um projeto. Após essa
introdução da situação, ele fala da tarefa que teve que cumprir, sendo essa traçar
estratégias para recuperar o market share de um cliente. Dando continuidade, ele
segue para qual ação tomou, descrevendo como separou a análise em 2 grupos, o que
demonstra habilidades de problem solving, criatividade e liderança, e desenvolveu a
partir disso. Por fim, ele cita o resultado, que consistiu no aumento do market share, e
conclui ressaltando a aprendizagem proporcionada por essa experiência.

Além da técnica STAR, existe também o método Parade, o qual é bem semelhante à
vista anteriormente. Além dos pontos já abordados, essa nova estratégia traz uma visão
de problema e consequência antecipada desse problema, além de ressaltar a
importância de demonstrar a racionalidade na tomada de decisões.

156
14ª edição do Programa de Preparação - UFRJ Consulting Club Apostila

4.6. Como se portar


Em uma entrevista de emprego, o objetivo principal é causar uma boa primeira
impressão no avaliador. Para isso, é importante que haja uma preocupação com a
imagem que o candidato passa.

4.6.1. Linguagem

Em primeiro lugar, o entrevistado deve possuir uma linguagem condizente com


a situação. O ato de treinar o porte antes da entrevista permite que, no momento
oficial, o candidato utilize as dicas como uma forma de controlar o nervosismo,
parecendo mais seguro e confiante.

● Verbal: começando pela linguagem verbal, é preciso que a comunicação


seja clara e objetiva.

○ É indicado evitar termos vagos e tentar sempre responder tudo


de forma específica.

○ Durante a fala, é crucial evitar gírias, palavrões e termos que


talvez não sejam do conhecimento do entrevistador, ou passem
muita informalidade e falta de profissionalismo.

○ Não é bem vista uma grande demora para responder as


perguntas, mas vale sempre ter a cautela de não interromper a
fala do outro.

○ Além disso, é uma boa prática demonstrar entusiasmo em sua


fala, para que o avaliador tenha a impressão que o candidato é
motivado para aquela vaga, porém não de maneira exagerada.

○ Caso o avaliado não tenha compreendido alguma pergunta, é


importante que ele não desvie dela, e sim, esclareça suas
dúvidas antes de responder. Ressaltando que pedir
esclarecimento, desde que seja sobre algo não óbvio, não é uma
prática mal vista

○ Durante a comunicação, evite discordar exageradamente de


algum ponto, ignorar perguntas, contar piadas ou demonstrar

157
14ª edição do Programa de Preparação - UFRJ Consulting Club Apostila

muita emoção. A comunicação deve refletir a clareza, equilíbrio


e calma do candidato.

○ Outro ponto relevante é se atentar ao tom de voz: é esperado


que ele seja equilibrado, ou seja, suficientemente alto para ser
ouvido, porém não alto demais. Um tom muito baixo de voz
pode transparecer monotonia e falta de entusiasmo, enquanto
um tom alto demais pode causar incômodo.

○ Esse equilíbrio também precisa existir no conteúdo da fala, logo


evitar auto elogios exagerados ou críticas negativas sobre si,
assim como não trazer aspectos negativos de outras firmas e
antigos colegas de trabalho, são práticas importantes a serem
consideradas.

○ Ter em mente o teste do avião: durante a sua avaliação, seu


entrevistador vai estar sempre se perguntando: “Se eu realizasse
um projeto com esse candidato, e nós tivéssemos que viajar até
a Inglaterra, eu teria 11 horas agradáveis com ele dentro de um
avião?”, então ser simpático e agradável é muito importante para
quem estiver conduzindo a entrevista, ter vontade de trabalhar
com o entrevistado.

○ Resumidamente, é importante que o candidato seja formal,


evitando também chamar o entrevistador por apelidos, e
transpareça sua personalidade na fala.

● Corporal: além da linguagem verbal, também é preciso se atentar à


linguagem corporal.

○ É importante manter uma postura ereta, a qual também ajuda a


melhor projeção da voz, evitando demonstrar insegurança e
ansiedade como uma postura curvada faz.

○ Uma boa prática é manter o foco no entrevistador, ou seja,


manter o contato visual com a câmera, caso seja online, ou com
as pessoas presentes na sala, caso seja presencial, evitando fixar
o olhar em janelas, mesas, chão, teto e outros objetos. Em suma,

158
14ª edição do Programa de Preparação - UFRJ Consulting Club Apostila

é importante demonstrar interesse e foco, mas sem deixar as


pessoas desconfortáveis.

○ Com relação às mãos, é recomendável que o candidato


gesticule, porém não de maneira exagerada com gestos muito
grandes, evitando também palmas, estalos de dedos, e outros
sons com as mãos.

○ No caso de uma entrevista onde o candidato permanece


sentado em uma cadeira, como na maioria das vezes, deve
evitar-se o movimento pendular, movimento de um lado para o
outro ou para frente e para trás da cadeira, assim como é
importante evitar se sentar de forma muito relaxada. Também
deve-se evitar balançar os pés.

○ Uma boa prática é tentar sorrir em momentos cabíveis, como na


hora de cumprimentar o entrevistador, com o objetivo de
parecer amigável, mas reconhecer momentos mais sérios da
entrevista que requerem a seriedade do candidato.

○ Nessa perspetiva, de forma geral, a expressão facial deve ser


neutra, com sorrisos e empolgação em momentos que reflitam
isso, como ao falar sobre a vontade de entrar na firma ou a
superação de um desafio, por exemplo.

○ Para disfarçar um possível nervosismo, evite mexer em celulares,


unhas, colares, cabelos ou quaisquer objetos, pois candidatos
que não conseguem manter suas mãos paradas demonstram
insegurança e ansiedade.

○ Além disso, é uma boa prática fazer movimentos discretos de


concordância com a cabeça, inclinar-se sutilmente para frente
para ouvir as perguntas, e, de forma alguma, cruzar os braços.
Por fim, quando sair da entrevista, saia de forma confiante.

159
14ª edição do Programa de Preparação - UFRJ Consulting Club Apostila

4.6.2. Vestimenta

Tendo em vista a importância da primeira impressão, é preciso estar de acordo


com o dress code, separando, com antecedência, uma roupa profissional. Opte
por roupas com cores neutras como: branco, preto, cinza, bege, creme, etc.

Para mulheres, evitam-se roupas com decotes muito acentuados ou roupas


muito curtas, sendo vestidos ou saias abaixo do joelho, assim como camisas
formais e calças sociais, boas opções. Com relação aos sapatos, o uso de salto,
sobretudo os mais baixos, é uma opção segura, prezando sempre pelo conforto e
simplicidade. Por fim, falando sobre os acessórios, é válido que mulheres evitem
colares, brincos e outras jóias muito coloridas e chamativas, assim como
maquiagens muito intensas.

Para homens, frequentemente opta-se por ternos ou camisas sociais e calças


sociais. Deve-se usar sapato social.

Caso haja a necessidade de utilizar bolsas ou mochilas, é indicado optar por itens
discretos e, durante a entrevista, deve-se manter o acessório ao lado tanto
homens quanto mulheres.

4.7. Dicas finais


Além de tudo que já foi abordado previamente, existem 4 últimas boas práticas a
serem destacadas para serem aplicadas no momento da Fit Interview. São elas:

● Seja objetivo: de acordo com o conceito de answer first, é importante que


o candidato inicie sua fala com a resposta, para depois destrinchá-la. Além
disso, é recomendável ir direto ao ponto e não "enrolar" muito nas
respostas, porém com o adendo de que ser objetivo não é ser genérico,
pois deve-se trazer detalhamento.

● Escolha suas experiências com cautela: traga exemplos de experiências


que aconteceram preferencialmente no âmbito profissional e que
ocorreram recentemente. Se for possível, traga exemplos relacionados à
consultoria que demonstrem interesse na empresa. Também é preciso ter
cuidado e não falar negativamente sobre um antigo trabalho, e, assim
como na Cover Letter, é primordial não focar em justificativas ou

160
14ª edição do Programa de Preparação - UFRJ Consulting Club Apostila

desculpas por falhas e erros, mas sim mostrar, por meio de exemplos,
como determinado aspecto vem sendo melhorado. Quando a experiência
envolve uma equipe, o candidato deve manter o foco em si, citando outras
pessoas e suas ações quando for pertinente para que o entrevistador
entenda a experiência relatada, mas sem perder o protagonismo da
história.

● Não minta: trazer sempre informações e histórias verdadeiras que reflitam


a personalidade real do candidato é importante, pois, caso consiga a vaga,
é esperado que o candidato seja apto a fazer o trabalho e se torne um
bom membro para a empresa.

● Seja agradável: ser simpático e bem humorado é primordial em qualquer


entrevista: afinal, o objetivo do candidato é causar uma boa primeira
impressão. Logo, a comunicação deve ser leve, respeitosa e fluida, de
modo que o entrevistador perceba como o candidato pode ter um
impacto positivo ao trabalhar em equipe ou lidar com clientes.

4.8. Como se preparar?


● Para se preparar para uma entrevista, é recomendável começar com uma
pesquisa sobre a cultura da empresa.

● Após essa etapa, o candidato deve estudar quais são as perguntas mais
frequentes e quais são as mais difíceis também, a fim de diminuir as
chances de surpresa e gerar confiança.

● Além disso, o candidato deve preparar exemplos antecipadamente de


experiências passadas que demonstrem as soft skills desejadas em um
consultor, pensando em situações coringas que podem ser adaptáveis
para diferentes perguntas.

● Vale sempre relembrar sobre formular perguntas para o momento final da


entrevista, repensando elas a cada round para sempre se ter uma boa
variedade de opções e adequar isso a cada entrevistador.

● Uma boa forma de se preparar é treinar a fala e os recursos de


comunicação, com o objetivo de conseguir uma fala fluida e confiante,

161
14ª edição do Programa de Preparação - UFRJ Consulting Club Apostila

que não pareça ensaiada no dia da entrevista. Entretanto, embora seja


importante ter respostas pensadas, é fundamental não repeti-las de
maneira robótica, pois pode passar ao entrevistador a sensação de que as
mesmas foram decoradas. Ressaltando também a importância de treinar
com amigos para analisar os vícios de linguagem.

● Assistir vídeos de entrevistas reais é um bom jeito de entender mais sobre


o contexto da situação e como funciona a dinâmica entre candidato e
entrevistador.

● Por fim, a prática de pedir feedbacks de amigos e colegas, treinar com


membros da equipe do Programa de Preparação e participar de eventos
de consultoria são estratégias essenciais.

4.9. Lista de perguntas

4.9.1. Exemplos para praticar

● "Conte-me sobre você."

● "O que você está fazendo aqui?"

● "Por que consultoria?"

● "Por que você escolheu sua universidade?"

● "O que você acha que os consultores fazem?"

● "O que você sabe sobre esse trabalho e sobre a nossa consultoria?"

● "Como são suas habilidades quantitativas?"

● "Conte-me um momento em que você demonstrou característica de


liderança."

● "Conte-me um momento em que você incentivou e influenciou um


grupo."

● "Conte-me sobre uma crise recente que você tenha enfrentado."

● "Você alguma vez já falhou em algo?"

● "Conte-me um momento em que você teve a iniciativa de começar algo."

162
14ª edição do Programa de Preparação - UFRJ Consulting Club Apostila

● "Qual o tipo de trabalho que mais te agrada?"

● "Em quais outras consultorias você está fazendo processo seletivo?"

● "Em quais outros ramos você está fazendo processo seletivo?"

● "Qual realização te deu uma maior sensação de satisfação?"

● "Quais características suas você acha que mais vão agregar a nossa
consultoria?"

● "Por que eu devo te contratar?"

● "Deseja me fazer alguma pergunta?"

4.9.2. As perguntas mais difíceis

● Você acha que já falhou em algo alguma vez? Diga sim! Todo mundo já
falhou em algo. Pessoas cometem erros o tempo todo e é dessa forma
que o aprendizado ocorre. Fale sobre quais erros você cometeu e o que
conseguiu tirar deles. Ainda melhor, fale como você falhou, o que
aprendeu desse erro e, depois, como reverteu isso com sucesso. Um ótimo
exemplo é o de Oprah Winfrey, que foi despedida do programa de notícias
que apresentava por ser “inapta para a televisão”. Anos depois, Oprah
começaria seu próprio programa na TV, que foi ao ar por 25 temporadas e
a garantiu um patrimônio líquido de quase U$3 bilhões. Tenha uma
história para contar e torne-a memorável.
● Em quais outras consultorias você está fazendo processo seletivo? É
normal dizer ao entrevistador que você está fazendo entrevistas com
outras consultorias. Caso isso não fosse verdade, não seria muito
inteligente depositar toda a energia em apenas uma consultoria.
Entretanto, deve-se deixar bem claro que aquela consultoria é sua
primeira opção.
● Em quais outros ramos você está fazendo processo seletivo? As
consultorias andam lado a lado com dois outros ramos. Enquanto estiver
fazendo uma entrevista em uma consultoria, é aceitável falar que você
também está procurando oportunidades em bancos de investimento ou
empresas de planejamento estratégico. Tais empregos exigem

163
14ª edição do Programa de Preparação - UFRJ Consulting Club Apostila

características e qualidades semelhantes.

5. Case Interview

5.1. Introdução

5.1.1. O que é a Case Interview?

A Case Interview é uma etapa de avaliação utilizada por consultorias em seus


processos seletivos, na qual é apresentada ao candidato uma situação de
negócios desafiadora, geralmente baseada em um projeto real realizado pela
firma mas em menor escala. Dessa forma, o entrevistado precisa entender o
contexto e o problema, estruturando uma maneira de resolvê-lo conforme o
objetivo estabelecido pelo cliente. Com isso, para a resolução, é preciso priorizar e
solicitar dados, analisar as informações concedidas, tirar insights e, por fim, dar
uma recomendação final do cenário apresentado.

5.1.2. Tipos de Aplicação

As entrevistas de case podem ser aplicadas em 2 estilos: candidate led e


interviewer led.

● Candidate Led (ou Interviewee Led)

É a forma mais comum de aplicação de cases. Nela, é avaliada a habilidade


do candidato para liderar o case, deixando que o entrevistador assuma
um papel secundário, no qual apenas fornece informações e, em
determinados momentos, direciona o candidato ao caminho correto, caso
ele desvirtue muito do esperado.

Esse estilo de aplicação é muito comum na Bain, BCG, Visagio e L.E.K,


enquanto não ocorre na McKinsey. Já na Mirow & Co, esse estilo é
diretamente dependente do entrevistador e de como ele busca avaliar o
candidato, podendo oscilar entre candidate led e a próxima forma a ser
abordada, a interviewer led.

164
14ª edição do Programa de Preparação - UFRJ Consulting Club Apostila

● Interviewer Led

São cases com mais perguntas pré-programadas para serem feitas pelo
entrevistador fora do que é pedido na apresentação inicial do case, de
modo que são compostos por várias partes pré-definidas. O método de
resolução não difere muito do candidate led, uma vez que o problema vai
ser fornecido e o candidato deve estruturar e ser o mais específico possível.
Como um cuidado especial nesse tipo de condução de case, é ideal que o
entrevistado continue exercendo o papel de líder com os devidos limites e
demonstrando iniciativa, o que, muitas vezes, é esquecido, já que há uma
condução mais ativa do entrevistador.

Esse tipo de aplicação é realizada em entrevistas da McKinsey, sendo o


principal diferencial das outras grandes firmas. Contudo, é possível,
também, que seja aplicado em outras companhias. Como um exemplo,
podemos ter:

“Um cliente abrirá um restaurante e contratou você para entender a


melhor estratégia para isso”

Nesse case, o entrevistador terá diversos dados sobre diferentes áreas de


atuação de um restaurante, como: especialidade, zona de instalação,
logística de custos e preços recomendados, que serão fornecidos
mediante às perguntas e organização do próprio candidato como parte do
case para a formação de uma recomendação geral. Entrando nessas
perguntas pré-programadas do exemplo, elas seriam exemplificadas
como:

○ Primeiramente, o cliente quer saber a especialidade do


restaurante. Que informações você considera relevantes para essa
decisão?

○ Agora, o cliente quer decidir qual zona é a melhor para a


instalação do restaurante. Que informação ajudaria você a
estruturar e sustentar sua recomendação para o cliente?

165
14ª edição do Programa de Preparação - UFRJ Consulting Club Apostila

○ Quais custos você imagina que um restaurante teria, além de


aluguel?

○ Agora que você selecionou a melhor especialidade e localização


para o negócio, o cliente gostaria de saber quantas pessoas
visitariam o restaurante por dia.

○ Por último, o cliente gostaria que você estimasse a receita e o lucro


que o restaurante ganharia por ano e, em seguida, fizesse uma
recomendação sobre abrir o negócio ou não. O cliente também
gostaria que você abordasse as principais decisões tomadas
durante o case.

Por fim, é importante frisar que os tipos podem se misturar um pouco


dependendo do estilo de entrevistador. Logo, essa divisão de Candidate e
Interviewer led não impede que, em um case Candidate Led, o aplicador faça
alguma pergunta, independentemente do candidato estar no caminho certo.
Nada impede também que, em um case Interviewer Led, o entrevistador deixe o
candidato ter mais iniciativa e vá analisando os pontos das perguntas sem que
elas sejam ativamente feitas a ele.

5.1.3. Habilidades Requeridas

Nessa etapa de entrevista, a firma busca por algumas habilidades nos


candidatos presentes no processo. São elas:

● Business sense

Esse conceito refere-se à capacidade do candidato de trazer


conhecimento sobre o mundo de negócios ou noções externas em geral
para agregar na resolução do case. Um exemplo seria, em um case sobre
lucratividade, demonstrar o entendimento de como é estruturada a
geração de lucro de uma empresa, divisão em custos fixos e variáveis e
análise de receita, e agregar esse conhecimento na resolução. Outra
habilidade analisada é o conhecimento sobre atualidades do mundo de
negócios e a capacidade de utilizar essas informações para elaborar

166
14ª edição do Programa de Preparação - UFRJ Consulting Club Apostila

insights mais realistas e contextualizados. Além disso, é interessante saber


conceitos comuns do mundo de negócios, como market size, market
share, break even.

● Problem solving

Essa habilidade se refere a como o entrevistado reage ao problema


apresentado inicialmente, assim como à sua capacidade de estruturação
ao longo do case, seja com nas perguntas realizadas, análises, cálculos,
recomendação final, entre outros. Tendo uma problemática grande para
ser resolvida, o ideal é segmentá-la em etapas para melhor organização do
pensamento e estruturação de ideias lógicas e lineares.

Esse ponto ajuda a simplificar o processo de problem solving, evitando,


assim, que o candidato fique sobrecarregado de ideias ou até não saiba
por onde começar a resolver o case. Essa habilidade é muito requisitada
pelo fato de, nas grandes empresas de consultoria, os projetos serem, em
sua maioria, muito longos e complexos, necessitando de uma organização
bem pensada e coerente para chegar a uma resposta que faça sentido.

● Raciocínio Lógico e Analítico

Esse ponto é referente à explicação lógica do porquê de cada ação ou


escolha tomada dentro de uma Case Interview, e se relaciona muito com a
habilidade citada anteriormente. Sendo assim, o entrevistado, primeiro,
estruturaria sua linha de pensamento, e, em seguida, explicaria toda a
lógica presente no seu raciocínio, assim como o fundamento para pedir os
dados do problema em questão.

O principal objetivo dessa análise é identificar se o entrevistado pensa


como um consultor, relacionando a linha de raciocínio desenvolvida por
ele com o entendimento da necessidade de cada informação sobre o
cenário abordado.

● Estruturação

Consiste em organizar a linha de pensamento, de forma a abranger todos


os pontos relevantes para o problema, de forma MECE, “Mutuamente

167
14ª edição do Programa de Preparação - UFRJ Consulting Club Apostila

Exclusivo e Coletivamente Exaustivo”. Dessa forma, busca-se abordar


todos esses pontos sem que eles se repitam ou se entrelacem. Essa
estruturação pode ser feita de diversas maneiras, incluindo o uso de
frameworks que podem ser adaptados a cada problema. O mais
importante é que o candidato consiga pensar em uma espécie de mapa
mental sobre os pontos que deseja abordar e siga essa organização
durante a resolução. Além disso, é importante ser top-down, partindo
sempre do aspecto mais geral e então especificar o raciocínio,
apresentando a análise feita como a primeira coisa a ser falada.

● Comunicação

Busca analisar se o entrevistado consegue transmitir suas ideias e


pensamentos de forma clara e organizada, de modo que o entrevistador
entenda e acompanhe seu raciocínio com exatidão. A comunicação é
fundamental, pois analisa se você, no papel de um consultor, conseguiria
passar todas as suas ideias e pensamentos à sua equipe e aos seus clientes
de forma objetiva, filtrando o que é relevante. Além disso, a comunicação
sinaliza a flexibilidade do candidato no sentido de conseguir reformular
suas ideias conforme a realidade do case, de modo que é uma habilidade
essencial para um consultor e muito avaliada na Case Interview.

Durante a entrevista de case, é extremamente recomendado que o


candidato comunique toda sua linha de raciocínio, incluindo pensamentos
óbvios e não-óbvios, contas básicas e hipóteses. É por meio disso que o
entrevistador consegue coletar todas as habilidades que o entrevistado
demonstra através da comunicação, seja ela o raciocínio lógico ou mesmo
a capacidade de se estruturar diante a uma situação abrangente.

5.1.4. Categorias de case

Existem diversos tipos de case, que abordam diferentes situações as quais


devem ser encontradas partindo de uma problemática. Também há cases que
envolvem mais de uma categoria, combinando duas análises ao mesmo tempo
ou trabalhando uma de cada vez, em etapas segmentadas do case. São eles:

168
14ª edição do Programa de Preparação - UFRJ Consulting Club Apostila

● Profitability

Trata dos assuntos de receitas, custos e lucros da companhia tratada no


case. Esse tipo de case pode ser direcionado para qualquer indústria, já
que o lucro é amplamente utilizado para analisar a situação financeira de
um negócio.

Exemplo: ”O cliente possui uma padaria, contudo suas vendas


vêm caindo nos últimos anos. Em contrapartida, outras padarias
similares estão em funcionamento normal. Sendo assim, o diretor
desse negócio quer entender a razão pela qual a receita caiu nos
últimos anos.”

● Market Sizing

Busca mensurar o tamanho de mercado em unidades ou em vendas ($).


Em geral, é dada mais ênfase ao processo de estruturação do que à
resposta final.

Exemplo: “Uma empresa europeia de materiais escolares quer


entrar no Brasil, mais especificamente no mercado de canetas.
Para isso, o cliente quer saber qual é o tamanho desse mercado no
país.”

● Market Entry

Aborda um cenário no qual uma empresa está interessada em entrar em


um novo mercado. Nesse tipo de case, espera-se que o candidato analise
o mercado, concorrentes e um possível retorno financeiro da empresa para
entender se essa entrada é ou não vantajosa para o cliente, tendo,
normalmente, objetivos e métricas que guiam essa decisão final e que,
geralmente, devem ser pedidos pelos candidatos.

Exemplo: “O seu cliente é uma grande indústria de refrigerantes


que atua em escala global e está considerando se expandir para o
mercado de bebidas alcoólicas. Nesse sentido, ele gostaria de
entender se ele deveria ou não entrar nesse novo mercado.”

169
14ª edição do Programa de Preparação - UFRJ Consulting Club Apostila

● M&A2

Trata da aquisição de uma empresa pela outra ou da fusão entre duas


companhias, de modo a explorar aspectos quantitativos (geralmente
financeiros) e qualitativos (relacionados a sinergias3) para chegar à
recomendação de fazer ou não a fusão ou a aquisição.

Exemplo: “O seu cliente é o CEO de uma empresa que produz


roupas casuais em um país com um grande litoral e clima quente.
Visando uma expansão geográfica, essa empresa estuda a
possibilidade de fazer uma fusão com a empresa X, a qual produz
roupas sociais em um país de condições climáticas semelhantes,
porém muito distante da primeira companhia. Com isso, o cliente
pediu que você recomende se ele deveria ou não se fundir com a
empresa X.”

● Crescimento

Procura analisar e traçar estratégias para fazer a empresa aumentar sua


receita, explorando aspectos orgânicos (inerentes somente à estrutura
interna da empresa ou ao crescimento natural do mercado na qual ela se
insere) e inorgânicos (M&As), bem como diferentes caminhos que auxiliam
nessa expansão do negócio.

Exemplo: Seu cliente é uma empresa de refrigerantes, que está


procurando por novas oportunidades depois de ter vários anos de
crescimento baixo. Para mudar esse panorama, o CEO da
empresa procurou você com a intenção de traçar estratégias para
a empresa crescer da melhor forma possível.

● Investment Go / No-Go

2
M&A: Sigla em inglês para Mergers and Acquisitions, traduzindo para o português: Fusões e Aquisições.
São operações que unem empresas, nas quais ambas as partes se juntam para conquistar mais clientes,
resultados, crescimento, entre outras métricas.

3
Sinergia: Trata da compatibilidade de duas empresas em determinado ponto, podendo ser uma sinergia
cultural, administrativa, de custos, financeira, entre outras. Quando uma sinergia é positiva, obtemos um
resultado maior que os resultados das empresas separadas ao agregarmos as duas partes, seja pela fusão
ou pela aquisição.

170
14ª edição do Programa de Preparação - UFRJ Consulting Club Apostila

Busca analisar se vale a pena realizar determinado investimento,


tomando como pontos da análise fatores qualitativos e quantitativos
fornecidos. Esses investimentos podem ser, por exemplo, entrar em um
novo mercado, começar uma nova linha de produtos, vender um mesmo
produto para outro segmento de clientes, entre outras abordagens.

Exemplo: Seu cliente é a Bevam, uma empresa global produtora


de bebidas alcóolicas, que, recentemente, vem considerando
adquirir uma nova planta industrial no Canadá. Com isso, o CEO
da Bevam contratou você para analisar se ele deveria ou não
investir capital da empresa nessa nova planta.

● Precificação

Tem como objetivo definir quanto o cliente deve cobrar/pagar por algum
produto, utilizando fatores quantitativos ou qualitativos do cenário que
sejam pertinentes à atuação da empresa em questão. Geralmente, cases
de precificação seguem uma estrutura bastante simular aos cases de
profitability e se guiam por métricas como payback4, breakeven5 e
margem de lucro6 para determinar, com base nos custos e na demanda,
quanto deveria ser um ticket médio ideal. Além disso, eles podem envolver
alguma análise de preços de concorrentes para determinar um intervalo
em que o ticket médio estabelecido seja competitivo.

Exemplo: Seu cliente está lançando um novo restaurante casual,


que oferece saladas e tigelas de grãos. Seu principal objetivo é
diferenciar a marca com base em uma mensagem de
responsabilidade social, enfatizando a gestão ambiental e as
condições de pagamento e trabalho dos funcionários. Este modelo,
claro, carrega custos com insumos mais altos do que seus
concorrentes. Quanto eles devem cobrar dos clientes por cada

4
Payback: Indicador que representa o tempo de retorno de um investimento.
5
Breakeven: Ponto em que não há prejuízos mas também não há lucros, ou seja, os custos se igualam a
sua receita e qualquer faturamento acima desse ponto pode ser considerado lucro.
6
Margem de lucro: diferença entre o faturamento com a venda de um produto ou serviço e os custos de
produção.

171
14ª edição do Programa de Preparação - UFRJ Consulting Club Apostila

refeição que vendem para que o cliente obtenha o payback no


ano 3?

● Redução de Custos

Procura, através de uma análise detalhada dos custos da companhia,


achar custos que podem ser modificados ou reduzidos para atingir o
objetivo requerido. Entre esses custos, podem aparecer custos de logística,
manufatura, salários, custos de fixação em um local, entre outros.

Exemplo: O seu cliente, LeGroup, produz acessórios coloridos


plásticos que podem ser conectados para a geração de outras
partes, e sua venda ocorre para clientes empresariais e
residenciais da Europa. Em um recente balanço financeiro,
notou-se que o custo de transporte, nos últimos anos, subiu muito
mais, relativamente ao custo geral, e está prejudicando o lucro da
empresa. Você foi contratado para determinar uma razão para
esses custos e recomendar o que deve ser feito para diminuir esses
custos alarmantes.

5.2. Resolução de um case


Para facilitar a estruturação do pensamento e uma resolução mais organizada do case,
podemos dividi-lo em 4 etapas: início, estruturação, desenvolvimento e recomendação.
Cada uma será abordada em mais detalhes a seguir.

5.2.1. Início

Esse será o momento que o entrevistador apresenta o problem statement


(caracterização por alto da empresa, passa brevemente por seu estado atual e
determina o que o candidato deve responder, ou seja, qual é o foco do case) para
o candidato. Como boas práticas, é importante que o candidato siga os seguintes
passos:

1. Anotação e entendimento

172
14ª edição do Programa de Preparação - UFRJ Consulting Club Apostila

O primeiro passo é anotar as palavras chaves de todas as informações


abordadas dentro do problem statement para ter a situação escrita e
poder revisitar sempre que preciso, lembrando a todo momento também
do objetivo principal ao longo do case. Vamos exemplificar esse ponto em
um case prático juntamente ao próximo tópico.

2. Validar o problem statement


Em seguida, mostrar que entendeu o problema é sempre fundamental
para fazer um case. Muitos candidatos acabam tendo uma ideia errada de
que validar informações seria algo ruim ou mal visto pelos aplicadores,
porém é justamente o contrário: é necessário se certificar que entendeu
tudo conforme é esperado no case e se comunicar sempre com o
entrevistador para evitar erros futuros. Com uma pequena falta de
atenção, informações podem passar despercebidas, e, assim, o case pode
se estruturar de forma totalmente diferente. Portanto, é essencial a
validação de todos os pontos principais da problemática, garantindo o
entendimento completo, e, caso não tenha sido possível anotar todas as
informações relevantes, é necessário pedir, logo nesse momento, que o
entrevistador repita a parte em específico em que o dado foi abordado.

Essa validação pode ser feita recapitulando todos os pontos chave para o
entendimento do cenário, porém com o uso de palavras diferentes das do
entrevistador, na medida do possível. Nessa validação do problem
statement, pode ser interessante que o candidato já tire conclusões
simples, como, em um case no qual a empresa já tem anos de história,
dizer que a marca já é bem consolidada. Caso o case não siga essa lógica,
o entrevistador pode invalidar o que foi dito, direcionando o candidato
sem nenhum problema.

Para entender melhor como seria a validação na prática, vamos observar


um exemplo, que será utilizado ao longo de toda a explicação.

Exemplo: Case Autobus S.r.l

173
14ª edição do Programa de Preparação - UFRJ Consulting Club Apostila

“Um amigo seu quer abrir uma empresa de ônibus que teria dois
veículos fazendo um trajeto entre as cidades de Padova e Florence,
e gostaria de saber qual é um ticket médio ideal para a empresa
operar.”

Após ler o enunciado, podemos marcar as palavras-chave do


exemplo acima como: Amigo, empresa de ônibus, 2 veículos, 1
trajeto (Padova - Florence), ticket médio ideal.

Então, para validar essa problemática, o candidato pode resumir todos os


pontos que ele anotou como importantes e repetir para o entrevistador, de
modo a validar se entendeu, de fato, a situação apresentada:

“Bom, então meu amigo vai abrir uma empresa de ônibus, na qual
ele operaria com somente um trajeto feito por 2 veículos, e quer
que eu recomende a ele qual seria um ticket médio ideal para as
passagens. Seria isso?”

3. Fazer perguntas esclarecedoras


Após validar, é importante realizar cerca de 3 perguntas esclarecedoras
para entender melhor o contexto do case e, assim, auxiliar na sua
estruturação de resolução - isso também evita que o candidato siga por
caminhos errados por falta de conhecimento do problema. As perguntas
também ajudam na linearidade e clareza do raciocínio, podendo evitar
idas e voltas em assuntos diferentes, o que pode afetar a estruturação e
organização. Quando o candidato percebe que não possui uma
informação e esqueceu de perguntá-la no início, ele precisa voltar para um
raciocínio já concluído, o que não é ideal.

Essa etapa também é interessante para pensar além do que o


entrevistador espera do candidato. Com a realização dessas perguntas, o
entrevistado irá obter respostas e entender mais ainda o objetivo e os
dados contidos no case, se mostrando, também, uma ótima oportunidade
de demonstrar business sense. Nesse sentido, é muito bem visto fazer
perguntas com um raciocínio de acordo com o escopo, lembrando sempre
de explicar a razão da necessidade de obtenção desses dados. Entre as
perguntas comuns de serem feitas nessa etapa, podem tratar sobre:

174
14ª edição do Programa de Preparação - UFRJ Consulting Club Apostila

● Crescimento do mercado;

● Situação econômica da empresa;

● Produtos vendidos;

● Segmentação de clientes;

● Mercado de atuação (Região, países);

● Clientes (B2B ou B2C, por exemplo).

Exemplo: Retomando o exemplo anterior do case Autobus,


podemos fazer algumas perguntas esclarecedoras como:

● Existe alguma segmentação em termos de assentos no


ônibus que influenciam em diferentes preços, como um
assento premium, ou um % de clientes que têm acesso a
passagens de ônibus gratuitas, como idosos, crianças ou
estudantes?

● Existe alguma diferença significativa em demanda, ocupação


ou quantidade de trajetos diários dos ônibus de acordo com
dias da semana ou eu poderia considerar valores médios?

● O ônibus do nosso cliente seria comprado, fazendo o


investimento de uma vez só, ou seria alugado, compondo um
custo?

Ao fazer as perguntas, é possível que o entrevistador não tenha respostas,


de modo que o aspecto pode ser desconsiderado do case. Ainda assim,
fazer uma pergunta que não tem resposta nem sempre é um problema:
muitas vezes, se for uma pergunta que, na realidade, seria pertinente para
um projeto, isso pode mostrar bastante business sense por parte do
candidato. Com as informações contidas no case, o candidato acharia:

● Todos os assentos do ônibus são idênticos, então o preço é o


mesmo para todos, e, ao analisar a taxa de ocupação de cada
viagem, já desconsideramos os clientes que não pagariam
pela passagem;

175
14ª edição do Programa de Preparação - UFRJ Consulting Club Apostila

● Os finais de semana têm menos clientes, mas nada que afete


significativamente a análise, então você pode considerar que
existe um valor médio;

● O ônibus seria alugado.

4. Buscar por um objetivo secundário


Essa etapa se relaciona muito com a de perguntas esclarecedoras e pode
vir ao final da anterior, não demandando, assim, mais tempo para
pensamentos. Nesse momento, o candidato deve buscar informações
sobre metas, métricas e restrições impostas pelo cliente, que, na maior
parte das vezes, não são comentadas no problem statement, apesar de
serem importantes na análise.

É fundamental que o entrevistado não pergunte diretamente pelo


objetivo secundário, pois isso mostra que você não está raciocinando de
acordo com o case, e decorou o formato de resolução. Por isso, sempre
busque ser específico para o problema, explicando sempre o porquê de
cada raciocínio ou exigência do dado julgado necessário. Por exemplo, em
um case que tem o objetivo de entender se o cliente deveria realizar um
investimento ou não, algumas possíveis perguntas seriam:

● Existe algum parâmetro ou métrica que sinalize se o investimento é


vantajoso para o cliente, como um valor de lucro anual?

● O cliente tem algum objetivo em termos de tempo de retorno do


investimento dele, como um payback ou tempo específico até o
breakeven?

● O que seria um negócio rentável para ele?

Vale lembrar que existem cases que não possuem um objetivo secundário
definido, mas, nos que possuem esse fator, desconsiderá-lo pode gerar um
erro grave na análise e alterar completamente a recomendação final.

Exemplo: Como uma abordagem para a pergunta que pode ser


feita no case analisado, podemos comentar sobre o que seria um
parâmetro para achar o ticket médio:

176
14ª edição do Programa de Preparação - UFRJ Consulting Club Apostila

● Para determinar o que seria o ticket médio ideal, esse amigo


tem em mente alguma métrica relacionada a, por exemplo,
lucro ou tempo de retorno do investimento?

● Ou obter lucro acima de um valor específico?

● Ou talvez apenas igualar os custos com a receita?

Para o caso do Autobus S.r.l, a métrica pré-definida pelo cliente seria


conhecer um ticket médio que possibilitasse que o breakeven da empresa
fosse de um ano. Desse modo, o preço estabelecido deve ser o suficiente
para que, no primeiro ano, a receita seja igual aos custos.

5.2.2. Estruturação

Esse é o momento em que será desenvolvida a estruturação do case para


quebrar a análise em partes mais específicas e facilitar o desenvolvimento da
questão. O ideal é pedir um tempo ao entrevistador para evitar um momento de
silêncio repentino, sem alinhamento prévio, e, também, para ajudar o candidato
a organizar seu pensamento sozinho, já que, nesse momento, surge uma grande
quantidade de ideias. Como um exemplo de como pedir tempo ao entrevistador,
podemos dizer:

“Bom, agora que entendi tudo sobre os objetivos, modelo de negócios e o


cenário geral do case, posso tirar 2 minutos para estruturar uma forma
de abordar o problema?”

Antes de explicar como fazer essa estruturação, é fundamental compreender os


conceitos de MECE e comunicação Top Down:

● Ser MECE - mutuamente exclusivo e coletivamente exaustivo - é fazer com


que, em uma lista, nenhum elemento se sobreponha e nenhum seja
omitido. Em outras palavras, diferentes partes não podem englobar o
mesmo aspecto e não pode haver nenhum aspecto relevante sendo
deixado de fora da análise. Aplicar esse conceito é muito necessário dentro
do case, pois faz com que o candidato seja organizado, fazendo análises
menores com um direcionamento correto, sem deixar nenhum ponto
relevante para o gabarito do case de lado.

Podemos exemplificar esse conceito com a seguinte figura:

177
14ª edição do Programa de Preparação - UFRJ Consulting Club Apostila

● Top Down, por sua vez, é uma estratégia de estruturação e comunicação


na qual o indivíduo busca começar pelo ponto mais abrangente e seguir
em direção aos pontos mais específicos e destrinchados. Em um livro, por
exemplo, ser top down seria começar falando o título, seguir para os
nomes dos capítulos, ir para os conteúdos das páginas e terminar com a
ideia de cada parágrafo. No case, essa forma de se comunicar e quebrar
uma ideia pode ser aplicada em qualquer momento, mas o principal deles
é na hora da estruturação.

Agora que já abordamos esses dois conceitos, podemos abordar dois aspectos
primordiais para uma estruturação eficiente da resolução do case:
brainstorming e issue tree.

● Brainstorming
Trata-se da geração de uma lista de hipóteses ou ideias lógicas de
forma estruturada e priorizada. Esse recurso se relaciona com o próximo
que citaremos, então é interessante aplicar o brainstorming na hora de
montar a issue tree.

Fora do momento de estruturação, vale ressaltar que também é possível


que o entrevistador, sobretudo em cases do estilo interviewer led, faça
Brainstorming Questions - perguntas que fazem parte do
desenvolvimento do case e que devem ser respondidas na hora pelo

178
14ª edição do Programa de Preparação - UFRJ Consulting Club Apostila

candidato. Porém, por enquanto, vamos focar em como o brainstorming


se aplica na parte de estruturação, que seria trazendo os pontos que o
candidato acredita que deveria analisar para que sua resolução seja
coletivamente exaustiva.

Continuando com o exemplo do case Autobus, com base no


problem statement, nas respostas para as perguntas
esclarecedoras e no objetivo secundário, sabemos que será
necessário calcular os custos e a receita do primeiro ano para
igualá-los, de modo a determinar o ticket médio, que será uma
incógnita nessa equação.

Na parte dos custos, o candidato pode fazer uma quebra em custos


fixos e custos variáveis, organizando esse brainstorming, e pensar
em listar o que se encaixaria em cada um, considerando uma
empresa de ônibus e lembrando que sabemos que o veículo é
alugado. Alguns exemplos para facilitar a análise desses custos são:

● Custos variáveis:

○ Custo anual do litro de gasolina

■ Km percorridos por litro de gasolina consumido;

■ Distância percorrida nos trajetos;

■ Quantidade de viagens que são realizadas por


ano;

○ Pedágios;

○ Taxas de vendas dos tickets.

● Custos fixos:

○ Salários e benefícios extras dos funcionários


(motoristas, administradores);

○ Aluguel dos ônibus;

○ Marketing para a disseminação da empresa;

○ Internet para os ônibus;

179
14ª edição do Programa de Preparação - UFRJ Consulting Club Apostila

○ Utilidades;

○ Seguro e manutenção dos veículos;

○ Imposto sobre os veículos.

Já na receita, ele pode pensar em uma forma de cálculo que faça


sentido para uma empresa que presta um serviço, multiplicando o
número de vezes que esse serviço é consumido por seu ticket
médio. O número em questão, por sua vez, pode ser ainda mais
destrinchado, englobando:

● Receita:

○ Capacidade do ônibus (número de assentos);

○ Taxa de ocupação;

○ Quantidade de viagens por dia;

○ Número de dias que a empresa operará no ano.

● Framework ideal - Issue Tree


Com os pontos de análise do case já pensados, é fundamental que o
candidato consiga organizá-los em uma estrutura MECE para apresentar
verbalmente ao entrevistador. Em grande parte dos cases, essa estrutura
tem a forma de uma árvore, com um ponto central (mais abrangente)
sendo destrinchado em ramos e sub-ramos (mais específicos) - por isso,
essa forma de estruturação recebe o nome de Issue Tree (em português,
árvore de problemas). Abaixo, é possível ver sua forma geral:

Além de permitir que o entrevistador compreenda melhor as análises a


serem feitas, as issue trees direcionam uma rota para a resolução do
problema: ao quebrar um problema grande em partes menores, é mais
fácil obter informações necessárias e fazer análises certeiras.

180
14ª edição do Programa de Preparação - UFRJ Consulting Club Apostila

Ao apresentar a issue tree, é importante ter uma comunicação top down


(de cima para baixo), começando do ponto mais central, seguindo para os
aspectos do primeiro nível de "galhos", até chegar aos mais específicos -
aqueles que estão nos "galhos" mais baixos.

Após essa apresentação, é imprescindível que o candidato faça 2 coisas:


valide sua análise, perguntando se algo não foi abordado ou pode ser
desconsiderado, e, em seguida, sob uma validação positiva do
entrevistador, indique por onde gostaria de começar e que dados quer
receber, de modo a entrar no desenvolvimento do case.

Vale ressaltar que o entrevistador, caso o candidato tenha fugido bastante


do esperado, pode direcionar o candidato para refazer sua estrutura. Nesse
caso, é essencial pensar fora do que foi apresentado para procurar outros
pontos pertinentes.

Dando prosseguimento ao exemplo do case Autobus, ao montar a


issue tree, teríamos um ponto central (o breakeven point), dividido
em 2 grandes ramos: os custos e a receita. Pegando os aspectos
mais específicos de cada um desses ramos e organizando eles de
forma MECE, poderíamos chegar à seguinte issue tree:

181
14ª edição do Programa de Preparação - UFRJ Consulting Club Apostila

Começando pelo topo, segundo o objetivo secundário que encontramos, o


amigo do consultor deseja operar em breakeven no primeiro ano de operação,
ou seja, lucro igual a zero. Para isso, já que Receita - Custo = Lucro, teremos que
os custos vão se igualar com as receitas.

Com essa informação, acharíamos assim a quantidade de pessoas que viajaram


com a empresa por ano e, igualando o valor do ticket médio vezes essa
quantidade com os custos, teríamos o valor da passagem média para a empresa
operar em breakeven.

5.2.3. Desenvolvimento

É a seção onde se concentra a maior parte do case e onde são realizadas as


diversas análises quantitativas e qualitativas necessárias para sua resolução.
Como uma boa prática, é sempre ideal justificar todo o seu raciocínio lógico e
buscar trazer insights sobre os dados analisados, os quais devem ser pedidos ao
entrevistador, sobretudo nesse momento. Em relação ao ponto de pedir por
dados, é sempre importante deixar claro o porquê se quer determinada
informação, caso não tenha sido explicado ao abordar a issue tree.

● Análise Quantitativa
Na análise quantitativa, busca-se obter informações numéricas e tirar
insights a partir de cálculos matemáticos, utilizando os dados fornecidos.

182
14ª edição do Programa de Preparação - UFRJ Consulting Club Apostila

Em geral, as contas envolvem uma matemática bastante básica, podendo


ser feitas rapidamente. Durante a entrevista, é primordial fazê-las sem o
uso de calculadora, por isso é essencial que o candidato treine a resolução
de cálculos.

Para uma análise quantitativa eficiente, é importante pensar em 6 passos:

1. Estruturar as contas de forma top down, pegando o que se quer


calcular e quebrando isso em partes menores.

2. Comunicar essa estrutura ao entrevistador, validando a forma


como serão feitas as contas. Como um diferencial, você pode
também elencar problemas que não foram mencionados, mas
poderiam afetar essa análise.

3. Solicitar os dados necessários conforme a estrutura montada e a


validação do entrevistador.

4. Realizar as contas, sempre explicando o que está sendo feito (o


que está sendo multiplicado pelo que, somado ao que, entre outras
operações, e quais são os resultados sendo obtidos nisso, passo a
passo). Nessa etapa, é fundamental achar, a partir do treino, o ponto
ideal entre precisão e eficiência, sempre em contato com o
entrevistador para validar se é permitido fazer aproximações.

5. Avaliar o valor obtido para entender se ele está correto e é


plausível. Para isso, é recomendado comparar o número achado
com um número da realidade para ver se realmente o valor
encontrado faz sentido, podendo, assim, seguir para o restante do
case. Por exemplo, se o candidato achar um valor unitário para um
chiclete de $100, é bem provável que tenha algo de errado. Se não
houver nenhum parâmetro estabelecido e o candidato não
conhecer plenamente o mercado e seus preços, pedir por essa
informação a caráter de comparação para o entrevistador é muito
válido.

6. Tirar o insight que o dado numérico gera, traçando os próximos


passos dentro do case (outra análise) ou chegando a uma
recomendação final. Em um case de growth, poderíamos calcular a

183
14ª edição do Programa de Preparação - UFRJ Consulting Club Apostila

receita que o cliente obteria caso lançasse um novo produto, e, ao


comparar o valor obtido ao valor tido como meta de crescimento da
empresa, recomendar que seja analisada outra estratégia de
crescimento que complemente essa, já que ela não é suficiente.

A comunicação de todos os passos é primordial para identificar


rapidamente qualquer pequeno erro que possa afetar o resultado final,
além de passar com clareza a linha de raciocínio seguida.

Se você, por acaso, acabar errando algum cálculo, não se desespere, pois
esses erros são comuns e não têm caráter eliminatório. O que não é ideal é
errar muitas vezes, sobretudo apresentando um resultado final errado
sem ter comunicado ponto por ponto com o entrevistador.

No case da Autobus, as contas já foram destrinchadas na própria


issue tree, visto que já é um case muito mais quantitativo. Com os
primeiros passos concluídos e os dados passados pelo
entrevistador, o candidato poderia começar a fazer suas contas. Ao
final, seguindo o que foi apresentado anteriormente ao
entrevistador, ele obteria o valor ideal para o ticket médio, conforme
os objetivos do cliente.

Começando pelos custos, teríamos as informações sobre:

● Combustível

○ 1,2 euros por Litro;

○ 25L consumidos a cada 100km percorridos;

○ 200km por viagem;

○ 1.440 viagens por ano.

● Marketing

○ 5.000 euros mensais, totalizando 60.000 ao ano.

● Salários

○ 4 motoristas, recebendo cada um 65.000 por ano;

○ 2 administradores, recebendo 50.000 cada por ano;

184
14ª edição do Programa de Preparação - UFRJ Consulting Club Apostila

○ 1 gerente, recebendo 100.000 por ano.

● Aluguel

○ 6.000 por ônibus por mês, sendo assim 72.000 por ano.

● Manutenção

○ 1.000 por mês, totalizando 12.000 ao ano.

● Utilidades

○ 2.000 por mês, totalizando 24.000 ao ano.

Ao todo, seria obtido um custo de 786.400 euros ao ano.

Passando agora para a parte da receita, teríamos as informações de:

● Quantidade de passageiros por ano

○ Viagens por ano

■ Número de viagens por dia já obtido no problem


statement, de 4 viagens diárias;

■ Número de dias que a empresa operaria por ano,


de 360 dias.

○ Pessoas por viagem

■ Capacidade de 40 assentos;

■ Utilização média de 80% desses lugares.

Com isso, teríamos agora também a receita em função do ticket


médio, sendo ela o número de passageiros x ticket médio (46.080 x
TM), como foi visto na issue tree citada no exemplo.

Por fim, para obter o valor aproximado do ticket médio para a


empresa operar em breakeven no primeiro ano, podemos igualar
os custos com a receita, resultando em uma equação de apenas
uma incógnita que pode ser resolvida:

Custo = Receita

786.400 = 46.080 x Ticket Médio

Ticket Médio = ~17 euros

185
14ª edição do Programa de Preparação - UFRJ Consulting Club Apostila

● Análise Qualitativa

A análise qualitativa baseia-se mais em tudo que não envolve cálculos,


passando por aspectos como concorrentes, sinergias, consumidores,
produtos, entre outros. Usando as sinergias como exemplo, poderíamos
analisar as culturas das empresas envolvidas em um M&A para entender
se esse movimento será benéfico ou não. Apesar dessa análise parecer
muito abstrata, com uma maior frequência de treinos de case é possível
adquirir um business sense mais apurado, e, com isso, essa subjetividade e
pensamento mais distante deixam de ser um impeditivo ao elencar
pontos na análise.

No case da Autobus, não temos nenhuma análise qualitativa a ser feita,


dado o objetivo secundário de alcançar o breakeven. Supondo, no
entanto, que o pedido não fosse esse, mas sim obter lucro a um preço
competitivo no mercado, seria interessante que o candidato analisasse os
preços das empresas de ônibus concorrentes e entendesse quais
empresas atendem a um segmento de clientes similar ao que a Autobus
visa a atender.

Com isso, seria possível fazer perguntas e pedir um gráfico ou tabela com
esses preços, observando os valores dos concorrentes e voltando para uma
análise quantitativa para testar se um preço hipotético inferior ao preço
médio dos concorrentes daria lucro para a empresa. Assim, misturaríamos
uma análise qualitativa (em termos de competição) com uma
quantitativa.

Por fim, é importantíssimo que você valide com o consultor se já pode


prosseguir para uma análise mais destrinchada (síntese) da resolução, ou se ele
acredita que você deixou algum detalhe a passar.

5.2.4. Recomendação, Riscos e Próximos Passos

A recomendação é o último passo de uma resolução de case. Nela, é ideal trazer


a resposta de forma clara e fazer um resumo dos pontos principais levantados
ao longo da entrevista. Em seguida, é preciso analisar os riscos e próximos passos

186
14ª edição do Programa de Preparação - UFRJ Consulting Club Apostila

da análise feita, o que, por mais que não seja esperado do candidato, é muito
bem visto em uma finalização de case, sendo um excelente diferencial.

Nessa etapa, para cases com recomendações mais complexas em termos de


justificativas, riscos e próximos passos, é recomendado pedir um momento ao
entrevistador para estruturar completamente a sua recomendação de forma
organizada e clara. Para cases estritamente quantitativos, como por exemplo de
market size, caso o candidato se sinta confiante e tenha em mente os aspectos
que irá abordar além do resultado final, pedir um tempo pode não se fazer tão
necessário.

Note que aqui é o momento perfeito para mostrar business sense e criatividade,
então aproveite essa etapa final caso você não mostre muito disso ao longo da
resolução do case.

1. Recomendação

Em primeiro lugar, é importante já começar com a resposta do case para


ser o mais answers first7 possível e, em seguida, fazer um breve pitch8, cujo
tamanho varia de acordo com o conteúdo de cada case. Nesse momento,
o candidato não deve explicar todo o caminho que traçou para chegar a
essa recomendação, e sim citar cerca de 3 pontos de forma objetiva que
sirvam como embasamento para a resposta.

Entrando no exemplo usado constantemente, podemos dizer que, para


atingir o breakeven, é necessário ter um ticket médio de
aproximadamente 17 euros, tendo em vista todos os custos anuais da
operação e a quantidade de pessoas que viajam em um ano entre essas
duas cidades.

2. Riscos

Após a recomendação em si, é muito bem vista também a estruturação


de riscos referentes à própria análise feita, sendo também uma boa
prática evitar ser genérico. Nesse sentido, é importante focar nos dados
com maior probabilidade de estarem incorretos, ou os que, se errados,

7
Answers first: estratégia de comunicação que prioriza a conclusão como primeira parte da resposta.
8
Pitch: breve e objetiva apresentação na qual devem ser apresentadas somente as informações essenciais.

187
14ª edição do Programa de Preparação - UFRJ Consulting Club Apostila

fariam grande diferença na análise final. Vale procurar, também, gaps na


análise, ou seja, aspectos que deveriam ter sido considerados para uma
recomendação mais embasada, mas que não foram abordados no case
por falta de dados ou direcionamento do entrevistador.

Esses riscos podem ser referentes ao mercado do negócio, a pontos que


foram assumidos em análises qualitativas, a valores que foram otimistas
ou a movimentos que podem acontecer no futuro do case. Tudo isso
mostra bastante business sense, pois vem por iniciativa do candidato de ir
além do que o entrevistador apresentou. Mas cuidado, pois os riscos
sempre devem ter embasamento, assim como todas as outras escolhas e
estratégias abordadas na entrevista.

Como exemplos de riscos, é possível listar:

● Riscos econômicos, como uma crise repentina;

● Riscos políticos, como um conflito de interesse que pode prejudicar


na projeção da análise;

● Resultado não ser como esperado;

● Custos extras que foram desconsiderados;

● Mão de obra;

● Taxa de penetração do produto no mercado não se concretizar;

● Aumentos nos preços dos insumos por parte dos fornecedores;

● Perda de clientes por uma mudança na qualidade dos produtos;

● Competidores realizando o mesmo movimento estratégico da


empresa;

● Implementação da recomendação.

Voltando ao case do Autobus, podemos citar como possíveis riscos dessa


análise de custos e receitas:

● Uma taxa muito otimista de 80% de ocupação dos ônibus;

188
14ª edição do Programa de Preparação - UFRJ Consulting Club Apostila

● Uma volatilidade muito grande no preço da gasolina;

● Não foi feita uma análise dos competidores para entender se o


preço estabelecido é competitivo no mercado de modo a viabilizar
a previsão de demanda;

● Uma frota de dois ônibus pode não ser suficiente para atender toda
a demanda, gerando insatisfação de possíveis passageiros.

3. Próximos passos

Por último, a projeção dos próximos passos de um case também mostra


um business sense muito positivo para o candidato. Essa fase pode tratar
de como mitigar os riscos citados anteriormente, bem como delimitar
estratégias para tornar um negócio benéfico para o cliente. Assim
como nas etapas anteriores, é fundamental a argumentação do objetivo
de cada próximo passo a ser tomado.

Em um projeto real de consultoria, esses próximos passos são um


momento ideal para prospectar novos projetos com os clientes já
conhecidos. Um bom exemplo disso é a questão de já fazer uma parte
desse projeto, e, na conclusão desse, apontar fatores que foram
identificados como pontos que podem melhorar na empresa e potenciais
projetos naquela linha com o cliente. Isso é muito bem visto, uma vez que,
para as consultorias, é sempre bom vender mais projetos agregadores aos
clientes.

No case do Autobus, o candidato poderia finalizar com:

"Para terminar a recomendação, como próximos passos, acredito


ser importante incluir outros 3 fatores na análise, que podem
torná-la mais completa e precisa:

● Considerar uma mudança na quantidade das viagens por dia em


diferentes épocas do ano (sazonalidade);

● Estudar um possível aumento na frota de ônibus dada a


demanda, que pode influenciar no ticket médio definido;

189
14ª edição do Programa de Preparação - UFRJ Consulting Club Apostila

● Realizar um benchmarking para comparar o preço definido ao


preço de concorrentes que realizam o mesmo trajeto ou trajetos
parecidos, entendendo se o preço é ou não competitivo.”

5.3. Aprofundando o desenvolvimento

5.3.1. Introdução

Após entender o que é uma Case Interview e quais suas principais etapas, é
importante aprofundar em aspectos de seu desenvolvimento. Com isso, os
tópicos abordados nesta seção serão: brainstorming, frameworks, estimate e
interpretação de gráficos.

5.3.2. Brainstorming

O brainstorming é um ponto crucial na resolução de cases, estando, geralmente,


presente já nos momentos iniciais quando o candidato pensa em perguntas
esclarecedoras, e, também, aparecendo outras vezes ao longo da entrevista.

Para aprender a realizá-lo com eficiência, é preciso, antes, fixar o conceito de


hipóteses. Estas são suposições da causa do problema que serão validadas ou
invalidadas ao decorrer do case. Uma boa hipótese, portanto, deve ser bem
estruturada a partir de uma linha de raciocínio desenvolvida e condizente com o
case. Além disso, ao definir hipóteses, é importante que o candidato pense nas
causas daquele acontecimento, assim como possíveis consequências do mesmo.

De modo geral, um bom brainstorming consiste em uma lista estruturada e


priorizada de boas hipóteses.

Para que seja possível uma melhor compreensão de como esse processo deve
ser realizado, pode-se analisar um exemplo a partir da seguinte questão:

“A academia Rio Fit enfrentou uma queda drástica em sua receita. O que
pode ter acontecido?”

190
14ª edição do Programa de Preparação - UFRJ Consulting Club Apostila

Candidatos que não possuem prática nessa etapa tendem a apresentar uma
lista aleatória e não estruturada. Nesse sentido, um simples exemplo de
brainstorming mal estruturado, nesse caso, seria:

“Pode ser que a demanda tenha caído, ou que nossos preços tenham
sido reduzidos, ou até que tenhamos conquistado menos clientes novos.”

O problema, nesse raciocínio, não são as ideias, mas sim a organização ao


comunicá-las. Além disso, ao citar várias possibilidades sem uma análise mais
profunda ou uma ordem clara, não temos como ter certeza de que atingimos
todas as possibilidades e incluímos a hipótese correta, por isso a importância de
ser MECE.

Portanto, para realizar um brainstorming completo, é necessário seguir três


passos: definir e estruturar, priorizar os fatores e, por fim, gerar hipóteses:

1. Definir e estruturar:

Esse é um ponto definitivo para todo o case, e, nele, é crucial ser claro e se
atentar aos detalhes. Sempre que possível, busque transformar o
problema em uma métrica, ou seja, uma medida quantificável para
analisar o resultado de um processo, ação ou estratégia específica.
Quando isso não for possível, é importante deixar o problema mais
específico com palavras. Por exemplo, se o cliente deseja identificar qual
opção de produto é melhor, tente entender o que isso pode significar,
com termos como “mais rápido”, “mais forte”, “melhor custo-benefício”,
entre outros.

No exemplo citado, estamos trabalhando com a receita, que já é uma


métrica, mas mesmo assim, ainda podemos especificar mais. Algumas
perguntas que podem ser feitas são: Quanto a receita caiu? Em quanto
tempo? Como a receita é estruturada?

A partir dessas perguntas, o avaliador pode nos fornecer novas


informações relacionadas à situação inicial, que pode ser reformulado de
forma mais detalhada, como visto a seguir:

“A academia Rio Fit enfrentou uma queda de 35% em sua receita


em um período de 8 meses. O que pode ter acontecido?”

191
14ª edição do Programa de Preparação - UFRJ Consulting Club Apostila

Agora, é possível destrinchar o problema de forma MECE e estruturar


uma issue tree. Nessa etapa, é importante ser específico, de modo que
não é recomendado apenas decorar uma estrutura, e sim saber se adaptar
às especificidades do case. Vale ressaltar que é esperado que o candidato
explique a estrutura do pensamento para o avaliador, com uma
comunicação clara, mostrando como chegou aos fatores finais. Uma boa
forma de fazer isso é sendo top down, ou seja, saindo do ponto principal e
destrinchando o raciocínio até chegar em cada ponto mais específico.
Além disso, é interessante buscar uma validação, perguntando se novas
possibilidades devem ser analisadas.

2. Priorizar os fatores:

Ao concluir a primeira etapa, é preciso pensar quais são os pontos mais


importantes entre aqueles levantados. No que devemos focar em uma
primeira análise? Para isso, deve-se olhar para o contexto e os valores do
case, buscando responder perguntas como:

● Quais fatores têm maior impacto?

● Qual a probabilidade de cada um ter ocorrido?

Nesse momento, é importante também solicitar dados, de modo que o


avaliador não pense que você está os subestimando ou negando as outras
possibilidades ao priorizar apenas uma. Para isso, é fundamental olhar

192
14ª edição do Programa de Preparação - UFRJ Consulting Club Apostila

para todos os possíveis fatores levantados na primeira etapa e pensar em


quais informações poderiam ser utilizadas para validá-los. Um exemplo de
como o entrevistado poderia pedir dados é:

“Gostaria de dados em relação XX e YY para poder analisar a


possibilidade de ZZ ter ocorrido.”

No exemplo analisado, o candidato poderia explicar sua estrutura e pedir


informações a respeito das quatro alternativas. Além disso, seria
interessante mencionar que, caso não houvesse dados para isso, ele
poderia apresentar sua teoria de qual dos fatores deveria ser priorizado.
Portanto, se o avaliador pedisse que ele compartilhasse suas ideias, uma
boa colocação do candidato seria:

“Minha teoria é de que os clientes da academia estão deixando a


empresa com mais frequência. Isso porque, como uma queda de
35% na receita em 8 meses é um valor significativo para um curto
período de tempo, acredito que a redução das vendas de
equipamentos tenha menor probabilidade de ser o fator principal,
uma vez que não é a maior fonte de receita e, provavelmente, tem
um menor impacto. Já em relação à queda nas mensalidade, os
valores teriam que ter sido reduzidos de forma brusca, ou muitos
clientes precisariam ter mudado para um plano mais barato.
Porém, também acho que isso teria menor probabilidade de
ocorrer a ponto de gerar o resultado observado. Por último, sobre a
captação de novos clientes, mesmo que houvesse uma queda
muito grande e a nossa atuação em marketing não estivesse boa,
acredito que não seria o suficiente para uma perda de 35% nesses
meses, visto que ainda teríamos os nossos antigos clientes.
Contudo, mesmo que a taxa de entrada de novos clientes, vendas
de equipamentos e mensalidade se mantivessem iguais, a saída
de clientes com maior frequência seria capaz de ter grande
impacto na receita."

3. Gerar hipóteses:

193
14ª edição do Programa de Preparação - UFRJ Consulting Club Apostila

Por fim, o candidato deve elaborar e apresentar hipóteses ao avaliador.


Para isso, é essencial falar o óbvio, tendo em vista que, muitas vezes, a
solução se encontra exatamente entre as hipóteses mais comuns e
conhecidas. Contudo, um bom diferencial é ser criativo e pensar além,
mesmo que o case acabe não seguindo por essa linha depois. Por fim,
lembre-se sempre de ser prático, visto que não é relevante pensar em
uma alternativa inovadora se não é algo plausível.

Retornando ao exemplo, o candidato poderia dividir o aumento na saída


dos clientes em três novos fatores:

● Competição com outras academias;

● Preferência por outras formas de atividade física;

● Redução na quantidade de pessoas dispostas a pagar para realizar


exercícios.

A partir desses pontos, seria possível trazer hipóteses para cada um, como
realizado abaixo para o tópico de competição com outras academias:

● Surgiu um novo competidor que possui certa vantagem em relação


à Rio Fit, como:

○ Preços melhores;

○ Espaço mais agradável;

○ Equipamentos mais modernos;

○ Profissionais melhores.

● A Rio Fit passou por mudanças que afastaram os clientes, que


migraram para outras opções. Algumas alternativas seriam:

○ Aumento no preço;

○ Troca no horário das aulas;

○ Transferência para nova localização.

Esse processo deve se repetir para cada ponto em cada um dos quatro
fatores principais, a menos que o avaliador já tenha validado ou invalidado
algum deles no decorrer da entrevista.

194
14ª edição do Programa de Preparação - UFRJ Consulting Club Apostila

Outro ponto importante a ser abordado é como finalizar sua resposta. Para isso, o
candidato deve pensar em quais dados seriam necessários para validar cada
hipótese. Por exemplo, para validar a hipótese de que os clientes optaram por
um competidor com preços melhores, seria possível solicitar a tabela de preços
da Rio Fit e de seus concorrentes.

Além disso, é esperado que o candidato pense nos próximos passos do case.
Por exemplo, ao validar uma das hipóteses levantadas, entender se deve ser
realizado um novo brainstorming para compreender as possíveis causas e
soluções desse fato ou até se já existem informações suficientes para encerrar o
case e realizar uma recomendação.

5.3.3. Frameworks

Frameworks são, essencialmente, esquemas que ajudam a estruturar e guiar o


pensamento para conseguir entender e solucionar um problema de forma
efetiva e organizada. Sendo assim, eles funcionam como um mapa, indicando
por quais pontos você precisa passar e qual rota seguir dentro do case. Contudo,
é crucial lembrar que não se deve depender de fórmulas e frameworks
decorados, pois esses são facilmente identificados pelo avaliador e não existe um
único molde que sirva para todos os cases. O que é avaliado é sua capacidade
de adaptar os conhecimentos para diferentes situações.

Apesar de os frameworks serem mais associados a issue trees, eles nem sempre
possuem essa estrutura ramificada, podendo ser, por exemplo, matrizes, como
será abordado mais adiante.

Alguns dos frameworks mais utilizados são:

● Profitability (lucratividade):

De modo geral, esse framework pode ser resumido na equação:

Lucro = Receita - Custos

A partir disso, cada braço da equação é destrinchado, podendo chegar no


modelo observado abaixo:

195
14ª edição do Programa de Preparação - UFRJ Consulting Club Apostila

Alguns exemplos comuns de custos fixos, como mencionado


anteriormente, são aluguel e salários, enquanto os variáveis podem ser
matérias-primas e embalagens..

Além disso, a receita pode ser segmentada por:

○ Tipo de produto;

○ Região geográfica;

○ Perfil de consumidor.

Sendo assim, é necessário entender qual o contexto do case e quais dados


são fornecidos para resolvê-lo, de modo a adaptar o framework para cada
situação específica.

● Framework de crescimento:

Esse framework, assim como o de lucratividade, tem o formato de uma


issue tree. Sua divisão foca em diferentes estratégias de crescimento e,
normalmente, seu uso está atrelado a alguma meta financeira ou algo
similar.

196
14ª edição do Programa de Preparação - UFRJ Consulting Club Apostila

○ Crescimento orgânico: ocorre de forma gradativa e natural, por


meio das próprias capacidades e esforços internos da empresa, ou,
então, impulsionado pelo crescimento do mercado.

○ Crescimento inorgânico: ao invés de crescer de dentro para fora, a


empresa foca no crescimento de fora para dentro. Esse crescimento
é impulsionado por aquisições, joint ventures9 ou parcerias, tendo
burocracias e contratos atrelados a ele.

Assim como em todos os outros frameworks, este precisa ser adaptado


para o contexto do caso, portanto é preciso entender quais possibilidades
são viáveis e condizentes com a questão levantada, se é preciso

9
Cooperação ou associação econômica entre duas empresas de forma geral, por um período de tempo
determinado. As duas - não necessariamente do mesmo ramo - criam uma nova empresa ou um novo
projeto, mantendo suas identidades jurídicas, independência administrativa e econômica, geralmente com
um objetivo bem definido.

197
14ª edição do Programa de Preparação - UFRJ Consulting Club Apostila

destrinchar mais um dos braços da issue tree, se deveria ser adicionado


algum fator, entre outras possíveis mudanças.

Além disso, uma boa estratégia é, logo após apresentar a estruturação do


raciocínio, perguntar se o cliente consideraria um M&A, para descartar
rapidamente a opção caso não seja relevante.

● 3Cs - Companhia, Concorrentes e Consumidores:

Essa ferramenta é útil quando há necessidade de analisar uma variedade


de fatores, incluindo estratégias competitivas. Esse framework pode ser
utilizado em cases de criação de um negócio, entrada em um novo
mercado, desenvolvimento de produto, entre muitos outros que
necessitam uma visão ampla e diversificada. É possível dividir a análise
dos três Cs em dois fatores extremamente importantes do mercado:

○ Oferta:

■ Competidores: O que cada competidor pode fazer com suas


forças, fraquezas e recursos?

■ Companhia: O que nossa empresa pode oferecer com as


próprias forças, fraquezas e recursos?

○ Demanda:

■ Consumidores: Como consumidores diferentes se


comportam? O que eles valorizam, precisam ou desejam?

198
14ª edição do Programa de Preparação - UFRJ Consulting Club Apostila

Vale ressaltar que, dependendo da necessidade do case, pode-se


adicionar o tópico de “Produtos e serviços” na análise, assim como retirar
um dos Cs.

Algumas perguntas complementares que podem ser feitas para entender


cada um dos 3 C’s são os 5W (What, Where, Who, Why e When) e 2H (How
e How much):

Assim, o que devemos perguntar é:

○ O que? Qual?

○ Onde?

○ Quem?

○ Por que?

○ Quando?

○ Como?

○ Quanto?

Alguns exemplos de perguntas formuladas a partir desse método são:

○ Quais são as forças e fraquezas dos meus competidores?

○ Quem são meus principais clientes e como eu sirvo eles?

○ Onde e quando os meus consumidores compram? Porque?

● M&A:

Esse framework deve ser utilizado em casos de Mergers and Acquisitions,


ou seja, fusões - união de duas empresas para se tornar uma só
companhia - e aquisições - uma das empresas adquire completamente
ou parcialmente os recursos e ativos da outra empresa.

A estrutura do framework é muito similar aos 3 Cs apresentados


anteriormente, contudo a análise, neste caso, deve ser segmentada em
três cenários:

○ Empresa A sozinha;

○ Empresa B sozinha;

199
14ª edição do Programa de Preparação - UFRJ Consulting Club Apostila

○ Empresa A junto à empresa B.

Assim, é preciso pensar em diferentes questões para cada área do


framework, podendo entrar tanto aspectos qualitativos, quanto
quantitativos. A partir dessas análises, é possível, então, ter uma visão do
que faz mais sentido para cada empresa.

Um conceito muito importante nesse tipo de case é a sinergia, que


consiste no valor adicional que seria agregado a partir da operação de
fusão ou aquisição. Um exemplo de sinergia entre empresas seria a
possível melhora de métricas financeiras a partir da união de ambas.
Sendo assim, a análise deve buscar entender o que é gerado como
benefício em cada ponto do framework ao juntar as empresas, assim
como o que seria prejudicial da mesma ação.

Um M&A feito corretamente pode trazer diversos benefícios aos


envolvidos, que justificam o interesse de algumas companhias em se
unirem ou comprarem outra, como:

○ Aumentar o acesso de mercado;

■ Aumentar market share;

200
14ª edição do Programa de Preparação - UFRJ Consulting Club Apostila

■ Aumentar a lucratividade;

■ Estimular o crescimento da marca;

■ Entrar em um novo mercado;

○ Diversificar portfólio de produtos;

○ Incorporar sinergia;

■ Cooperação entre as empresas: marketing, finanças e


operações;

■ Redução de custos;

■ Expansão dos canais de distribuição;

○ Situação atual da empresa alvo;

■ Saúde financeira;

■ Eficiência da empresa.

5.3.4. Estimate

Em um case de estimate, o candidato deve realizar uma estimativa de algum


valor solicitado. Apesar de o mais famoso ser o de market size - estimar o
tamanho do mercado -, os cases desse tipo podem abordar diversos tópicos,
como o lucro de uma empresa ou a quantidade de vendas de um produto. O
foco principal nessas situações é não só estimar um valor próximo ao correto,
mas mostrar que você consegue estruturar o pensamento, fazer análises como
um consultor e ter um bom senso crítico.

Sendo assim, passos claros e definidos são a melhor forma de resolver esses
problemas, pois, além de demonstrar organização, mostram ao avaliador que
você entendeu o problema e sabe como pretende resolvê-lo.

Para solucionar os cases de estimate, é recomendado seguir cinco passos:

1. Clarificar o escopo:

Tenha certeza de que você entendeu o problema e sabe exatamente o


que está tentando estimar. Nesse momento é recomendado que sejam
realizadas perguntas esclarecedoras ao avaliador, por exemplo, se o case

201
14ª edição do Programa de Preparação - UFRJ Consulting Club Apostila

pedir que você estime a receita com vendas de ingresso para um festival,
possíveis perguntas são:

● Quantos dias esse festival possui?

● Qual a ocupação máxima do local onde ele ocorrerá?

● Qual o tipo de festival?

● Existe uma idade mínima para entrar?

● Existe alguma segmentação do público alvo?

● O preço dos ingressos varia por idade?

2. Estruturar o problema:

Após compreender o problema, pense no que você vai usar nessa


estimativa e como pretende utilizar cada informação. Não use números
ainda, apenas indique o que será feito. Voltando ao exemplo do festival,
caso fosse informado que o festival oferece meia-entrada para todos entre
16 e 21 anos ou com mais de 65, seria possível montar a seguinte issue tree:

Receita total com ingressos:

1. Receita com meia-entrada

● Quantidade de meia-entradas vendidas

○ Quantidade de pessoas com direito à meia-entrada no


país

■ População entre 16 e 21 anos

■ População maior que 65 anos

○ Porcentagem dessa parcela da população do país que


é esperada no evento em cada dia

○ Número de dias de evento

● Preço da meia-entrada

2. Receita com inteira

● Quantidade de inteiras vendidas

○ Quantidade de pessoas que pagam inteira no país

202
14ª edição do Programa de Preparação - UFRJ Consulting Club Apostila

■ População entre 19 e 64 anos

○ Porcentagem dessa parcela da população do país que


é esperada em cada dia

○ Número de dias de evento

● Preço da inteira

É importante ressaltar que existem diversas formas de obter o mesmo


resultado e, muitas vezes, a sua estrutura inicial pode precisar ser
adaptada para os dados que o avaliador irá oferecer. Nesse caso, o
entrevistador poderia não possuir a porcentagem de cada parcela da
população que seria esperada no evento, mas sim a divisão percentual dos
ingressos do evento entre meia-entrada e inteira, logo, o candidato deve
ser flexível. Uma alternativa seria, então, calcular a mesma receita a partir
da seguinte issue tree:

Receita total com ingressos:

1. Receita com meia-entrada

● Quantidade de meia-entradas vendidas

○ Lotação máxima do evento por dia

○ Taxa de ocupação por dia

○ Porcentagem de meia-entradas entre o total de


ingressos vendidos

○ Número de dias de evento

● Preço da meia-entrada

2. Receita com inteira

● Quantidade de inteiras vendidas

○ Lotação máxima do evento por dia

○ Taxa de ocupação por dia

○ Porcentagem de inteiras vendidas entre o total de


ingressos vendidos

○ Número de dias de evento

203
14ª edição do Programa de Preparação - UFRJ Consulting Club Apostila

● Preço da inteira

Caso o raciocínio apresentado seja plausível, mas não seja o caminho do


case, não será mal visto pelo avaliador. Sendo assim, é mais vantajoso que
o candidato pense em pontos que vão além da estrutura idealizada no
case do que não elencar pontos suficientes. O que é crucial nesse
momento é não insistir em uma questão que o entrevistador já descartou
ou em um dado que ele já informou ser irrelevante ou inexistente no
cenário.

3. Atribua valores às variáveis:

Com base no que você identificou como necessário para resolver o


problema, solicite os dados ao entrevistador. Caso ele peça que você
estime números para essas variáveis, tente pensar em números plausíveis,
comparando com outros valores que você conheça. É importante buscar
destrinchar a conta o máximo possível, visto que variáveis simples são
mais fáceis de estimar.

Por exemplo, supondo que, ao estruturar o exemplo do festival e, seguindo


a primeira issue tree apresentada, seja necessário segmentar a
quantidade de pessoas com direito à meia-entrada no país em:

● População total do país;


● Porcentagem de pessoas com menos de 18 anos ou mais de 65
anos.

Caso o festival seja nos Estados Unidos e o avaliador não forneça valor
referente à população, é possível utilizar a população brasileira como
parâmetro para fazer uma estimativa. Sendo assim, sabendo que o Brasil
possui cerca de 200 milhões de habitantes e que, apesar da população
dos dois estarem entre as 10 maiores, os EUA ainda estão na frente, seria
possível estimar uma população de 300 milhões. Vale lembrar que não é
indicado estimar valores que atrapalhem a conta futuramente, como 1273
e 19, devendo, sempre, buscar por números arredondados.

Além disso, se necessário, também seria possível estimar a porcentagem


da população que teria direito ao desconto de meia entrada e a

204
14ª edição do Programa de Preparação - UFRJ Consulting Club Apostila

porcentagem que teria que pagar inteira. Para isso, pode-se adotar uma
estimativa de vida de 80 anos e considerar a divisão entre as idades como
iguais, caso o entrevistador valide esse raciocínio. Assim, somando as 21
diferentes idades que teriam direito ao desconto e dividindo pelo total de
80 idades, seria possível chegar a uma porcentagem de 26,25%. Com isso,
os 73,75% restante da população pagariam o ingresso inteiro. Nesse
momento, caso o entrevistador permitisse, seria possível arredondar os
valores para 26% e 74% ou, então, 25% e 75%.

4. Calcular:

Essa etapa consiste em chegar em uma resposta. Realize suas contas,


sempre comunicando ao entrevistador o que está sendo feito para que ele
possa acompanhar e validar o raciocínio. Nesse momento, um pequeno
erro de conta ou outro não será tão mal visto pelo avaliador, pois o
principal a ser avaliado é o raciocínio, mas é importante buscar evitar esses
acontecimentos, principalmente a partir da prática.

Ao contrário do momento da estruturação, aqui é importante ser


bottom-up, começando dos dados mais destrinchados para chegar ao
ponto principal da issue tree. Além disso, também pode ser interessante
seguir as boas práticas de cálculos e análises quantitativas apresentadas
em módulos anteriores.

Voltando ao exemplo do festival, com as porcentagens arredondadas de


25% e 75%, junto com a população estimada de 300 milhões, podemos
obter os valores abaixo:

● População que paga meia-entrada: 300 milhões x 25% = 75 milhões

● População que paga inteira: 300 milhões x 75% = 225 milhões

Dando continuidade aos cálculos, supondo que o avaliador já havia


fornecido os seguintes dados:

● Porcentagem de cada parcela da população do país que é esperada


no evento em cada dia:
○ Pagam inteira: 0,005% → 15 mil ingressos

205
14ª edição do Programa de Preparação - UFRJ Consulting Club Apostila

○ Pagam meia: 0,003% → 9 mil ingressos

● Preço dos ingressos:


○ Inteira: U$90 → U$1.350.000 em ingressos
○ Meia: U$45 → U$450.000 em ingressos

● 3 dias de evento
○ 3 x (1.350.000 + 450.000) = U$1.755.000 em ingressos

5. Conferir a resposta:

Por fim, veja se o resultado obtido faz sentido, comparando-o com outros
dados ou outros valores conhecidos por você ou oferecidos ao longo do
case. Uma possibilidade seria comparar o mercado analisado com outro
similar, ou até mesmo com a realidade, caso o candidato possua esse
conhecimento, e validar isso com o entrevistador.

Nesse caso, pode-se buscar comparar a quantidade esperada de pessoas


por dia. Considerando que um festival grande - como o Rock in Rio - pode
ter uma média de 100 mil pessoas por dia, enquanto alguns festivais
menores têm capacidade para 10 mil. Dessa forma, cerca de 24 mil
pessoas é um valor plausível.

5.3.5. Interpretação de gráficos

Em alguns cases, as informações necessárias para a resolução podem ser


passadas para o candidato em formato de gráficos. Essa é uma ótima
oportunidade para se diferenciar de outros candidatos, pois é comum que as
pessoas não pratiquem esse tipo de análise e se assustem ao receber um gráfico
mais complexo ou não saibam como proceder.

Apesar dessas situações parecerem complexas, nelas, é como se alguém já


tivesse feito a análise, e agora fosse necessário apenas olhar os resultados,
interpretá-los e guiar o cliente a partir disso. Sendo assim, duas perguntas
importantes ao olhar qualquer gráfico são: “O que os dados significam?” e “O que
deve ser feito a partir disso?”.

206
14ª edição do Programa de Preparação - UFRJ Consulting Club Apostila

Para entender melhor o que é esperado na análise de gráficos, é importante


saber quais são os principais erros cometidos nesses momentos, assim como os
principais acertos e práticas que devem ser seguidas.

● Más práticas:

○ Não conectar os dados com o case e apontar somente o que está


sendo exposto. Com isso, a análise fica superficial, pois dizer que “A
empresa A tem lucro maior”, “A empresa C tem custos fixos mais
baixos” ou “O market size de B é maior que D” é algo que qualquer
universitário consegue fazer. O que é esperado nesse momento são
insights e conclusões relevantes ao case a partir dessas
informações.

○ Falar sem entender o gráfico, tentando adivinhar e realizar


análises sem ter propriedade ou certeza.

○ Não guiar quais devem ser as ações tomadas a partir da análise, ou


seja, quais são os próximos passos a partir das conclusões tiradas.

● Boas práticas:

○ Entender os dados e conseguir trabalhar com informações que


você nunca viu antes. Um consultor nunca vai tirar conclusões
precipitadas sem antes entender com o que ele está trabalhando,
então faça o mesmo. Se necessário, pergunte sobre as legendas, os
eixos, os números, qualquer coisa que você não entenda ou queira
confirmar. Muitos gráficos podem vir com informações em inglês,
então, caso não seja explicado, é importante que o candidato tire as
dúvidas que possam ter ficado.

○ Praticar diferentes cases com análises gráficas e aprender a lidar


com gráficos diversos. Isso mostra que o candidato está
acostumado a trabalhar com gráficos e é capaz de tirar insights
deles com facilidade. Sendo assim, é importante estar familiarizado
com os tipos de gráficos que mais aparecem.

207
14ª edição do Programa de Preparação - UFRJ Consulting Club Apostila

Além de seguir essas boas práticas, existem quatro passos rápidos que podem
ser aplicados para guiar o pensamento e alcançar um bom resultado ao
interpretar um gráfico:

1. Mapear o território:

Em um primeiro momento, o mais importante é olhar para o gráfico,


esquecendo os números. Isso significa olhar para os eixos e entender a
informação que está sendo dada. Com base nisso, busque responder
perguntas como: “O que o gráfico pode estar dizendo?”, “Qual informação
pode ser tirada dele?” e “O que depende desses dados?”.

2. Observar os detalhes:

Não tenha pressa em tirar conclusões, pois tudo está no gráfico por um
motivo. Desse modo, é essencial ler as observações, as legendas, etiquetas,
unidades, asteriscos, entre outros, além de fazer perguntas se necessário.
Não tenha medo! Consultores têm receio de falar sem ter um
embasamento correto e não de perguntar para entender melhor sobre o
assunto, então é esperado dos candidatos esse mesmo comportamento.

3. Ler os dados:

Somente nessa etapa deve-se tirar conclusões numéricas dos dados,


ainda que de forma mais básica e rasa. Quando possível, é interessante
listar as conclusões e sintetizá-las de modo organizado. Além disso, é
fundamental mencionar primeiro as informações mais importantes - as
principais mensagens passadas pelo gráfico - e depois seguir para análises
menores e mais específicas.

4. Interpretar os dados:

Por fim, busque conectar cada um dos insights com a problemática


principal do case. A partir disso, deve-se pensar em próximos passos, ou
seja, o que precisa ser feito em seguida, como, por exemplo, seguir para
outra análise ou encerrar o case com a recomendação do que o cliente
deve fazer.

208
14ª edição do Programa de Preparação - UFRJ Consulting Club Apostila

Alguns exemplos de gráficos que podem ser encontrados em diferentes cases


são:

● Gráfico de posicionamento em ecossistema:


Market size x Market share

Esse tipo de gráfico pode ser utilizado para entender o posicionamento de


uma empresa não só em relação às concorrentes do mesmo mercado,
mas também em relação ao ecossistema em que está inserida.

Para realizar a análise desse gráfico é necessário não só observar os eixos,


como também a área ocupada por cada empresa para compreender o
tamanho delas dentro de todo o ecossistema.

Essa análise mostra, por exemplo, como um market share pequeno em


certo setor pode ser mais vantajoso do que um grande em outro, por
conta dos tamanhos diferentes entre eles.

209
14ª edição do Programa de Preparação - UFRJ Consulting Club Apostila

● Gráfico de exigência do cliente:


Exigência x (Critério / Empresa)

Gráficos como esse têm como função demonstrar o quanto uma empresa,
em relação a outras, está cumprindo com as exigências do consumidor.

Para isso, são elencados os critérios de avaliação e, através das barras, é


demonstrada a proporção de importância de cada um deles. Com isso, é
colocado o desempenho de cada firma com uma linha que oscila de
acordo com o critério.

No gráfico acima, por exemplo, pode-se notar que a empresa 1 possui o


melhor desempenho no critério A, que é o mais importante para o cliente.
Além disso, a mesma empresa apresenta o pior desempenho no critério B,
o segundo mais importante, e, novamente, o melhor desempenho no C, o
menos relevante dos três.

210
14ª edição do Programa de Preparação - UFRJ Consulting Club Apostila

● Gráfico de desempenho no mercado:


Market share relativo x Lucro

Por meio desse gráfico, é possível identificar se a performance da empresa


está coerente com a posição dela no mercado. Assim, é possível identificar
problemas internos e estratégias externas, para, então, traçar planos de
ação.

A faixa de referência sinalizada no gráfico é onde, segundo estudos de


padrões de empresas, a firma deveria estar posicionada. O RMS,
representado no eixo x, significa Relative Market Share.

● Para empresas que não são líderes de mercado, esse valor


corresponde ao quanto do mercado ela detém em relação à líder:

𝑚𝑎𝑟𝑘𝑒𝑡 𝑠ℎ𝑎𝑟𝑒
𝑚𝑎𝑟𝑘𝑒𝑡 𝑠ℎ𝑎𝑟𝑒 𝑑𝑎 𝑙í𝑑𝑒𝑟

● Para empresas que são líderes, aplica-se a mesma lógica, porém em


relação ao segundo maior player:

𝑚𝑎𝑟𝑘𝑒𝑡 𝑠ℎ𝑎𝑟𝑒 𝑑𝑎 𝑙í𝑑𝑒𝑟


𝑚𝑎𝑟𝑘𝑒𝑡 𝑠ℎ𝑎𝑟𝑒 𝑑𝑜 𝑠𝑒𝑔𝑢𝑛𝑑𝑜 𝑙𝑢𝑔𝑎𝑟

211
14ª edição do Programa de Preparação - UFRJ Consulting Club Apostila

● Gráfico de comparação de custos:


Empresas x Custo

Com essa ferramenta, é possível comparar a estrutura de custos entre


diferentes empresas, identificando a participação de cada categoria no
custo total. Além disso, pode-se comparar tanto esses custos individuais
como o total com os de outras firmas de interesse.

5.4. Dicas gerais

Ao resolver um case, é importante estar sempre atento a diferentes fatores, como:

● Manter a comunicação constante, de modo que o avaliador possa acompanhar


toda a estruturação e resolução do case. O objetivo principal da entrevista é
entender qual a sua linha de raciocínio, assim como sua habilidade de
comunicação, então não deixe de explicar cada passo realizado de forma clara e
organizada. Isso é crucial, por exemplo, ao explicar porque você quer uma
determinada informação nas perguntas esclarecedoras, assim como descrever
suas contas e diversas outras situações.

212
14ª edição do Programa de Preparação - UFRJ Consulting Club Apostila

● Ser answers first, ou seja, é importante estruturar seu pensamento de forma


objetiva e, primeiro, falar o ponto principal de sua análise, para, depois, explicar
os motivos que te levaram até ele.

● Validar as premissas, contas e análises, evitando chegar a conclusões erradas


ou perder tempo em coisas desnecessárias. Quanto mais cedo o avaliador puder
identificar um erro ou invalidar algo que o candidato fez, mais tempo ele terá
disponível para encontrar o caminho certo e chegar na resposta do case. Sendo
assim, é importante sempre buscar confirmar os passos tomados na entrevista e
nunca levar aspectos externos de mercado como premissas sem antes validá-los
com o avaliador.

● Fazer anotações organizadas, para que você consiga seguir um raciocínio claro
e lógico, facilitando o entendimento do entrevistador e evitando com que você
se perca em meio as informações obtidas ao longo do case. Uma boa estratégia
é utilizar um rascunho para contas e outros para informações e análises, além de
circular ou destacar de alguma forma aquilo que considerar mais importante.

● Pedir tempo quando for necessário. Muitas vezes, os candidatos se sentem


pressionados a não deixar momentos de silêncio e acabam tendo pressa em
resolver o case, prejudicando o resultado. Apesar da comunicação ser
extremamente importante, isso não deve impedir que os entrevistados tirem um
tempo para estruturar sua estratégia de resolução ou ler um gráfico mais
complicado, desde que deixem claro o que pretendem realizar nesse tempo e
evitem tomar mais de 2 minutos. Os principais momentos em que esse tempo
deve ser pedido são: estruturação inicial e recomendação final.

● Treinar diferentes tipos de case, buscando aprender a se adaptar a novas


situações e exercitar diversos tipos de análises. Um ponto muito importante é
que o foco dos treinos deve ser a qualidade das resoluções, e não a velocidade,
visto que é crucial ficar bom antes de buscar ficar rápido. Além disso, é
recomendado que procure realizar esses treinos com diferentes pessoas, visto
que cada aplicação e feedback é único.

● Realizar as contas sem calculadora e com calma, evitando erros de contas


mais básicas e buscando ganhar prática e velocidade para otimizar o tempo.

213
14ª edição do Programa de Preparação - UFRJ Consulting Club Apostila

Outras estratégias para otimizar as contas, como as citadas nas explicações de


GMAT, podem ser utilizadas nesse momento.

● Simplificar os cálculos, evitando perder tempo fazendo contas muito


complicadas e arredondando os valores. Lembre-se que é imprescindível que o
arredondamento seja validado com o entrevistador antes de utilizado. Além
disso, se, para uma variável, você arredondou para cima, pense em arredondar
para baixo alguma outra, a fim de aumentar a precisão nas suas contas.

● Olhar para o framework utilizado, assim como o brainstorming e todas as


análises e contas realizadas para pensar nos riscos e próximos passos. Analisar
rigorosamente cada ponto é essencial para garantir que nada será esquecido.

● Ser coachable, ou seja, estar aberto a mudanças e saber tocar o case. É


extremamente importante entender qual a realidade do case, quais informações
estão disponíveis e quais devem ser consideradas irrelevantes. Um bom
candidato sabe lidar com contratempos e entende o momento de buscar
analisar o case por outro ângulo quando a visão idealizada não está levando a
nenhuma conclusão. Por fim, é essencial que o entrevistado tenha iniciativa para
guiar o case com confiança e demonstre a capacidade de definir o que pretende
fazer ao longo da entrevista.

214
14ª edição do Programa de Preparação - UFRJ Consulting Club Apostila

Conclusão
Chegando ao fim desta apostila, gostaríamos de agradecer pelo seu interesse e
participação no Programa de Preparação, pelo qual, em suas 12 edições, já passaram
centenas de alunos com o mesmo objetivo que você: serem aprovados nos processos
seletivos das grandes consultorias, vistos como uns dos mais difíceis do mundo.

Esperamos que os conceitos apresentados aqui e nas aulas complementem a sua


preparação e ajudem você a sentir mais segurança para encarar esse desafio. Mais
ainda, torcemos para que, se você busca sua vaga em consultoria estratégica, você
consiga alcançar esse sonho e se torne um profissional não só qualificado, mas,
principalmente, apaixonado pelo que faz.

Equipe do Programa de Preparação 23.2


UFRJ Consulting Club

"O futuro é brilhante!"

215

Você também pode gostar